Você está na página 1de 189

ITA 2023

SOLUÇÕES

AULA 10
Soluções

Prof. Thiago Cardoso

www.estrategiamilitares.com.br
Prof. Thiago Cardoso

Sumário
Apresentação da Aula 5

1. Coeficiente de Solubilidade 5

2. Medidas de Concentração 9

2.1. Concentração Comum ou em Massa 10

2.2. Concentração em Mol/L 12

2.3. Concentração Molal ou Molalidade 13

2.4. Título 13
2.4.1. Água Oxigenada 15

2.5. Soluções Ideais e não-Ideais 17

2.6. Lei da Diluição 19

3.Titulação 23

3.1. Equipamentos de Laboratório 24

3.2. Curva de PH 25
3.2.1. Ácido Forte com Base Forte 25
3.2.1. Titulação de uma Solução de Ácido Fraco por uma Solução de Base Forte 28

3.3. Indicadores de PH 31

4. Propriedades Coligativas 35

4.1. Efeito Tonoscópico 35

4.2. Efeitos Ebulioscópico e Crioscópico 39

4.3. Pressão Osmótica 42

4.4. Determinação de Massas Molares 46

4.5. Condutividade Elétrica de Soluções Aquosas 49


4.5.1. Mobilidade Iônica 50
4.5.2. Condutividade de Soluções Concentradas 52

5. Lista de Questões Propostas 53

6. Gabarito 89

7. Lista de Questões Comentadas 91

7.1. Lista de Questões Nível I 91

7.2. Lista de Questões Nível II 123

AULA 10 – SOLUÇÕES 2
Prof. Thiago Cardoso

8. Considerações Finais 189

AULA 10 – SOLUÇÕES 3
Prof. Thiago Cardoso

Siga minhas redes sociais!

Prof. Thiago Cardoso @thiagofernando.pe

AULA 10 – SOLUÇÕES 4
Prof. Thiago Cardoso

Apresentação da Aula
Nessa aula, vamos falar sobre as Soluções, com foco nas medidas de concentração e nas
Propriedades Coligativas. Também aproveitaremos o final da aula para comentar sobre as Reações de
Oxirredução.
Devemos ter em mente que, nesse capítulo, estamos falando das soluções verdadeiras, que são
misturas homogêneas, de maneira geral, transparentes.
Já vimos que a classificação entre solução, dispersão e suspensão depende do tamanho das
partículas dispersas no solvente.

Homogênea
< 1 nm Solução
Partículas Dispersas

Transparente
Tamanho das

Heterogênea
1 a 100 nm Dispersão
Translúcida ou
Opaca

Heterogênea
> 100 nm Suspensão
Opaca

Figura 1: Classificação das Misturas de acordo com o tamanho das partículas dispersas

1. Coeficiente de Solubilidade
O coeficiente de solubilidade diz respeito a o quanto de um determinado soluto é possível
dissolver em uma determinada quantidade de solvente.
É normalmente expresso em unidade de massa por volume.

𝑚á𝑥𝑖𝑚𝑎 𝑞𝑢𝑎𝑛𝑡𝑖𝑑𝑎𝑑𝑒 𝑑𝑒 𝑠𝑜𝑙𝑢𝑡𝑜 𝑞𝑢𝑒 é 𝑝𝑜𝑠𝑠í𝑣𝑒𝑙 𝑑𝑖𝑠𝑠𝑜𝑙𝑣𝑒𝑟


𝐶=
𝑞𝑢𝑎𝑛𝑡𝑖𝑑𝑎𝑑𝑒 𝑑𝑒 𝑠𝑜𝑙𝑣𝑒𝑛𝑡𝑒

O coeficiente de solubilidade varia com a temperatura.


Por ser possível dissolver, devemos entender formando uma solução estável. São duas palavras
importantes:
• Solução: presume a formação de uma mistura homogênea. Portanto, não pode apresentar sinais
de turvidez;

AULA 10 – SOLUÇÕES 5
Prof. Thiago Cardoso

• Estável: a mistura não pode se decompor por decantação, por agitação nem por outro
procedimento físico.
Por exemplo, o coeficiente de solubilidade do óxido de magnésio é 0,86 mg/L de água. Isso significa
que, em 1 L de água, é possível dissolver apenas 0,86 mg de óxido de magnésio.
Se tentarmos dissolver menos, forma-se uma solução insaturada. Nas soluções insaturadas, existe
uma quantidade de soluto inferior ao coeficiente de solubilidade. Portanto, é possível dissolver ainda mais
soluto.

Figura 2: Solução Saturada e Insaturada

Por outro lado, se tentarmos dissolver 3 mg/L de óxido de magnésio em água, o que acontecerá é
que somente 0,86 mg/L se dissolverá. Todo o restante do óxido de magnésio permanecerá no fundo do
recipiente, na sua forma cristalina, o que é denominado corpo de fundo.
A presença de corpo de fundo é uma prova de que a solução acima é saturada.
Podemos visualizar a formação de corpo de fundo também quando tentamos dissolver açúcar
demais na água, por exemplo, ao fazer um suco. Se você pegar um copo de água e começar a acrescentar
açúcar, chegará um momento em que você não vai mais conseguir dissolver e parte do soluto se
depositará no fundo do copo.
É interessante observar que tanto o excesso de açúcar com o excesso de óxido de magnésio pode
ser agitado na água. A agitação permitirá a dispersão, não a dissolução do excedente que não estava
dissolvido.

AULA 10 – SOLUÇÕES 6
Prof. Thiago Cardoso

Figura 3: Agitação de uma Solução Saturada com Corpo de Fundo

Ao agitar uma solução saturada com corpo de fundo, obtém-se uma dispersão turva, não uma
solução homogênea. Essa dispersão rapidamente decanta, se deixada em repouso, formando a solução
saturada estável com corpo de fundo.
É interessante observar que, em alguns casos, é possível formar uma solução supersaturada.
Normalmente, essas soluções são formadas pelo lento resfriamento de soluções mais concentradas.
Como o coeficiente de solubilidade varia com a temperatura, é possível, que a 50 °C, seja possível
dissolver 5 mg/L de uma determinada substância em água, mas que, à temperatura de 25 °C, a
solubilidade tenha caído para 3 mg/L.
Se a solução 5 mg/L for resfriada lentamente, ela pode permanecer homogênea e transparente.
Porém, é importante observar que a solução supersaturada não é estável. Ela se decompõe
violentamente quando levemente agitada ou quando é depositado um pequeno cristal do soluto.
Por conta disso, as soluções supersaturadas não podem conter nenhum corpo de fundo, porque a
presença de um minúsculo cristal do soluto é suficiente para desencadear a precipitação do excedente
dissolvido.
Sendo assim, repetimos que a presença de corpo de fundo é realmente uma prova de que a
solução acima é saturada.

(Estratégia Militares – TFC - Inédita)


Sabendo-se que a solubilidade do cloreto de sódio em água é igual a 40 g/L, determine se as soluções
formadas são saturadas, insaturadas ou supersaturadas:
a) 3g de cloreto de sódio e 100 mL de água
b) 2g de cloreto de sódio e 50 mL de água
c) 2 mol de cloreto de sódio e 3 L de água

AULA 10 – SOLUÇÕES 7
Prof. Thiago Cardoso

d) 1 mol de cloreto de sódio e 2 L de água


e) 5 g de cloreto de sódio e 100 mL de água
Dados: Na = 23 g/mol, C = 35,5 g/mol

Comentários
Vamos analisar as proposições individualmente.
a)
3
𝐶= = 30 𝑔/𝐿 < 40 𝑔/𝐿
0,1
Como a concentração obtida foi inferior ao coeficiente de solubilidade, a solução formada é
insaturada.
b)
2
𝐶= = 40 𝑔/𝐿
0,05
Como a concentração obtida foi igual ao coeficiente de solubilidade, a solução formada é saturada.
c) Como foi fornecida a quantidade em mol, precisamos
𝑀𝑁𝑎𝐶𝑙 = 1.23 + 1.35,5 = 58,5 𝑔/𝑚𝑜𝑙
Para calcular a massa de cloreto de sódio, precisamos multiplicar o número de mols do sal pela
massa molar.
𝑚𝑁𝑎𝐶𝑙 = 𝑛𝑁𝑎𝐶𝑙 . 𝑀𝑁𝑎𝐶𝑙 = 2.58,5 = 117 𝑔
Agora, vamos calcular a concentração em gramas por litro da solução formada.
117
𝐶= = 39 𝑔/𝐿
3
Como a concentração obtida foi inferior ao coeficiente de solubilidade, a solução formada é
insaturada.
d) Para calcular a massa de cloreto de sódio, precisamos multiplicar o número de mols do sal pela
massa molar.
𝑚𝑁𝑎𝐶𝑙 = 𝑛𝑁𝑎𝐶𝑙 . 𝑀𝑁𝑎𝐶𝑙 = 1.58,5 = 58,5 𝑔
Agora, vamos calcular a concentração em gramas por litro da solução formada.
58,5
𝐶= = 29,25 𝑔/𝐿
2
Como a concentração obtida foi inferior ao coeficiente de solubilidade, a solução formada é
insaturada.
e) Vamos calcular a concentração em grama por litro da solução formada.
5
𝐶= = 50 𝑔/𝐿
0,1
Observe que esse valor é acima do coeficiente de solubilidade. Portanto, haverá um corpo de
fundo e a solução formada será saturada, com a concentração exatamente igual a 40 g/L.

AULA 10 – SOLUÇÕES 8
Prof. Thiago Cardoso

Vale lembrar que as soluções supersaturadas são instáveis e que elas só se formam por
resfriamento lento.
Ao se misturar uma quantidade de NaCl superior ao próprio coeficiente de solubilidade, o excesso
de sal se depositará no fundo do recipiente.
Nesse caso, 4 g se dissolvem formando uma solução saturada (40 g/L) e o restante (1 g) aparece
como corpo de fundo.
Gabarito: 1, 3 e 4 insaturadas; 2 e 5 saturadas

(Estratégia Militares – TFC - Inédita)


A solubilidade de um determinado sal em água é igual a 2 g/L. Dessa forma, ao se misturar 1,5 g de sal
a 500 mL de água, forma-se uma solução:
a) insaturada.
b) saturada sem corpo de fundo.
c) saturada com corpo de fundo.
d) supersaturada.

Comentários
Considerando que a solubilidade do sal é 2 g/L, a sua solubilidade em 500 mL de água seria de
apenas 1 g. Portanto, foi feita uma tentativa de se dissolver mais sal do que o que é possível pelo
coeficiente de solubilidade.
Dessa forma, forma-se uma solução saturada, dissolvendo-se exatamente 1 g do sal, deixando 0,5
g como corpo de fundo, um precipitado não dissolvido.
Portanto, a solução final é saturada com corpo de fundo.
Devemos nos lembrar que as soluções supersaturadas são instáveis e que só se formam por
resfriamento lento.
Gabarito: C

2. Medidas de Concentração
No estudo de soluções, muitas grandezas são utilizadas para avaliar a quantidade de solutos
dissolvidos em uma solução.
As medidas de concentração de uma solução são grandezas intensivas, que indicam a proporção
entre soluto e solvente. Portanto, elas não dependem da quantidade de matéria analisada.
Por exemplo, quando falamos que a concentração de sais no soro fisiológico é 90 mg/L, isso
significa que uma gota de soro fisiológico contém 90 mg/L de sais; e um litro de soro fisiológico contém
exatamente 90 mg/L de sais.
A medida mais utilizada no dia-a-dia é a concentração comum. Porém, as mais importantes para a
Química são a molaridade e a molalidade. Portanto, um dos principais aprendizados que você deve levar

AULA 10 – SOLUÇÕES 9
Prof. Thiago Cardoso

dessa seção é como converter uma unidade de concentração em outra, em especial, a concentração
comum em molaridade, e vice-versa.
Ao longo dessa seção, utilizaremos alguns sub-índices que é útil você conhecer:
• 1: soluto;
• 2: solvente;
• Sem índice: solução como um todo.
No caso de soluções diluídas, o volume da solução é aproximadamente igual ao volume do
solvente.

2.1. Concentração Comum ou em Massa


No dia-a-dia, a forma mais comum de expressar a concentração de um soluto é por meio da razão
entre a massa do soluto e o volume da solução.

𝑚1 𝑚𝑎𝑠𝑠𝑎 𝑑𝑜 𝑠𝑜𝑙𝑢𝑡𝑜
𝐶= =
𝑉 𝑣𝑜𝑙𝑢𝑚𝑒 𝑑𝑎 𝑠𝑜𝑙𝑢çã𝑜

Em soluções aquosas, a concentração comum também é expressa pela abreviatura P/V ou M/V,
que significa peso/volume ou massa/volume. Por exemplo, dizer “uma solução de cloreto 1% (M/V)” é o
mesmo que “uma solução de cloreto de sódio 1 g /100 mL”.
Não devemos confundir a concentração comum com a densidade da solução que é expressa em
termos da razão entre a massa da solução.

𝑚 𝑚𝑎𝑠𝑠𝑎 𝑑𝑎 𝑠𝑜𝑙𝑢çã𝑜
𝑑= =
𝑉 𝑣𝑜𝑙𝑢𝑚𝑒 𝑑𝑎 𝑠𝑜𝑙𝑢çã𝑜

Vamos frisar para você não se esquecer.

Nas soluções aquosas, a unidade mais comum de medida de concentração é o mg/L, em que a
massa do soluto é expressa em miligramas. Nesse caso, ela costuma ser expressa em ppm (ou partes por
milhão).
A razão para isso é que 1 L de água corresponde a aproximadamente 1 kg. Dessa forma, 1 mg do
soluto corresponde a aproximadamente 1 milhonésimo da massa total da solução.

AULA 10 – SOLUÇÕES 10
Prof. Thiago Cardoso

1 𝑚𝑔
1 𝑝𝑝𝑚 =
1𝐿
1 𝑚𝑔 10−3 𝑔 1
1 𝑝𝑝𝑚 = ≅ =
1𝐿 103 𝑔 106

No aquarismo marinho, é fundamental monitorar o nível de diversos parâmetros, como o nitrato


e o fosfato, que interferem no desenvolvimento de corais. Para o cultivo de Acroporas, é essencial manter
os níveis de nitrato (𝑁𝑂3− ) e fosfato (𝑃𝑂43− ) abaixo de 5 mg/L (ou 5 ppm) e 0,06 mg/L (ou 0,06 ppm),
respectivamente. Acima disso, eles perdem a cor natural, ficam marrons e, com o tempo, podem começar
a perder tecido. Essas importantes medidas são feitas recorrendo a testes colorimétricos.

Figura 4: Exemplos de Testes Colorimétricos

A aFigura 4 mostra um exemplo de teste colorimétrico caseiro que mede a concentração dos íons
nitrato e fosfato na água. A colorimetria é o tipo de teste mais utilizado em laboratórios químicos. É por
isso que a prova do ITA coloca aquelas questões chatas em que você tem que saber a cor do reagente. Na
Química, muitas vezes, é a maneira mais fácil de detectar a existência de um determinado composto em
alguma mistura específica.

AULA 10 – SOLUÇÕES 11
Prof. Thiago Cardoso

Figura 5: Efeito do Excesso de Fosfato sobre a Coloração de Acroporas – importância das pequenas concentrações
em sistemas biológicos

A Figura 5 mostra a importância das pequenas concentrações na Química. A evolução do


aquarismo marinho até chegar ao nível de sermos capazes de manter as Acroporas, os corais mais
sensíveis à poluição e ao aumento da temperatura nos mares, só foi possível, porque aprendemos a
controlar níveis baixíssimos de alguns parâmetros da água.
O mesmo é válido para sistemas laboratoriais, em que é preciso sintetizar medicamentos com
altíssima precisão, pois qualquer miligrama a mais ou menos pode tornar a droga ineficiente ou letal ao
seu usuário.

2.2. Concentração em Mol/L


A concentração em mol/L, antigamente chamada de molaridade, é a medida de concentração
mais utilizada na Química. Ela costuma ser representada por uma letra M maiúscula manuscrita ou
colocando a fórmula química do soluto entre colchetes.

𝑛2 𝑛ú𝑚𝑒𝑟𝑜 𝑑𝑒 𝑚𝑜𝑙𝑠 𝑑𝑜 𝑠𝑜𝑙𝑢𝑡𝑜


𝔐 = [𝑋] = =
𝑉 𝑉𝑜𝑙𝑢𝑚𝑒 𝑑𝑎 𝑆𝑜𝑙𝑢çã𝑜

Na Química, essa medida é extremamente importante, porque é utilizada em diversas áreas, como
a Cinética Química para o cálculo de velocidades de reação.
Ela pode ser extraída diretamente da concentração comum em gramas por litro dividindo-se pela
massa molar do soluto.

𝐶
𝔐=
𝑀1

AULA 10 – SOLUÇÕES 12
Prof. Thiago Cardoso

Por exemplo, no exemplo anterior, a concentração do íon nitrato em solução é de 5 mg/L.


Podemos obter a concentração em mol/L desse íon. Para isso, vamos obter o fator de conversão.
𝑀1 = 𝑀𝑎𝑠𝑠𝑎 𝑀𝑜𝑙𝑎𝑟 𝑑𝑒 𝑁𝑂3− = 1.14 + 3.16 = 14 + 48 = 62 𝑔/𝑚𝑜𝑙
5.10−3
𝔐= ≅ 0,08.10−3 = 8.10−5 𝑚𝑜𝑙/𝐿
62
Uma forma comum de representar a molaridade da solução é pelo símbolo M. Por exemplo, a
solução 8.10-5 M é equivalente a 8.10-5 mol/L.
Outro fato importante sobre a molaridade é que, como ela é proporcional ao número de mols do
soluto, ela pode ser utilizada diretamente na proporção estequiométrica. Isso é bastante útil nas
titulações.

2.3. Concentração Molal ou Molalidade


A molalidade ou concentração molal é muito utilizada no estudo das Propriedades Coligativas.
𝑛2 𝑛ú𝑚𝑒𝑟𝑜 𝑑𝑒 𝑚𝑜𝑙𝑠 𝑑𝑜 𝑠𝑜𝑙𝑢𝑡𝑜
𝑊= =
𝑚2 (𝑘𝑔) 𝑚𝑎𝑠𝑠𝑎 𝑑𝑜 𝑠𝑜𝑙𝑣𝑒𝑛𝑡𝑒 (𝑒𝑚 𝑘𝑔)
Nas soluções aquosas diluídas, a massa do solvente é aproximadamente igual à massa da própria
solução. Como a densidade da água é de 1 kg/L, nesses casos, a molalidade será numericamente
aproximadamente igual à concentração em mol/L.
Por exemplo, no caso da solução 8.10-5 mol/L, pode-se considerar que é aproximadamente 8.10-5
molal.
Outro comando que pode ser utilizado é dizer que “1 litro da solução contém aproximadamente 1
kg de solvente”. Ao dizer isso, o enunciado está igualando os denominadores das expressões da
concentração molar e da concentração molal.
Porém, é possível também fazer a conta exata da conversão. Para isso, precisamos saber o fator
de compactação entre o soluto e o solvente. Esse dado normalmente vem de observações experimentais.
Portanto, se não for fornecido na hora da prova, você vai considerar que o volume da solução é igual ao
volume do solvente. Mostraremos um exemplo desse cálculo ainda nesse material.

2.4. Título
O título é uma unidade que pode gastar bastante confusão, porque ela pode se referir a massa,
volume ou número de mols.

𝑚𝑎𝑠𝑠𝑎 𝑑𝑜 𝑠𝑜𝑙𝑢𝑡𝑜
𝜏=
𝑚𝑎𝑠𝑠𝑎 𝑑𝑎 𝑠𝑜𝑙𝑢çã𝑜
𝑣𝑜𝑙𝑢𝑚𝑒 𝑑𝑜 𝑠𝑜𝑙𝑢𝑡𝑜
𝜏=
𝑣𝑜𝑙𝑢𝑚𝑒 𝑑𝑎 𝑠𝑜𝑙𝑢çã𝑜
𝑛ú𝑚𝑒𝑟𝑜 𝑑𝑒 𝑚𝑜𝑙𝑠 𝑑𝑜 𝑠𝑜𝑙𝑢𝑡𝑜
𝜏=
𝑛ú𝑚𝑒𝑟𝑜 𝑑𝑒 𝑚𝑜𝑙𝑠 𝑑𝑎 𝑠𝑜𝑙𝑢çã𝑜

O título em massa é também referido pela notação P/P ou M/M. Por exemplo, uma solução de
etanol 10% (M/M) é equivalente a uma solução de etanol 10% em massa.

AULA 10 – SOLUÇÕES 13
Prof. Thiago Cardoso

Analogamente, o título em volume também pode ser referido pela notação V/V. Por exemplo, uma
solução de etanol 10% (V/V) é o mesmo que uma solução de etanol 10% em volume.
É interessante citar que nem sempre o volume da solução é igual à soma dos volumes do soluto e
do solvente. Veremos mais adiante que o processo de dissolução pode acontecer com redução de volume.
Nesses casos, não podemos dizer que o título é expresso pela razão a seguir:
𝑉1
𝜏≠
𝑉1 + 𝑉2
Uma das formas mais comuns de se registrar o título é em ppm, que significa “partes por milhão”.
Para isso, basta multiplicar por 1 milhão ou 106.

𝑚𝑎𝑠𝑠𝑎 𝑑𝑜 𝑠𝑜𝑙𝑢𝑡𝑜
𝐶= (× 106 ) [𝑝𝑝𝑚]
𝑚𝑎𝑠𝑠𝑎 𝑑𝑎 𝑠𝑜𝑙𝑢çã𝑜
𝑣𝑜𝑙𝑢𝑚𝑒 𝑑𝑜 𝑠𝑜𝑙𝑢𝑡𝑜
𝐶= (× 106 ) [𝑝𝑝𝑚]
𝑣𝑜𝑙𝑢𝑚𝑒 𝑑𝑎 𝑠𝑜𝑙𝑢çã𝑜
𝑛ú𝑚𝑒𝑟𝑜 𝑑𝑒 𝑚𝑜𝑙𝑠 𝑑𝑜 𝑠𝑜𝑙𝑢𝑡𝑜
𝐶= (× 106 ) [𝑝𝑝𝑚]
𝑛ú𝑚𝑒𝑟𝑜 𝑑𝑒 𝑚𝑜𝑙𝑠 𝑑𝑎 𝑠𝑜𝑙𝑢çã𝑜

Em alguns casos, também se utiliza ppt como “partes por mil” e o ppb como “partes por bilhão”.
Nesses caso, basta multiplicar a expressão do título por 10³ e por 109, respectivamente, em substituição
ao 106 do ppm.
Infelizmente, é muito comum que as questões de prova apresentem a medida de título,
frequentemente a concentração em ppm, mas não digam se ela se refere a massa, volume ou número de
mols. Por isso, eu vou te apresentar de acordo com a minha experiência o que você deve levar em
consideração, caso o enunciado nada diga. Sendo assim, no caso de:
• Uma solução de um sólido em água, suponha que o ppm se refere à razão entre massas;
• Uma mistura de dois gases, suponha que o ppm se refere à razão entre os números de mols.

Em soluções aquosas diluídas, utiliza-se bastante a medida em ppm como a razão entre as massas
de soluto e solvente utilizadas.
1 ppm significa 1 parte por milhão. No caso específico de soluções aquosas diluídas, 1 ppm será
aproximadamente igual a 1 mg/L.
A razão para isso é que o volume da solução é aproximadamente igual ao volume do solvente.
Sendo assim, em 1 litro de solução, teremos aproximadamente 1 quilograma de água, que é igual a 1
milhão de miligramas. Se esse volume tiver aproximadamente 1 mg de soluto, de fato, teremos para cada
1 milhão de miligramas da solução, 1 parte do soluto.
Portanto, quando falamos que a água possui 5 ppm de nitrato, é basicamente o mesmo de falar
que a concentração do íon nitrato é 5 mg/L. Sendo assim, 1 L da solução apresenta 5 mg do íon.
Por outro lado, no caso de misturas de água e álcool, são usadas duas formas de medição do título:
• °INPM: refere-se ao título em massa do etanol na mistura. É um padrão do Instituto
Nacional de Pesos e Medidas, sendo bastante utilizado no Brasil. 1 °INPM significa 1% de
etanol em massa.
• °GL: ou graus Gay-Lussac, refere-se ao título em volume do etanol na mistura. 1 °GL
significa 1% de etanol em volume.

AULA 10 – SOLUÇÕES 14
Prof. Thiago Cardoso

Em questões de prova, recomendamos que você considere que o percentual de álcool se refere
ao seu percentual em volume. Por exemplo, no caso de uma bebida destilada de teor 40%, isso significa
que 1 litro dela ela é feita com 400 mL de álcool e 600 mL de água, totalizando aproximadamente 1000
mL de solução.
Porém, é bastante importante você conhecer o °INPM, pois é uma medida bastante utilizada no
Brasil e que pode vir a ser cobrada.

O álcool de limpeza é vendido na forma de solução 46,2 °INMP (ou 46% em massa) de etanol
(C2H6O), também conhecida como solução 54 °GL (ou 54% em volume).
Porém, o álcool em gel utilizado como antisséptico ou bactericida é uma solução 70 °INPM (70%
em massa de etanol) ou 77 °GL (77% em volume de etanol). No Brasil, o álcool acima de 46 °INPM somente
pode ser comercializado na forma de gel, porque, como essa substância é bastante inflamável, os riscos
de explosão na sua manipulação no estado líquido por pessoas comuns é grande.
O álcool de limpeza comum 46,2% (°INPM) tem pouco efeito bactericida, não sendo útil para esse
fim.
Com o aumento do teor alcóolico, em 70%, a solução passa a ter um pronunciado efeito
bactericida. É interessante que o poder bactericida dessa solução é maior do que até mesmo o poder do
álcool absoluto ou quase puro.
O provável efeito bactericida do álcool é que, como ele é bastante higroscópico, ou seja, absorve
água com facilidade, ele adere às membranas celulares, matando as bactérias por desidratação. Também
acontece que, ao penetrar no citoplasma celular, o álcool desnatura as proteínas bacterianas e provoca a
coagulação de diversas enzimas necessárias ao seu metabolismo.
Quando se utiliza o etanol absoluto (99,6%), o efeito bactericida é menor do que o do álcool 70%.
Uma das explicações mais comuns para esse fato é que a presença de um pouco da água na mistura facilita
a absorção do álcool pelas bactérias.
Isso acontece, porque as próprias bactérias possuem afinidade pela água, então, elas absorvem
mais da solução quando encontram água do que quando encontram o etanol absoluto. Outra possível
explicação é que o etanol absoluto rapidamente se volatiliza, enquanto que o álcool 70% permanece mais
tempo no estado líquido, permitindo maior tempo de contato.
O álcool 70 é bastante eficiente contra bactérias vegetativas, fungos e vírus (envelopados, com o
vírus influenza H1N1). Porém, os esporos bacterianos são mais resistentes.

2.4.1. Água Oxigenada


A água oxigenada é uma solução aquosa de peróxido de hidrogênio (H2O2), que é um poderoso
agente oxidante.
O peróxido de hidrogênio se decompõe na presença de luz ou de enzimas, como a catalase
liberando oxigênio nascente.

AULA 10 – SOLUÇÕES 15
Prof. Thiago Cardoso

𝑙𝑢𝑧 𝑜𝑢 𝑐𝑎𝑡𝑎𝑙𝑎𝑠𝑒
𝐻2 𝑂2 (𝑎𝑞) → 𝐻2 𝑂 (𝑙) + [𝑂]
O oxigênio nascente nada mais é do que um átomo de oxigênio isolado, que é um radical livre
extremamente reativo, com altíssimo poder oxidante, sendo capaz de reagir com basicamente qualquer
molécula que encontre na sua frente, inclusive as proteínas e gorduras que formam a membrana celular
de bactérias.
Por conta disso, a água oxigenada é um poderoso bactericida, que é bastante utilizado como
antisséptico no tratamento de feridas.
Além disso, ela pode ser bem mais útil. Por exemplo, se você mergulhar suas escolhas de dente
em uma solução de água oxigenada, elas ficarão livres de coliformes fecais e você poderá utilizá-la com
mais segurança.
É importante citar que, se não encontrar outra molécula, os átomos de oxigênio nascente se
combinam para formar uma molécula de oxigênio (O2). Portanto, a decomposição da água oxigenada
pode ser escrita genericamente como:
𝑙𝑢𝑧 𝑜𝑢 𝑐𝑎𝑡𝑎𝑙𝑎𝑠𝑒
2𝐻2 𝑂2 (𝑎𝑞) → 𝐻2 𝑂 (𝑙) + 𝑂2 (𝑔)

Uma medida comum de concentração da água oxigenada é pela quantidade de volumes, que é, na
verdade, uma razão entre o volume de oxigênio liberado e o volume da solução.

𝑉𝑂2
𝑥=
𝑉𝑠𝑜𝑙𝑢çã𝑜

O volume de O2 é considerado nas CNTP (sigla para as condições normais de temperatura e


pressão), que corresponde à temperatura de 0 °C e pressão de 1 atm. Nessas condições, o volume de 1
mol de qualquer gás é igual a:
𝑉 = 22,4 𝐿/𝑚𝑜𝑙
Por exemplo, um frasco de água oxigenada 11,2 volumes significa que 1 litro dessa solução
produzirá 11,2 litros de oxigênio molecular nas CNTP. Com base nisso, podemos calcular o número de
mols de gás produzido.
11,2
𝑛𝑂2 = = 0,5 𝑚𝑜𝑙
22,4
Com base na estequiometria da decomposição do peróxido de hidrogênio, podemos calcular no
número de mols presentes na solução desse composto.
𝑙𝑢𝑧 𝑜𝑢 𝑐𝑎𝑡𝑎𝑙𝑎𝑠𝑒
2𝐻2 𝑂2 (𝑎𝑞) → 𝐻2 𝑂 (𝑙) + 𝑂2 (𝑔)
𝑛𝐻2 𝑂2 𝑛𝑂2
= ∴ 𝑛𝐻2 𝑂2 = 2. 𝑛𝑂2 = 2.0,5 = 1 𝑚𝑜𝑙
2 1
Agora, somos capazes de calcular a concentração molar.
𝑛 1
[𝐻2 𝑂2 (𝑎𝑞)] = = = 1 𝑚𝑜𝑙/𝐿
𝑉 1

AULA 10 – SOLUÇÕES 16
Prof. Thiago Cardoso

2.5. Soluções Ideais e não-Ideais


Prestemos atenção ao conceito de solução ideal.

Uma solução ideal é aquela em que as interações intermoleculares soluto-solvente são muito
semelhantes às interações soluto-soluto e solvente-solvente. Para isso, é necessário que as moléculas do
soluto e do solvente sejam muito parecidas entre si.
Não existe uma solução completamente ideal, porém, algumas misturas que se aproximam
razoavelmente desse modelo. Como exemplo, podemos citar: benzeno e tolueno; água e etanol.

O volume de uma solução ideal é exatamente igual à soma do volume do soluto com o volume do
solvente separados. Além disso, não ocorre nenhum efeito térmico – liberação ou absorção de calor –
durante a dissolução.

Explicaremos agora o efeito da variação de volume em soluções não-ideias. Esse é um efeito pouco
comentado, mas que gostaria de abordar nesse material, a fim de que você esteja à frente de sua
concorrência. Tenho certeza de que uma questão sobre esse efeito traria bastantes dúvidas entre os
alunos.

Em regra, a dissolução de um soluto na água provoca redução do volume total. Foi o que
aconteceu na mistura em apreço.
Na preparação de água salgada, a produção de 1 L requer o uso de 40 gramas de sal marinho. Com
isso, obtém-se uma mistura de densidade igual a 1,025 g/cm³. A medida da densidade é a mais utilizada
por aquaristas marinhos para avaliar a concentração de sal no aquário.
Com isso, podemos calcular a massa de água que foi efetivamente utilizada na solução.
Primeiramente, calcularemos a massa de 1 litro da solução por meio da densidade.
𝑚 = 𝑑𝑉 = 1,025.1000 = 1025 𝑔
Em 1 litro da solução, temos a massa de 1025 g, dos quais já sabemos que 40 gramas se devem ao
sal.
𝑚á𝑔𝑢𝑎 = 𝑚𝑠𝑜𝑙𝑢çã𝑜 − 𝑚𝑠𝑜𝑙𝑢𝑡𝑜
𝑚á𝑔𝑢𝑎 = 1025 − 40 = 985 𝑔
A massa de água presente em 1 litro da solução é, portanto, igual a 985 g ou 0,985 kg. Por conta
disso, a molalidade real da solução é ligeiramente diferente da sua concentração molar.

AULA 10 – SOLUÇÕES 17
Prof. Thiago Cardoso

8.10−5
𝑊= = 8,1.10−5 𝑚𝑜𝑙𝑎𝑙
0,985
Para calcular e mostrar a existência dessa redução de volume, precisamos de dados sobre as
densidades da água pura e do sal puro, que são:
• Densidade da Água Pura: 1,00 g/cm³
• Densidade do Sal Puro: 2,16 g/cm³
Podemos obter uma expressão para o volume em função da massa e da densidade.
𝑚 𝑚 𝑚𝑎𝑠𝑠𝑎
𝑑= ∴𝑉= ∴𝑉=
𝑉 𝑑 𝑑𝑒𝑛𝑠𝑖𝑑𝑎𝑑𝑒
Com base nessa expressão, podemos calcular as massas
985
𝑉𝐻2 𝑂 = = 985 𝑚𝐿
1,00
40
𝑉𝑠𝑎𝑙 = = 18,5 𝑚𝐿
2,16
O volume total da água pura e do sal puro, antes da dissolução é, portanto:
𝑉𝑎𝑛𝑡𝑒𝑠 = 985 + 18,5 = 1003,5 𝑚𝐿
Depois da dissolução, o volume passou a ser de 1000 mL, registrando-se, portanto, uma redução
de 3,5 mL de volume. Em termos percentuais, a redução de volume na produção de água salgada a partir
de sal marinho é de apenas 0,3%.
O efeito da redução de volume se deve ao fato de que as interações entre a água e o sal são mais
intensas do que as próprias interações soluto-soluto e solvente-solvente. O fato de que a mistura soluto-
solvente apresenta interações mais fortes não só possibilita a dissolução do soluto, mas também contribui
para que o volume total da solução seja menor que a soma dos volumes de seus componentes separados.
Com o conhecimento da densidade da solução, podemos também calcular a diferença entre o
título em ppm e a concentração em mg/L. A água de um aquário marinho tem densidade de cerca de
1,025 g/mL, o que representa um desvio de 2,5% em relação à água destilada.
Considerando essa informação, podemos calcular o valor real da concentração de 5 mg/L de
nitrato em ppm. Para isso, utilizamos a expressão da concentração comum.
𝑚1
𝐶=
𝑉
Em 1 litro de solução, temos:
𝑚1
5= ∴ 𝑚1 = 5.1 = 5 𝑚𝑔
1
A massa real da solução é igual ao produto do seu volume pela densidade.
𝑚 = 𝑑𝑉 = 1,025.1000 = 1025 𝑔 = 1,025.106 𝑚𝑔
Portanto, a concentração em ppm será:
5 6
5
𝑇= . 10 = = 4,87 𝑝𝑝𝑚
1,025.106 1,025
Em algumas situações práticas, a diferença entre 5 ppm e 4,87 ppm pode ser irrelevante, como no
caso do aquarismo marinho. Em outras situações, pode não ser, como no caso da fabricação de um
medicamento, em que as porções de cada substância devem ser controladas rigidamente.

AULA 10 – SOLUÇÕES 18
Prof. Thiago Cardoso

Na Química, o uso de aproximações deve ser feita sempre com bastante atenção à necessidade de
precisão exigida pela aplicação.

2.6. Lei da Diluição


A diluição consiste em adicionar mais solvente, de modo a diminuir a concentração (molar ou em
massa) do solvente.
Popularmente, é o que fazemos quando preparamos o suco concentrado. Esse tipo de suco é
vendido em pequenas embalagens, por exemplo, 500 mL. O modo de preparação consiste simplesmente
em adicionar água até atingir o volume sugerido na embalagem, por exemplo, 2 litros.
Após adicionar água, o suco se tornou diluído. Isso acontece, porque a mesma quantidade de
soluto é dissolvida numa quantidade maior de solvente. A diluição pode ser visualizada, por exemplo, na
redução da intensidade da cor.
O processo de diluição é muito útil no dia a dia e também no laboratório e, por isso, também
aparece muito em questões. Portanto, precisamos saber a famosa Lei da Diluição, que estabelece que a
concentração será inversamente proporcional ao volume da solução. Matematicamente, podemos
escrever:
[𝑿]𝟎 𝑽𝟎 = [𝑿]𝒇 𝑽𝒇
A razão para essa igualdade é que o número de mols do soluto permanece constante durante a
diluição. Somente o solvente é adicionado. E o número de mols do soluto pode ser calculado como o
produto entre a concentração molar e o volume da solução.
𝑛𝑋 = [𝑋]0 𝑉0 = [𝑋]𝑓 𝑉𝑓
Suponha que temos 100 mL de uma solução aquosa de cloreto de sódio, cuja concentração inicial
é igual a 1 mol/L. O que aconteceria com a concentração da solução, se fosse acrescentado mais 400 mL
de solvente?
Nesse caso, note que o volume total da solução passou a 500 mL. Com base nisso, podemos usar
a lei da diluição.
1.0,100 = [𝑋]𝑓 . 0,500
1.0,100
∴ [𝑋]𝑓 = = 0,200 𝑚𝑜𝑙/𝐿
0,500
Observe que a mesma relação é válida para as concentrações em massa.
𝑪 𝟎 𝑽𝟎 = 𝑪 𝒇 𝑽𝒇
Nesse caso, a explicação é que os produtos das concentrações em massa pelo volume da solução
são iguais à massa de soluto. Como, na diluição, a quantidade de soluto se mantém inalterada, concluímos
que os produtos da concentração da solução pelo seu volume permanece também constante.
𝑚𝑋 = [𝑋]0 𝑉0 = [𝑋]𝑓 𝑉𝑓
Dessa forma, guarde em mente que, numa diluição, em que a quantidade de soluto não é alterada,
a concentração da solução é inversamente proporcional ao seu volume.

AULA 10 – SOLUÇÕES 19
Prof. Thiago Cardoso

(Estratégia Militares – TFC - Inédita)


Converta as seguintes concentrações de g/L para mol/L
a) 53 mg/L de carbonato de sódio
b) 88 mg/L de dióxido de carbono
c) 1 g/L de nitrato de potássio
d) 11,7 g/L de cloreto de sódio
Dados de Massas Atômicas: Na = 23 g/mol, K = 39 g/mol, C = 12 g/mol, N = 14 g/mol, O = 16 g/mol, C
= 35,5 g/mol.

Comentários
Devemos nos lembrar que, para converter de concentração em massa para concentração molar,
devemos dividir pela massa molar.
a) Calculemos a massa molar do carbonato de sódio (Na2CO3).
𝑀𝑁𝑎2 𝐶𝑂3 = 2.23 + 1.12 + 3.16 = 106 𝑔/𝑚𝑜𝑙
𝐶 53.10−3 1
[𝑁𝑎2 𝐶𝑂3 ] = = = ⋅ 10−3 = 5.10−4 𝑚𝑜𝑙/𝐿
𝑀 106 2
b) Calculemos a massa molar do dióxido de carbono (CO2).
𝑀𝐶𝑂2 = 1.12 + 2.16 = 44 𝑔/𝑚𝑜𝑙
𝐶 88.10−3
[𝐶𝑂2 ] == = 2 ⋅ 10−3 𝑚𝑜𝑙/𝐿
𝑀 44
c) Calculemos a massa molar do nitrato de potássio (KNO3).
𝑀𝐾𝑁𝑂3 = 1.39 + 1.14 + 3.16 = 101 𝑔/𝑚𝑜𝑙
𝐶 1
[𝐾𝑁𝑂3 ] = = ≅ 0,0099 𝑚𝑜𝑙/𝐿
𝑀 101
d) Calculemos a massa molar do cloreto de sódio (NaCl).
𝑀𝑁𝑎𝐶𝑙 = 1.23 + 1.35,5 = 58,5 𝑔/𝑚𝑜𝑙
𝐶 11,7
[𝑁𝑎𝐶𝑙] = = = 0,2 𝑚𝑜𝑙/𝐿
𝑀 58,5

Gabarito: a) 5.10–4 mol/L; b) 2.10–3 mol/L; c) 9,9.10–3 mol/L; d) 0,2 mol/L

AULA 10 – SOLUÇÕES 20
Prof. Thiago Cardoso

(Estratégia Militares – TFC - Inédita)


Converta as seguintes concentrações de mol/L para g/L
a) 0,025 mol/L de hidrogenocarbonato de sódio
b) 4.10–3 mol/L de nitrito de cálcio
c) 5.10–5 mol/L de fosfato de cálcio
d) 0,4 mol/L de cloreto de potássio
Dados de Massas Atômicas: Na = 23 g/mol, K = 39 g/mol, C = 12 g/mol, N = 14 g/mol, O = 16 g/mol, C
= 35,5 g/mol, H = 1 g/mol, Ca = 40 g/mol, P = 31 g/mol.

Comentários
Para fazer a conversão de concentração molar em concentração de massa, basta multiplicar pela
massa molar.
a) Calculemos a massa molar do hidrogenocarbonato de sódio (NaHCO3).
𝑀𝑁𝑎𝐻𝐶𝑂3 = 1.23 + 1.1 + 1.12 + 3.16 = 84 𝑔/𝑚𝑜𝑙
𝐶 = [𝑁𝑎𝐻𝐶𝑂3 ]. 𝑀𝑁𝑎𝐻𝐶𝑂3 = 0,025.84 = 2,1 𝑔/𝐿
b) Calculemos a massa molar do nitrito de cálcio (Ca(NO2)2).
𝑀𝐶𝑎(𝑁𝑂2)2 = 1.40 + 2. [1.14 + 2.16] = 132 𝑔/𝑚𝑜𝑙
𝐶 = [𝐶𝑎(𝑁𝑂2 )2 ]. 𝑀𝐶𝑎(𝑁𝑂2)2 = 4.10−3 . 132 = 528.10−3 𝑔/𝐿
𝐶 = 528 𝑚𝑔/𝐿
c) Calculemos a massa molar do fosfato de cálcio (Ca3(PO4)2).
𝑀𝐶𝑎3(𝑃𝑂4)2 = 3.40 + 2. [1.31 + 4.16] = 310 𝑔/𝑚𝑜𝑙
𝐶 = [𝐶𝑎3 (𝑃𝑂4 )2 ]. 𝑀𝐶𝑎3 (𝑃𝑂4)2 = 5.10−5 . 310 = 1550.10−5 = 15,5.10−3
𝐶 = 15,5 𝑚𝑔/𝐿
d) Calculemos a massa molar do cloreto de potássio (KCl).
𝑀𝐾𝐶𝑙 = 1.39 + 1.35,5 = 74,5 𝑔/𝑚𝑜𝑙
𝐶 = [𝐾𝐶𝑙]. 𝑀𝐾𝐶𝑙 = 0,4.74,5 = 29,8 𝑔/𝐿

Gabarito: a) 2,1 g/L; b) 528 mg/L; c) 15,5 mg/L; d) 29,8 g/L

AULA 10 – SOLUÇÕES 21
Prof. Thiago Cardoso

(Estratégia Militares – TFC - Inédita)


Deseja-se preparar uma solução 0,05 mg/L de cloreto de sódio a partir da diluição de 10 mL de uma
solução 1 mg/L do sal em água pura. Assinale a alternativa que indica o volume de água que deve ser
adicionado para produzir a diluição desejada:
a) 20 mL
b) 90 mL
c) 140 mL
d) 190 mL
e) 200 mL

Comentários
Devemos nos lembrar que, para converter de concentração em massa para concentração molar,
devemos dividir pela massa molar.
[𝑁𝑎𝑂𝐻]0 𝑉0 = [𝑁𝑎𝑂𝐻]𝑓 𝑉𝑓
De acordo com os dados do enunciado, a concentração inicial da solução era 1 mg/L e o volume
inicial era 10 mL. Desejamos a concentração final de 0,05 mg/L
1.10 = 0,05. 𝑉𝑓
1.10 10
∴ 𝑉𝑓 = = = 200 𝑚𝐿
0,05 0,05
Como o volume final de solução obtido é igual a 200 mL, podemos calcular o volume de água
adicionado como a diferença entre o volume final e o volume inicial de solução.
VH2𝑂 = 𝑉𝑓 − 𝑉0 = 200 − 10 = 190 𝑚𝐿
Portanto, devemos adicionar 190 mL de água a 10 mL de solução 1 mg/L para obter um total de
200 mL da solução conhecida.
Gabarito: D

(Estratégia Militares – TFC - Inédita)


Considere que a solubilidade do cloreto de sódio em água é igual a 40 g/L e que uma amostra de água
do mar tem uma concentração do sal igual 10 g/L. Uma técnica muito utilizada para a obtenção do sal
é a evaporação da água do mar. Uma amostra de 20 L de água do mar é posta para evaporar, assinale a
alternativa que indica a quantidade de água que precisa ser evaporada até que o sal comece a cristalizar:
a) 2,5 L
b) 5 L
c) 10 L
d) 15 L
e) 17,5 L

AULA 10 – SOLUÇÕES 22
Prof. Thiago Cardoso

Comentários
Podemos usar a Lei da Diluição às avessas, tendo em vista que o solvente foi retirado, mas a massa
do soluto presente na solução permaneceu inalterada.
[𝑁𝑎𝐶𝑙]0 𝑉0 = [𝑁𝑎𝐶𝑙]𝑓 𝑉𝑓
O sal começa a se cristalizar quando a concentração da solução atinge 40 g/L, que é o coeficiente
de solubilidade. Notando que a concentração inicial de 10 g/L e o volume inicial da solução de 20 L foram
fornecidos no enunciado, podemos escrever:
10.20 = 40. 𝑉𝑓
10.20
∴ 𝑉𝑓 = = 5𝐿
40
Portanto, o volume final da solução passou a ser de 5 litros. O volume evaporado de água
corresponde à diferença entre o volume inicial e o final.
𝑉𝐻2𝑂 = 20 − 5 = 15 𝐿
Gabarito: D

3.Titulação
A titulação é um procedimento químico que visa a determinar a concentração de um determinado
reagente. As reações mais simples e, por isso mesmo, mais abordado em questões de prova é a
determinação da concentração de um ácido ou de uma base.
Existem, portanto, duas situações, com as quais devemos nos preocupar:
• Quando se deseja determinar a concentração de uma amostra de solução aquosa de um ácido por
meio de reação com uma solução aquosa básica de concentração conhecida;
• Quando se deseja determinar a concentração de uma mostra de solução aquosa de uma base com.
A técnica utilizada é análoga: deve-se reagi-la com uma solução aquosa ácida de concentração
conhecida.
O agente titulante é a solução aquosa de concentração conhecida que vai ser utilizada para titular
a outra espécie química, cuja concentração é desconhecida.
Em ambos os casos, existe um princípio geral de titulação. Vamos prestar bastante atenção a ele.

Na titulação de um ácido ou de uma base, o ponto de equivalência é a quantidade da base


titulante adicionada, para a qual tem-se:
á𝑐𝑖𝑑𝑜 𝑏𝑎𝑠𝑒
𝑛𝐻 + = 𝑛𝑂𝐻 −

No ponto de equivalência, todo o ácido ou a base original é neutralizada. É importante destacar


que a titulação não leva em consideração o grau de ionização, portanto, pouco importa se o ácido é fraco
ou forte.

AULA 10 – SOLUÇÕES 23
Prof. Thiago Cardoso

Deve-se simplesmente multiplicar o número mols de ácido presentes pelo número de hidrogênios
ionizáveis que ele possui na molécula e igualar ao número de mols da base pelo número de OH – que ela
possui no composto.
𝑛𝑎 𝑚𝑜𝑙é𝑐𝑢𝑙𝑎 𝑐𝑜𝑚𝑝𝑜𝑠𝑡𝑜
𝑛𝐻 + . 𝑉á𝑐𝑖𝑑𝑜 . [á𝑐𝑖𝑑𝑜] = 𝑛𝑂𝐻 − . 𝑉𝑏𝑎𝑠𝑒 . [𝑏𝑎𝑠𝑒]
Portanto, a quantidade de base necessária para titular 100 mL de uma solução 0,100 mol/L de um
ácido forte (como HC) é rigorosamente a mesma quantidade necessária para titular 100 mL de uma
solução 0,100 mol/L de um ácido fraco (como CH3COOH).
Um fato interessante é que nem sempre o pH no ponto de equivalência é neutro (pH = 7). A razão
para isso é que, embora todo o H+ do ácido tenha sido neutralizado pelo OH– da base, eles formam um
sal. E a solução salina nem sempre é neutra. De maneira geral, tem-se:
• Sal de ácido forte e base forte: pH neutro (pH = 7);
• Sal de ácido fraco e base forte: pH alcalino (pH > 7);
• Sal de ácido forte e base fraca: pH ácido (pH < 7);
• Sal de ácido fraco e base fraca: o pH será ácido se o ácido forte mais forte que a base, e será
alcalino, se a base for mais forte que o ácido;
Esse fato é importante, porque as titulações são acompanhadas em laboratório pelo gráfico de pH
com o auxílio de indicadores ácido-base. Vejamos como funciona esse procedimento na prática.

3.1. Equipamentos de Laboratório


O primeiro passo para a realização de uma titulação em laboratório é reunir a aparelhagem
necessária.
Primeiro, é necessário um meio reacional, que pode ser um balão de fundo chato ou redondo.
Esses equipamentos são muito úteis, porque permitem a agitação da solução, se necessário.
O segundo ponto que devemos saber é que a solução titulante deve ser
colocada de maneira gradual e muito lenta. Por isso, não se pode utilizar uma
solução exageradamente concentrada como titulante.
Além disso, deve-se utilizar um equipamento que permita o
gotejamento lento da solução titulante sobre a amostra a ser titulada. A
vidraria utilizada deve também ser graduada para saber o volume que foi gasto
na titulação.
Entre os equipamentos de laboratório que se prestam a esse papel,
podemos citar a pipeta e a bureta. A bureta é mais utilizada, pois ela vem com
uma torneira que permite o fluxo contínuo e mais regular da solução titulante.

Figura 6: Titulação com uma


Bureta e um Balão de Fundo
Chato

AULA 10 – SOLUÇÕES 24
Prof. Thiago Cardoso

3.2. Curva de PH
Um fato interessante das curvas de titulação é que elas inicialmente resistem à variação de pH,
que pouco varia quando é adicionada a solução titulante – a resistência é ainda maior quando o titulado
é um ácido ou base fraca.
Em um dado momento, o pH se rompe e varia bruscamente. Vejamos os vários casos possíveis.

3.2.1. Ácido Forte com Base Forte


Consideremos que inicialmente a solução a ser titulada tem volume igual a 1 litro e concentração
0,1 mol/L e que a solução titulante tem a mesma concentração. Traçaremos o gráfico do pH da solução
em função da quantidade de solução titulante adicionada.
Vale ressaltar que supusemos conhecida a concentração da solução do ácido apenas por fins
didáticos, para facilitar a sua compreensão. Nas questões de prova e na vida real, a solução titulada tem
concentração desconhecida.
Para o caso de um monoácido forte (como o HC) sendo titulado com uma monobase forte (como
NaOH), temos que a concentração inicial de íons H+ é igual 0,1 mol/L. Com isso, temos:
[𝐻 + ] = 0,1𝑚𝑜𝑙/𝐿 ∴ 𝑝𝐻 = − log([𝐻 + ]) = − log(0,1) = −(−1) = 1
Como as duas concentrações têm a mesma concentração o ponto de equivalência será quando o
volume de NaOH adicionado for igual ao volume do ácido, no caso, 1 L. No ponto de equivalência, o pH
será exatamente igual a 7.

Figura 7: Titulação de Ácido Forte com Base Forte

AULA 10 – SOLUÇÕES 25
Prof. Thiago Cardoso

Depois do ponto de equivalência, a solução passa a ser ligeiramente alcalina, pois todo o ácido já
foi neutralizado.
Quando o volume da base adicionado for de 2 litros, podemos dizer que 1 litro da base já foi
neutralizado pelo ácido, portanto, sobrou o número de mols de OH– que pode ser calculado como:
𝑛𝑂𝐻 − = 1. 𝑉𝑁𝑎𝑂𝐻 . [𝑂𝐻 − ] = 1.1.0,1 = 0,1 𝑚𝑜𝑙
Utilizamos o volume de 1 litro, porque dos 2 litros que foram adicionados, 1 litro foi usado para
neutralizar completamente o HC presente na amostra e sobrou apenas o volume de 1 litro sem ser
neutralizado.
No entanto, a concentração de OH– final não será de 0,1 mol/L, porque houve o efeito da diluição.
Vejamos:
𝑛𝑂𝐻 − 0,1 0,1 0,1
[𝑂𝐻 − ] = = = =
𝑉 𝑉𝐻𝐶𝑙 + 𝑉𝑁𝑎𝑂𝐻 1 + 2 3
Podemos calcular o pOH da solução final.
0,1
𝑝𝑂𝐻 = − log([𝑂𝐻 − ]) = − log ( ) = log(3) − log(0,1) = 0,48 + 1 = 1,48
3
Usando a relação que o pH + pOH = 14, temos que:
𝑝𝐻 + 𝑝𝑂𝐻 = 14 ∴ 𝑝𝐻 = 14 − 𝑝𝑂𝐻 = 14 − 1,48 = 12,52
Devido à diluição, o pH da solução final será ligeiramente menor que o pH da solução titulante.
Vale ressaltar que a curva de titulação para o caso de uma titulação de NaOH com uma solução de
HC seria semelhante. A única diferença é que o pH seria inicialmente alcalino e passaria a ser
progressivamente mais ácido.

Figura 8: Curva de Titulação de uma Solução Aquosa de Hidróxido de Sódio (NaOH) com Ácido Clorídrico (HC)

AULA 10 – SOLUÇÕES 26
Prof. Thiago Cardoso

Como o pH é uma escala logarítimica, o efeito da diluição se torna menos visível. Porém, podemos
enxergá-lo bem melhor, se analisarmos a condutividade da solução.
Inicialmente, como o ácido é forte, pode-se considerá-lo completamente dissociado. Sendo assim,
se a concentração do ácido é 0,1 mol/L, ele proverá 0,2 mol/L de partículas carregadas devido à sua
dissociação, que é considerada completa.

𝑯𝑪𝒍(𝒂𝒒) → 𝑯+ (𝒂𝒒) + 𝑪𝒍− (𝒂𝒒)

início 0,1 mol/L 0 0

Reage 0,1 mol/L 0,1 mol/L 0,1 mol/L

Final 0 0,1 mol/L 0,1 mol/L

No início, quando se adiciona a base (NaOH), ocorre uma precipitação de íons. Considere a Tabela
Estequiométrica da reação em que consideramos o número de mols de íons até o ponto de equivalência.

𝑯𝑪𝒍(𝒂𝒒) + 𝑵𝒂𝑶𝑯(𝒂𝒒) → 𝑵𝒂𝑪𝒍 (𝒂𝒒) + 𝑯𝟐 𝑶 (𝒍)

início 0,2 mol 0,2 mol 0 0

Reage 0,2 mol 0,2 mol 0,2 mol não dissocia

Final 0 mol 0 mol 0,2 mol não dissocia

Perceba que, no ponto de equivalência, a concentração de íons será:


𝑛í𝑜𝑛𝑠 0,2 0,2 0,2
[í𝑜𝑛𝑠] = = = = = 0,1 𝑚𝑜𝑙/𝐿
𝑉 𝑉𝐻𝐶𝑙 + 𝑉𝑁𝑎𝑂𝐻 1 + 1 2
Sendo assim, entre a situação inicial e o ponto de equivalência, ocorre uma redução na
concentração de íons. E, portanto, uma redução na condutividade da solução.
A partir do ponto de equivalência, a reação cessa. O que vai acontecer é que a solução neutralizada
(com 0,1 mol/L de íons) começa a receber a adição de uma solução mais concentrada em íons (0,2 mol/L).
Com isso, a condutividade vai aumentando, mas não chega ao valor original.
Podemos calcular o número de mols de íons presentes na solução usando a Tabela
Estequiométrica semelhante à anterior em termos de número de mols de íons presentes na mistura.

𝑯𝑪𝒍(𝒂𝒒) + 𝑵𝒂𝑶𝑯(𝒂𝒒) → 𝑵𝒂𝑪𝒍 (𝒂𝒒) + 𝑯𝟐 𝑶 (𝒍)

início 0,2 mol 0,4 mol 0 0

Reage 0,2 mol 0,2 mol 0,2 mol não dissocia

Final 0 mol 0,2 mol 0,2 mol não dissocia

Sendo assim, a concentração final de íons será:


𝑛í𝑜𝑛𝑠 0,4 0,4 0,4
[í𝑜𝑛𝑠] = = = = = 0,13 𝑚𝑜𝑙/𝐿
𝑉𝑓𝑖𝑛𝑎𝑙 𝑉𝐻𝐶𝑙 + 𝑉𝑁𝑎𝑂𝐻 1 + 2 3

AULA 10 – SOLUÇÕES 27
Prof. Thiago Cardoso

Figura 9: Variação da Condutividade durante a Titulação de uma Solução de Ácido Forte (HC) por uma Solução de
Base Forte (NaOH)

É importante citar que, nesse caso, o ponto de equivalência corresponderá à menor


condutividade.
O gráfico de condutividade no caso de uma titulação de uma solução de base forte por uma
solução de ácido é bastante semelhante ao ilustrado na Figura 9.

3.2.1. Titulação de uma Solução de Ácido Fraco por uma Solução de


Base Forte
Agora, examinaremos o que acontece quando titulamos uma solução de 1 litro de ácido acético
(CH3COOH) 0,1 mol/L por uma solução de hidróxido de sódio de mesma concentração.
O ácido acético é um ácido com baixo grau de ionização, de modo que o pH não é igual a 1, mas
bastante superior. No momento, não temos ferramentas para calcular esse pH. Somente no Capítulo
sobre Equilíbrio Iônico, aprenderemos como calcular. Por enquanto, esses valores devem ser fornecidos
pelo enunciado da questão.
O pH de uma solução de ácido acético 0,1 mol/L é igual a 4,8, o que é equivalente a uma
concentração de íons [𝐻 + ] ≅ 1,4.10−3 𝑚𝑜𝑙/𝐿.

AULA 10 – SOLUÇÕES 28
Prof. Thiago Cardoso

Figura 10: Curva de pH na titulação de Ácido Fraco (CH3COOH) por uma solução aquosa de Base Forte (NaOH)

No ponto de equivalência, a solução é alcalina, porque o sal formado é de ácido fraco e base forte.
O valor exato 9,2 somente pode ser calculado com as ferramentas que serão vistas em Equilíbrio Iônico.
Por hora, você precisa que esses valores lhe sejam fornecidos pela questão.
Durante a titulação, forma-se uma interessante região em torno de pH igual a 4,8 – em que se
verifica pH praticamente constante. Essa região é denominada tampão e será estudada também em
Equilíbrio Iônico. Trata-se de uma interessante região em que se obtém um sistema resistente a variações
de pH, muito importante para sistemas biológicos.

Uma solução 0,1 mol/L de ácido acético apresenta 0,1 mol/L de hidrogênios ionizáveis. Porém,
quando puro, a concentração de íons H+ é bem inferior [𝐻 + ] ≅ 1,4.10−3 𝑚𝑜𝑙/𝐿. Ou seja, nem todos os
hidrogênios ionizáveis do ácido realmente aparecem na forma de H+ em solução.
Porém, a quantidade de hidróxido de sódio necessária para titular uma solução de ácido forte
(HC) e uma solução de ácido fraco (CH3COOH) de mesma concentração é exatamente a mesma.
Isso acontece porque a reação de neutralização atinge todos os hidrogênios ionizáveis presentes
na solução de ácido fraco.

AULA 10 – SOLUÇÕES 29
Prof. Thiago Cardoso

De forma análoga ao que acontece na titulação de ácido forte, uma vez passado o ponto de
equivalência, o pH da solução se aproxima do pH da solução titulante, no caso, NaOH 0,1 mol/L, que é
igual a 13.
O efeito da diluição também acontece, de modo que não se atinge o mesmo pH = 13, mas sim
ligeiramente inferior, pH = 12,5 como já havia sido calculado.
Em relação à condutividade, devemos notar que uma solução de ácido acético é pouco condutora.
[í𝑜𝑛𝑠]0 = 2.1,4.10−3 = 2,8.10−3 𝑚𝑜𝑙/𝐿
Quando a solução reage, ela forma o acetato de sódio, que é um sal, portanto, se encontra
completamente dissociado. Logo, a solução original que tem baixa condutividade dá lugar a uma solução
com maior concentração de íons.
Para o ponto de equivalência, podemos escrever.

𝑪𝑯𝟑 𝑪𝑶𝑶𝑯(𝒂𝒒) + 𝑵𝒂𝑶𝑯 (𝒂𝒒) → 𝑪𝑯𝟑 𝑪𝑶𝑶− 𝑵𝒂+ + 𝑯𝟐 𝑶 (𝒍)

início Baixa 0,2 mol 0 0

Reage Baixa 0,2 mol 0,2 mol não dissocia

Final 0 mol 0,2 mol 0,2 mol não dissocia

Sendo assim, a concentração de íons no ponto de equivalência é bem maior que a concentração
inicial.
0,2 0,2 0,2 0,2
[í𝑜𝑛𝑠]𝑒𝑞 = = = = = 0,1 𝑚𝑜𝑙/𝐿
𝑉 𝑉𝐻𝐶𝑙 + 𝑉𝑁𝑎𝑂𝐻 1 + 1 2
Depois do ponto de equivalência, a solução passa a se tornar cada vez mais concentrada em íons,
pois está sendo adicionada uma solução de hidróxido de sódio que tem uma concentração 0,2 mol/L em
íons.
A concentração final é igual à que foi calculada no caso da titulação de ácido forte e base forte.
[í𝑜𝑛𝑠]𝑓 = 0,13 𝑚𝑜𝑙/𝐿

Figura 11: Comportamento da Condutividade em uma Solução de Ácido Fraco (CH3COOH) por uma Base Forte
(NaOH)

AULA 10 – SOLUÇÕES 30
Prof. Thiago Cardoso

3.3. Indicadores de PH
Os indicadores de pH (ou ácido-base) são substâncias que variam a sua coloração de acordo com
o pH do meio.
Um bom indicador ácido base tem uma faixa de viragem específica, que é uma faixa estreita de
pH em que ele troca rapidamente de cor.
Tabela 1: Faixa de Viragem de Diversos Indicadores de pH

Indicador Cor a pH baixo Faixa de Viragem Cor a pH alto

Azul de Timol (primeira transição) Vermelho 1,2 a 2,8 Amarelo

Azul de Bromofenol Amarelo 3,0 a 4,6 Violeta

Vermelho de Metila Vermelho 4,4 a 6,2 Amarelo

Vermelho de Fenol Amarelo 6,6 a 8,0 Vermelho

Fenolftaleína Incolor 8,2 a 10,0 Rosa Carmim

Vamos entender a Tabela 1. O azul timol tem faixa de viragem entre pH 1,2 a 2,8. Isso significa que
para pH abaixo de 1,2, ele é vermelho, enquanto que, para pH acima de 2,8, ele é amarelo.
Tabela 2: Indicadores de pH e suas respectivas cores em diferentes valores de pH

Com uma boa gama de indicadores com faixas de viragens diferentes, é possível construir um
medidor de pH. Essa é a ideia da fita medidora de pH.
Por exemplo, se construirmos uma fita com esses cinco indicadores, podemos medir o pH de uma
solução qualquer de forma aproximada. Basta comparar o conjunto de cores obtido com o conjunto de
cores presentes na escala.
Suponha que tenhamos aferido o pH de uma solução desconhecida com a fita formada pelos cinco
indicadores e que tenhamos obtido o seguinte conjunto de cores.

AULA 10 – SOLUÇÕES 31
Prof. Thiago Cardoso

Figura 12: Resultado de uma Aferição de pH com uma fita medidora

Qual o pH dessa solução?


Olhe para a Tabela 2 e compare as cores obtidas,
Como o azul de timol está amarelo, o pH deve ser na faixa de 3 ou superior. Olhamos o azul de
bromofenol roxo e concluímos um pH acima de 4,5. O vermelho de metila está vermelho, então, o pH não
pode chegar a 6, sendo provavelmente abaixo de 5,5.
Com isso, concluímos que a solução tem pH em torno de 5 – entre 4,5 e 5,5.
A fita medidora permite uma análise rápida e barata do pH. Você pode comprar uma facilmente
para usar em casa para fins recreativos em sites, como o Mercado Livre ou em lojas de produtos químicos.
A desvantagem dela é que não permite medidas muito precisas. Como vimos, fomos capazes de
concluir que o pH da solução está em torno de 4,5 a 5,5. Algumas situações práticas requerem medidas
mais precisas.
Os indicadores de pH também são bastante úteis na titulação.
Em uma titulação, o indicador de pH será útil quando for capaz de apontar com cores diferentes o
momento em que ocorreu o ponto de equivalência.
Por exemplo, a fenolftaleína é muito utilizada na titulação de ácidos fortes por bases fortes, e vice-
versa.

AULA 10 – SOLUÇÕES 32
Prof. Thiago Cardoso

Figura 13: Cores da Fenolftaleína na Titulação de uma Solução de Base Forte (NaOH) por um Ácido Forte (HC)

A Tabela 2 mostra que a viragem da fenolftaleína acontece antes do ponto de equivalência. A


viragem da fenolftaleína acontece na faixa de pH de 8,2 a 10, enquanto que o ponto de equivalência se
situa no pH igual a 7,0.
Porém, como vemos na Figura 13, a diferença entre o volume de ácido gasto para atingir o pH igual
a 8,2 e para atingir o pH igual a 7,0 é muito pequena. Portanto, em geral, podemos assumir que, assim
que o indicador muda de cor, atingiu-se o ponto de equivalência.
Por conta disso, vários indicadores poderiam ser utilizados nessa titulação. Por exemplo, se fosse
utilizado o vermelho de metila – faixa de viragem de 4,4 a 6,2 – teríamos. Nesse caso, a viragem ocorre
um pouco depois do ponto de equivalência. Porém, mais uma vez, a diferença de volume de ácido
empregada é praticamente desprezível.

AULA 10 – SOLUÇÕES 33
Prof. Thiago Cardoso

Figura 14: Cores do Vermelho de Metila na Titulação de uma Solução de Base Forte (NaOH) por um Ácido Forte
(HC)

O azul de bromofenol, por sua vez, não é o mais adequado para a titulação de uma solução de
ácido fraco com pH inicial igual a 5,0. Como esse pH já está acima da sua faixa de viragem, ele permanecerá
violeta durante toda a titulação.

Figura 15: Exemplo de Titulação Mal Sucedida porque o Indicador não possui a faixa de viragem adequada

AULA 10 – SOLUÇÕES 34
Prof. Thiago Cardoso

A escolha adequada do indicador de pH em uma titulação é muito importante. Deve-se utilizar,


portanto, um indicador que tenha a sua faixa de viragem próxima do ponto de equivalência da titulação.

4. Propriedades Coligativas
Nesse capítulo, estudaremos os efeitos da adição de um soluto não-volátil sobre as propriedades
físicas do solvente.
É importante você conhecer esse conceito, porque, à exceção do Efeito Tonoscópico, todas as
expressões que mostraremos nesse capítulo são válidas somente para solutos não-voláteis.
Um soluto não-volátil idealmente é aquele que apresenta pressão de vapor nula. Porém, esse
modelo ideal de soluto não existe, tendo em vista que qualquer soluto apresenta pressão de vapor, ainda
que pequena.
De maneira geral, considera-se um soluto não-volátil aquele que apresenta pressão de vapor
menor que a do solvente.
Como a redução da pressão de vapor decreta o aumento da temperatura de ebulição, uma regra
que podemos utilizar é que os solutos não-voláteis são os que apresentam temperatura de ebulição
maior que a do solvente.
São exemplos de solutos não-voláteis comuns:
• O açúcar;
• Os compostos iônicos.
Por outro lado, alguns solutos voláteis bastante comuns são:
• Álcool etílico;
• Acetona
Tome muito cuidado com as questões que envolvem solutos voláteis, pois eles não exercem os
efeitos de modificar as temperaturas de fusão e ebulição do solvente.

4.1. Efeito Tonoscópico


A pressão parcial de vapor de cada componente da solução depende de dois fatores:
• A pressão de vapor do componente puro;
• A fração molar dos mesmos componentes na mistura.
Em uma solução ideal, a pressão parcial de cada componente da solução é igual ao produto de sua
pressão quando puro pela sua fração molar.

𝑃𝑖 = (𝑃𝑖 )𝑝𝑢𝑟𝑜 . 𝑥𝑖

Essa expressão é conhecida como Lei de Raoult (lê-se “Raul”) e faz bastante sentido. Pensemos
em um exemplo concreto para facilitar o seu entendimento dela. A pressão de vapor da água pura a 25
°C é igual a 18 mmHg e a pressão de vapor é igual a 60 mmHg.
Suponhamos que tenhamos uma solução 50% água e 50% álcool. Nesse caso, teríamos que as
pressões parciais de vapor de cada componente seriam:
𝑃𝐻2 𝑂 = 0,50.18 = 9 𝑚𝑚𝐻𝑔

AULA 10 – SOLUÇÕES 35
Prof. Thiago Cardoso

𝑃𝐶2𝐻5 𝑂𝐻 = 0,50.60 = 30 𝑚𝑚𝐻𝑔


Como as moléculas de água só ocupam metade da mistura, a sua pressão parcial de vapor se reduz
à metade do que seria, caso ela estivesse pura. O mesmo efeito acontece com o etanol.
Podemos aplicar a mesma equação e calcular as pressões de vapor da água e do etanol para vários
teores molares (ou frações molares).
Tabela 3: Comportamento das Pressões de Vapor da Mistura e de cada componente em Função do Teor de Etanol

Teor de Teor de Pressão de Vapor da Pressão de Vapor da Água Pressão de Vapor da


Etanol Água Etanol (mmHg) (mmHg) Mistura

0% 100,00% 0 18 18

10% 90,00% 6 16,2 22,2

20% 80,00% 12 14,4 26,4

30% 70,00% 18 12,6 30,6

40% 60,00% 24 10,8 34,8

50% 50,00% 30 9 39

60% 40,00% 36 7,2 43,2

70% 30,00% 42 5,4 47,4

80% 20,00% 48 3,6 51,6

90% 10,00% 54 1,8 55,8

100% 0,00% 60 0 60

AULA 10 – SOLUÇÕES 36
Prof. Thiago Cardoso

Vale lembrar que a Tabela 3 foi construída com base nas frações molares de etanol e de água.
Cuidado para não confundir com fração de volume ou de massa, que são mais utilizadas.

Figura 16: Pressão de Vapor da Mistura e de cada componente em Função do Teor de Etanol

É interessante observar que a pressão de vapor da mistura água-etanol cresce com o etanol de
etanol. Isso acontece, porque o etanol é mais volátil que a água.
Porém, no caso de uma solução aquosa de um soluto não-volátil, o incremento do teor molar do
soluto provoca o abaixamento na pressão de vapor do solvente.
Podemos expressar esse abaixamento de duas formas: a pressão de vapor real do solvente será
proporcional à sua própria fração molar; ou o abaixamento da pressão de vapor do solvente é
proporcional à fração molar do soluto.

𝑃𝑠𝑜𝑙𝑣𝑒𝑛𝑡𝑒 = 𝑃0 . 𝑥2
Δ𝑃 = 𝑃0 . 𝑥1

Vamos estudar a pressão de vapor de uma mistura de água e açúcar, que pode ser considerado
um solvente não-volátil, cuja pressão de vapor é aproximadamente nula.
Tabela 4: Abaixamento na Pressão de Vapor da Água pela Adição de Açúcar

Teor de Açúcar Teor de Água Pressão de Vapor da Água (mmHg)

0% 100,00% 18

10% 90,00% 16,2

20% 80,00% 14,4

30% 70,00% 12,6

40% 60,00% 10,8

AULA 10 – SOLUÇÕES 37
Prof. Thiago Cardoso

50% 50,00% 9

60% 40,00% 7,2

70% 30,00% 5,4

80% 20,00% 3,6

90% 10,00% 1,8

100% 0,00% 0

É interessante observar que a pressão parcial de vapor de água não depende do tipo de soluto que
nela está dissolvido, mas apenas de sua fração molar. Observe que o mesmo abaixamento registrado com
a dissolução de etanol (soluto volátil) foi registrado com a dissolução de açúcar (soluto não-volátil).
O único caso em que precisa ter cuidado é quando a dissolução envolve alterações na estrutura
do soluto, inclusive com alteração no número de partículas.
O caso mais conhecido é quando o soluto sofre dissociação iônica ou ionização em meio aquoso.
Por exemplo, a dissolução de cloreto de sódio em água libera os íons sódio (Na+) e cloreto (C –) em
solução.
𝐻2 𝑂
𝑁𝑎𝐶𝑙 (𝑠) → 𝑁𝑎 + (𝑎𝑞) + 𝐶𝑙 − (𝑎𝑞)
Sendo assim, cada fórmula de NaC, quando dissolvida em água, libera duas partículas.
Vamos exemplificar calculando a pressão de vapor da água do meu aquário, que, para fins de
cálculo, pode ser considerada como uma solução 40 g/L de cloreto de sódio em água.
Considerando essa solução a mais ideal possível, em 1 litro dessa solução, encontram-se 1000 g
de água. Precisamos calcular o número de mols do solvente, o que pode ser feito dividindo-se a massa
pela sua massa molar. Para isso, precisamos lembrar as massas dos elementos que compõem a água (H =
1 g/mol; O = 16 g/mol).
𝑚𝐻2 𝑂 1000 1000
𝑛𝐻2 𝑂 = = = = 55,6 𝑚𝑜𝑙
𝑀𝐻2 𝑂 2.1 + 1.16 18
Em 1 litro dessa solução, encontramos 40 g de cloreto de sódio. O número de mols correspondente
pode ser calculado dividindo-se a massa de cloreto de sódio pela sua massa molar. Para isso, precisamos
lembrar as massas dos elementos (Na = 23 g/mol; C = 35,5 g/mol).
40 40
𝑛𝑁𝑎𝐶𝑙 = = ≅ 0,68 𝑚𝑜𝑙
23 + 35,5 58,5
Porém, quando dissolvido em água, cada mol do soluto libera 2 mols de íons.
𝑛1 = 2.0,68 = 1,36 𝑚𝑜𝑙
Dessa maneira, o total de mols em solução é igual à soma do número de mols de água com o
número de mols de íons liberados pelo cloreto de sódio.
𝑛 = 𝑛1 + 𝑛2 = 1,36 + 55,6 ≅ 57 𝑚𝑜𝑙
Portanto, o abaixamento da pressão de vapor provocada pela adição de soluto é:
1,36
Δ𝑃 = 𝑃0 . 𝑥1 = 18. = 0,43 𝑚𝑚𝐻𝑔
57

AULA 10 – SOLUÇÕES 38
Prof. Thiago Cardoso

Mesmo em uma solução bastante concentrada, que é a água salgada, o abaixamento da pressão
de vapor, consideração a solução ideal, seria praticamente desprezível. A pressão de vapor da água
destilada a 25 °C é de 18 mmHg, enquanto que a pressão de vapor da água de aquário marinho calculada
teoricamente é 17,57 mmHg.
A despeito disso, a pressão de vapor da água do meu aquário a 25 °C é ligeiramente inferior (entre
16,5 e 17 mmHg) ao que foi calculado teoricamente.
A explicação para essa ligeira diferença é que a mistura de água e sal tem interações
intermoleculares mais intensas do que as interações da própria água pura.
Com isso, os íons de sal dissolvidos na água atraem para si as moléculas de água com mais
intensidade do que o que acontece na água pura. Logo, as moléculas de água têm maior dificuldade de
passar para o estado de vapor, portanto, a pressão de vapor da água salgada é inferior ao que é previsto
teoricamente.
Por fim, convém ressaltar que, em algumas situações, é possível ter um número de mols
dissolvidos inferior ao número de mols original do soluto. Essa situação é mais comum em soluções
apolares.
Por exemplo, quando dissolvemos ácido benzóico em benzeno, as moléculas do ácido sofrem
dimerização.

Figura 17: Exemplo de Dissolução com Dimerização

Considerando a dimerização completa, se fizermos uma solução de 1 mol/L de ácido benzoico em


benzeno, na verdade, teremos apenas 0,5 mol/L de substâncias dissolvidas.

4.2. Efeitos Ebulioscópico e Crioscópico


Em uma solução de um soluto não-volátil, é importante registrar que somente o solvente sofre
mudanças de estado físico.
Pense, por exemplo, em mistura de água e açúcar.
Quando aquecida, somente a água entra em ebulição, mas o açúcar que está dissolvido permanece
dissolvido. O que evapora, portanto, é vapor de água pura. Em uma situação limite, toda a água evaporará,
deixando apenas o açúcar sólido no fundo do recipiente.
Analogamente, quando a mistura é resfriada, somente a água passa para o estado de gelo. O
açúcar permanecerá dissolvido na fase líquida.
Vamos apresentar um princípio que devemos ter em mente para o estudo das próximas
propriedades coligativas, que são: o Efeito Ebulioscópico, o Efeito Crioscópico e também a Pressão
Osmótica.

AULA 10 – SOLUÇÕES 39
Prof. Thiago Cardoso

As interações intermoleculares do soluto não-volátil com o solvente são fortes, de modo que
o soluto tentará permanecer dissolvido. Logo, ele vai dificultar tanto a ebulição como o
congelamento do solvente.

Portanto, a adição de um soluto não-volátil – jamais se esqueça de que as Propriedades Coligativas


se referem a esse tipo de soluto – provoca aumento na temperatura de congelamento e redução na
temperatura de congelamento.

Figura 18: Variação da Temperatura de Ebulição e Congelamento da Água após receber solutos não-voláteis

As variações nas temperaturas de mudança de estado são expressas em função da molalidade do


soluto e do fator de van’t Hoff. Dependem também das constantes ebulioscópica e crioscópica, que são
características do solvente.

Δ𝑇𝐸 = 𝐾𝑒 𝑊𝑖
𝛥𝑇𝐶 = 𝐾𝑐 𝑊𝑖

No caso da água, as constantes ebulioscópica e crioscópica valem, respectivamente:


𝐾𝑒 = 0,512 𝐾. 𝑘𝑔/𝑚𝑜𝑙
𝐾𝑐 = 1,853 𝐾. 𝑘𝑔/𝑚𝑜𝑙
A molalidade, no caso de soluções aquosas diluídas, é considerada como sendo aproximadamente
igual à molaridade.
O fator de van’t Hoff (i) diz respeito à razão do número de mols de partículas em solução pelo
número de mols da substância que foi originalmente dissolvida.
Esse fator busca levar em consideração efeitos, como a dissociação iônica, ionização ou
dimerização.
Caso a substância não sofra nenhum processo de quebra ou junção de moléculas, o seu fator de
van’t Hoff será igual a 1. É o caso da maioria dos solventes moleculares, exceto os ácidos, como o açúcar.
Quando o soluto sofre ionização ou dissociação iônica, devemos levar em conta esse fato. Por
exemplo, considere a dissolução de ácido clorídrico (HC) em água. Considerando que esse ácido é forte
(100% de ionização), temos que:
𝐻2 𝑂
𝐻𝐶𝑙 → 𝐻 + (𝑎𝑞) + 𝐶𝑙 − (𝑎𝑞)

AULA 10 – SOLUÇÕES 40
Prof. Thiago Cardoso

Ao se dissolver 1 mol de HC, temos o seguinte.

𝑯𝑪𝒍 𝑯𝟐 𝑶
→ 𝑯+ (𝒂𝒒) + 𝑪𝒍− (𝒂𝒒) Total de Espécies Químicas

Início 1 mol 0 0 1 mol

Reage 1 mol 1 mol 1 mol

Final 0 mol 1 mol 1 mol 2 mol

Dessa maneira, o fator de van’t Hoff é igual à razão entre o número de mols ao final da reação de
dissolução e o número de mols inicialmente dissolvidos.
2
=2𝑖=
1
Portanto, os efeitos coligativos do HC devem ser multiplicados por dois.
Se, por acaso, for fornecido algum grau de dissociação, você precisará utilizar essa informação. Por
exemplo, suponha que o ácido acético (CH3COOH) possui grau de dissociação igual a 30%.

𝑪𝑯𝟑 𝑪𝑶𝑶𝑯 𝑯

𝟐𝑶 𝑯+ (𝒂𝒒) + 𝑪𝑯𝟑 𝑪𝑶𝑶− (𝒂𝒒) Total de Espécies Químicas

Início 1 mol 0 0 1 mol

Reage 0,3 mol 0,3 mol 0,3 mol

Final 0,7 mol 0,3 mol 0,3 mol 1,3 mol

Para o ácido acético, o fator de van’t Hoff é, portanto, igual à razão entre o número de mols de
partículas em solução ao final (1,3 mol) pelo número de mols inicialmente dissolvido (1 mol).
1,3
𝑖=
= 1,3
1
Podemos também estabelecer uma regra geral para ajudar a calcular o fator de van’t Hoff para
solutos que se dividem exclusivamente em duas com grau de ionização 𝛼.

𝑖 =1+𝛼

Outro caso que podemos comentar é quando ocorre uma reunião de partículas. Considere que a
dimerização do ácido benzoico em benzeno tenha um grau de dimerização de 60%.

𝑪𝟔 𝑯𝟓 𝑪𝑶𝑶𝑯 𝑯

𝟐𝑶 (𝑪𝟔 𝑯𝟓 𝑪𝑶𝑶𝑯)𝟐 Total de Espécies Químicas

Início 1 mol 0 1 mol

Reage 0,6 mol 0,3 mol

Final 0,4 mol 0,3 mol 0,7 mol

Nesse caso, o fator de van’t Hoff calculado será menor que 1, refletindo que houve redução no
número de partículas dissolvidas.

AULA 10 – SOLUÇÕES 41
Prof. Thiago Cardoso

0,7
𝑖= = 0,7
1

4.3. Pressão Osmótica


A osmose é um fenômeno natural que acontece quando se tem uma conexão semipermeável
entre dois meios.
Uma conexão semipermeável é aquela que só permite a passagem do solvente, não permitindo
que o soluto se disperse.
O caso mais conhecido são as membranas semipermeáveis. As membranas celulares e a
membrana do ovo (que fica abaixo da casca) são bons exemplos. Elas permitem somente a passagem de
água, mas não dos solutos nela dissolvidos.
No processo de osmose, dois meios são importantes:
• Hipertônico: é o meio em que o soluto está em maior concentração;
• Hipotônico: é o meio em que o soluto está em menor concentração.
No processo de osmose, o solvente passa do meio mais hipotônico para o meio mais hipertônico.
A tendência natural da osmose é igualar as concentrações entre os dois meios.

Figura 19: Processo de Osmose

Contudo a osmose pode ser evitada se for aplicada uma pressão sobre o lado mais hipertônico.

Figura 20: Pressão Osmótica

Com a aplicação de pressão, as moléculas de solvente no meio hipertônico se deslocam para as


fronteiras da membra semipermeável, dificultando que as moléculas do outro lado a atravessem.

AULA 10 – SOLUÇÕES 42
Prof. Thiago Cardoso

A pressão osmótica é definida como a menor pressão necessária para cessar a osmose. Ela é
calculada em função da diferença entre as concentrações dos solutos dos lados da membrana.

𝜋 = 𝛥[𝑆𝑜𝑙𝑢𝑡𝑜𝑠]𝑅𝑇

A pressão osmótica é a menor pressão necessária para impedir a osmose. Caso a pressão aplicada
seja superior, ocorrerá o fenômeno da osmose reversa.
A osmose reversa consiste na passagem do solvente do meio hipertônico para o meio hipotônico,
contrariando a sua tendência natural. Ela é um dos meios mais utilizados para a produção de água
destilada com altíssimo grau de pureza.

Figura 21: Produção de Água Destilada a partir de Osmose Reversa

A osmose reversa é bastante utilizada para a produção de água destilada (nesse caso, denominada
água de RO, que significa reverse osmose) em escala de laboratório. Para isso, prefere-se utilizar a água já
com baixa concentração de sais para evitar que a membrana sature com facilidade.
Embora a osmose mais conhecida ocorra através da membrana semipermeável, esse fenômeno
também pode acontecer por outros modos que permitam exclusivamente a passagem do solvente. Por
exemplo, por meio aéreo, quando se tem dois potes abertos.

AULA 10 – SOLUÇÕES 43
Prof. Thiago Cardoso

Figura 22: Osmose através da Evaporação Seletiva do Solvente

A Figura 22 mostra um processo semelhante à osmose, que ocorre entre duas soluções de
concentrações de soluto diferentes.
Como a presença de solutos não-voláteis provoca o abaixamento da pressão de vapor do solvente,
a solução mais hipertônica terá menor pressão de vapor que a solução hipotônica.
Com isso, mais moléculas passarão ao estado de vapor em volta do frasco da solução hipotônica.
Por outro lado, o condensamento será mais intenso na solução hipertônica.
Em termos práticos, o que vai acontecer é que moléculas de solvente vão evaporar migrando da
solução hipotônica para a hipertônica. Esse processo somente cessará quando as concentrações das duas
soluções forem iguais.

Os termos R e T na expressão da Pressão Osmótica serão mais bem estudados no Capítulo sobre
Gases. Porém, como eles já apareceram aqui nesse capítulo, você precisa entender um pouco sobre eles.
O coeficiente R é a Constante Universal dos Gases, cujo valor é:

𝑅 = 0,082 𝑎𝑡𝑚. 𝐿/(𝑚𝑜𝑙. 𝐾) = 8,31 𝐽/(𝑚𝑜𝑙. 𝐾)

É preciso ter bastante atenção para utilizar R nas unidades coerentes com o que se deseja calcular
a pressão osmótica. Se a pressão osmótica for pedida em atm, que é a unidade mais comum na Química,
utiliza-se o valor 0,082. Se ela for pedida nas unidades do SI (N/m²), deve-se utilizar o valor de 8,31.
Já T se refere à temperatura absoluta. Essa temperatura é usualmente medida na Escala Kelvin,
que pode ser obtida somando 273 à temperatura em graus Celsius.

AULA 10 – SOLUÇÕES 44
Prof. Thiago Cardoso

𝑇 (𝐾) = 𝑇 (°𝐶) + 273

(Estratégia Militares – TFC - Inédita)


Determine:
a) A temperatura de ebulição de uma solução aquosa 0,2 mol/kg de cloreto de cálcio.
b) A pressão osmótica de uma solução 0,03 mol/L de uma solução aquosa de sacarose a 27°C.
c) A temperatura de ebulição de uma solução 2 mol/kg de uma solução aquosa de cloreto de sódio.
Dados de Massas Atômicas: KE = 1,86 °C.molal –1; R = 0,082 atm.L/(mol.K) = 8,31 J/(mol.K)

Comentários
a) Vamos aplicar a expressão para a temperatura de ebulição.
Δ𝑇𝐸 = 𝐾𝐸 . 𝑊. 𝑖
O cloreto de cálcio (CaCl2) se decompõe, liberando 3 íons.
𝐶𝑎𝐶𝑙2 (𝑎𝑞) → 𝐶𝑎2+ (𝑎𝑞) + 2 𝐶𝑙 − (𝑎𝑞)
Dessa forma, o fator de van’t Hoff para o sal é i = 3.
Δ𝑇𝐸 = 1,86.0,2.3 = 1,116 °𝐶
O fator calculado corresponde ao aumento da temperatura de ebulição em relação ao solvente
puro, que é igual a 100°C.
𝑇𝐸 = 100 + 1,116 = 101,116°𝐶
b) A pressão osmótica é calculada pela seguinte expressão.
𝜋 = [𝑠𝑜𝑙𝑢𝑡𝑜]. 𝑅𝑇
O soluto é a sacarose, que não se dissocia, quando dissolvido em água. Portanto, o seu fator de
van’t Hoff é i = 1.
Além disso, note que a temperatura deve ser calculada em Kelvin. Para isso, precisamos somar
273. Logo, a temperatura em Kelvin é T = 27 + 273 = 300 K.
𝜋 = 0,03.0,082.300 ≅ 0,738 𝑎𝑡𝑚
c) Vamos aplicar a expressão para a temperatura de ebulição.
Δ𝑇𝐸 = 𝐾𝐸 . 𝑊. 𝑖
O cloreto de potássio (KCl) se decompõe, liberando 2 íons.
𝐾𝐶𝑙 (𝑎𝑞) → 𝐾 + (𝑎𝑞) + 𝐶𝑙 − (𝑎𝑞)
Dessa forma, o fator de van’t Hoff para o sal é i = 2.
Δ𝑇𝐸 = 1,86.2.2 = 7,44 °𝐶

AULA 10 – SOLUÇÕES 45
Prof. Thiago Cardoso

O fator calculado corresponde ao aumento da temperatura de ebulição em relação ao solvente


puro, que é igual a 100°C.
𝑇𝐸 = 100 + 7,44 = 107,44°𝐶
Gabarito: a) 101,116 °C; b) 0,738 atm; c) 15,5 mg/L; d) 107,44 °C

(Estratégia Militares – TFC - Inédita)


A pressão osmótica de uma solução:
a) depende do tipo de soluto.
b) depende exclusivamente da concentração molar do soluto e de sua dissociação.
c) é invariante com a temperatura.
d) é a pressão necessária para promover a osmose.
e) 0,1 mol/L de glicose é igual à pressão osmótica de uma solução 0,1 mol/L de cloreto de sódio.

Comentários
A pressão osmótica de uma solução é calculada pela expressão:
𝜋 = [𝑠𝑜𝑙𝑢𝑡𝑜𝑠]. 𝑅𝑇
Portanto, ela é obtida como o produto da concentração molar do total de soluções pela constante
universal dos gases e a temperatura absoluta. Com base nisso, vamos analisar as afirmações propostas no
enunciado.
a) Não depende. Depende apenas das concentrações molares totais de solutos presentes na
solução. Afirmação incorreta.
b) É isso mesmo. Depende apenas da concentração molar dos solutos, não sendo influenciada pelo
tipo de soluto, exceto pelo fato de ele estar ionizado ou dissolvido. Afirmação correta.
c) Como vimos, ela é proporcional à temperatura absoluta do sistema. Afirmação incorreta.
d) Na verdade, é a pressão necessária para impedir a osmose. Afirmação incorreta.
e) Como o cloreto de sódio se dissocia, essa solução terá 0,2 mol/L de íons no total, sendo 0,1
mol/L de íons sódio e 0,1 mol/L de cloreto.
Gabarito: B

4.4. Determinação de Massas Molares


Na Estequiometria, vimos que a Análise de Combustão é uma das técnicas mais utilizadas para a
determinação da Fórmula Mínima de diversos compostos, em especial, compostos orgânicos.
Porém, principalmente no caso de compostos orgânicos, a fórmula mínima não é suficiente para
identificar o composto. Por exemplo, o ácido lático (C3H6O3) e a glicose (C6H12O6) são compostos
diferentes, com fórmulas moleculares diferentes, porém, apresentam a mesma fórmula mínima (CH 2O).
Ao queimar uma determina amostra de material, não seremos capazes de saber se é glicose ou se
é ácido lático somente pela análise dos produtos da combustão.

AULA 10 – SOLUÇÕES 46
Prof. Thiago Cardoso

Porém, podemos melhorar o nosso conhecimento sobre a substância analisada avaliando o


comportamento de suas propriedades coligativas.
Isso se deve ao fato de que as propriedades coligativas dependem do número de mols do soluto
presentes na amostra.
Por exemplo, considere duas soluções contendo 36 g de soluto em 1 kg de água: na solução A, o
soluto é o ácido lático (C3H6O3); na solução B, o soluto é a glicose (C6H12O6). Vamos calcular o aumento da
temperatura de ebulição da solução nos dois casos, desconsiderando a ionização do ácido lático.
Para isso, precisaremos calcular as massas molares da glicose e do ácido lático.
𝑀𝐶3 𝐻6 𝑂3 = 3.12 + 6.1 + 3.16 = 36 + 6 + 48 = 90 𝑔/𝑚𝑜𝑙
𝑀𝐶6𝐻12 𝑂6 = 6.12 + 12.1 + 6.16 = 72 + 12 + 96 = 180 𝑔/𝑚𝑜𝑙
O número de mols presentes em 36 g de cada um dos possíveis solutos é:
𝑚𝐶3𝐻6 𝑂3 36
𝑛𝐶3𝐻6 𝑂3 = = = 0,4 𝑚𝑜𝑙
𝑀𝐶3𝐻6 𝑂3 90
𝑚𝐶6𝐻12 𝑂6 36
𝑛𝐶6 𝐻12 𝑂6 = = = 0,2 𝑚𝑜𝑙
𝑀𝐶3𝐻6 𝑂3 180
Dessa maneira, podemos calcular as molalidades dos solutos nas duas soluções.
𝑛𝐶3 𝐻6 𝑂3 0,4
𝑊𝐴 = = = 0,4 𝑚𝑜𝑙𝑎𝑙
𝑚𝐻2𝑂 1
𝑛𝐶6𝐻12 𝑂6 0,2
𝑊𝐵 = = = 0,2 𝑚𝑜𝑙𝑎𝑙
𝑚𝐻2 𝑂 1
Como as molalidades são diferentes, o efeito ebulioscópico é diferente.
Δ𝑇𝐴 = 𝐾𝐶 . 𝑊. 𝑖 = 1,86.0,4.1 = 0,744 °𝐶
Δ𝑇𝐵 = 𝐾𝐶 . 𝑊. 𝑖 = 1,86.0,2.1 = 0,372 °𝐶
Sendo assim, é possível com base na massa fornecida do soluto e do solvente e nos efeitos
coligativos observados, calcular a massa molar desconhecida do soluto.
Temos, portanto, um caminho a seguir na Química Analítica para determinar uma substância
desconhecida. Até o momento, aprendemos as duas primeiras etapas. A terceira somente será vista nos
nossos estudos de Química Orgânica.

Determinação da Determinação da Determinação da


Fórmula Mínima Massa Molar pelas Fórmula Estrutural
pela Análise de Propriedades pelas Propriedades
Combustão Coligativas Reacionais

Figura 23: Procedimento para a Determinação Experimental de uma Substância Desconhecida

AULA 10 – SOLUÇÕES 47
Prof. Thiago Cardoso

(Estratégia Militares – TFC - Inédita)


Determine:
a) A temperatura de ebulição de uma solução aquosa 0,2 mol/kg de cloreto de cálcio.
b) A pressão osmótica de uma solução 0,03 mol/L de uma solução aquosa de sacarose a 27°C.
c) A temperatura de ebulição de uma solução 2 mol/kg de uma solução aquosa de cloreto de sódio.
Dados de Massas Atômicas: KE = 1,86 °C.molal –1; R = 0,082 atm.L/(mol.K) = 8,31 J/(mol.K)

Comentários
a) Vamos aplicar a expressão para a temperatura de ebulição.
Δ𝑇𝐸 = 𝐾𝐸 . 𝑊. 𝑖
O cloreto de cálcio (CaCl2) se decompõe, liberando 3 íons.
𝐶𝑎𝐶𝑙2 (𝑎𝑞) → 𝐶𝑎2+ (𝑎𝑞) + 2 𝐶𝑙 − (𝑎𝑞)
Dessa forma, o fator de van’t Hoff para o sal é i = 3.
Δ𝑇𝐸 = 1,86.0,2.3 = 1,116 °𝐶
O fator calculado corresponde ao aumento da temperatura de ebulição em relação ao solvente
puro, que é igual a 100°C.
𝑇𝐸 = 100 + 1,116 = 101,116°𝐶
b) A pressão osmótica é calculada pela seguinte expressão.
𝜋 = [𝑠𝑜𝑙𝑢𝑡𝑜]. 𝑅𝑇
O soluto é a sacarose, que não se dissocia, quando dissolvido em água. Portanto, o seu fator de
van’t Hoff é i = 1.
Além disso, note que a temperatura deve ser calculada em Kelvin. Para isso, precisamos somar
273. Logo, a temperatura em Kelvin é T = 27 + 273 = 300 K.
𝜋 = 0,03.0,082.300 ≅ 0,738 𝑎𝑡𝑚
c) Vamos aplicar a expressão para a temperatura de ebulição.
Δ𝑇𝐸 = 𝐾𝐸 . 𝑊. 𝑖
O cloreto de potássio (KCl) se decompõe, liberando 2 íons.
𝐾𝐶𝑙 (𝑎𝑞) → 𝐾 + (𝑎𝑞) + 𝐶𝑙 − (𝑎𝑞)
Dessa forma, o fator de van’t Hoff para o sal é i = 2.
Δ𝑇𝐸 = 1,86.2.2 = 7,44 °𝐶
O fator calculado corresponde ao aumento da temperatura de ebulição em relação ao solvente
puro, que é igual a 100°C.

AULA 10 – SOLUÇÕES 48
Prof. Thiago Cardoso

𝑇𝐸 = 100 + 7,44 = 107,44°𝐶


Gabarito: a) 101,116 °C; b) 0,738 atm; c) 15,5 mg/L; d) 107,44 °C

4.5. Condutividade Elétrica de Soluções Aquosas


De acordo com a Primeira Lei de Ohm, a resistência elétrica de um material é a dificuldade que ele
impõe à passagem de corrente.
𝑈
𝑈 = 𝑅𝐼 ∴ 𝑅 =
𝐼
Assim, dada uma fonte de tensão fixa, quanto maior a resistência do material, menor será a
corrente elétrica que o atravessará.
A Segunda Lei de Ohm estabelece que a resistência de um material está relacionada com a sua
geometria. Para um condutor cilíndrico, podemos escrever que ela é proporcional ao comprimento do
filamento e inversamente proporcional à sua área.

O parâmetro ρ é conhecido como resistividade é uma propriedade intensiva, isto é, independe


do tamanho da amostra. É uma característica do material em estudo. Na Química, o parâmetro que nos
interessa é a condutividade, que é o inverso da resistividade.
𝜎 = 𝜌−1
Como estudado no capítulo sobre metais, eles são os melhores condutores da natureza. Porém,
existe outros materiais que também apresentam a propriedade de permitir a passagem de corrente
elétrica.
Para que um material qualquer seja condutor de eletricidade, é necessário que ele apresente
portadores de carga livres. Esses portadores de carga podem ser:
• elétrons;
• íons.
A água pura tem condutividade muito baixa, que se deve principalmente à presença de pequenas
concentrações de íons H+ e OH–.
Porém, a condutividade da água pode ser melhorada significativamente, se for acrescentado um
soluto eletrolítico, isto é, um soluto que é capaz de produzir íons em solução aquosa. Os principais
exemplos de solutos eletrolíticos são os ácidos, as bases e os sais.

AULA 10 – SOLUÇÕES 49
Prof. Thiago Cardoso

4.5.1. Mobilidade Iônica


A condutividade elétrica consiste justamente em íons se movendo pela solução e transportando a
corrente elétrica. Desse modo, um dos principais fatores que influencia a condutividade de uma solução
aquosa é a mobilidade dos íons ali presentes.
Em uma solução diluída, os íons presentes interagem pouco entre si, sendo muito mais comuns as
interações íon-solvente. Estudando esse fenômeno, a Lei de Kohlrausch estabelece que a condutividade
elétrica de uma solução diluída é dada pela soma das contribuições de condutividade de cada íon presente
em solução. Assim, podemos escrever:

Λ = 𝑣+ 𝜆+ + 𝑣− 𝜆−
Nessa expressão, λ+ e λ– são as condutividades limites dos cátions e ânions, respectivamente.
O principal fator que influencia na condutividade limite é a carga iônica. De maneira geral, íons
que possuem cargas com o mesmo valor, em módulo, possuem condutividades limites bastante
semelhantes. Naturalmente, quanto maior a carga do íon, maior será a sua facilidade de percorrer
diferentes de potencial e conduzir corrente elétrica.
Tabela 5: Condutividades Limites de Diversos Cátions e Ânions em mS.m².mol–1

Cátions λ+ Ânions λ–

𝑯𝟑 𝑶 + 34,96 𝑶𝑯− 19,91

𝑳𝒊+ 3,87 𝑪𝑯𝟑 𝑪𝑶𝑶− 4,09

𝑵𝒂+ 5,01 𝑵𝑶−


𝟑 7,15

𝑨𝒈+ 6,19 𝑪𝒍− 7,68

𝑲+ 7,35 𝑩𝒓− 7,84

𝒁𝒏𝟐+ 10,56 𝑺𝑶𝟐−


𝟒 16,00

𝑴𝒈𝟐+ 10,62 𝑭𝒆(𝑪𝑵)𝟒−


𝟔 44,2

𝑩𝒂𝟐+ 12,72

Um ponto que chama a atenção nos íons da água é que eles possuem condutividades limites muito
acima da condutividade de outros íons de carga unitária. Uma das explicações para isso é que eles podem
transportar cargas sem se deslocar. Para isso, basta interagir com uma molécula vizinha por meio de
transferência de íons H+.

AULA 10 – SOLUÇÕES 50
Prof. Thiago Cardoso

Como a condutividade se relaciona diretamente com a mobilidade dos íons, deve-se esperar que
o aumento da viscosidade do meio e o aumento do tamanho dos íons resultarão em menor condutividade
elétrica da solução. Porém, é preciso tomar cuidado, porque não se deve considerar os raios iônicos
tabelados, mas sim os raios hidrodinâmicos.
Os raios hidrodinâmicos levam em consideração que os íons são solvatados por moléculas do
solvente. Íons muito pequenos geram um campo elétrico muito intenso, e, com isso, tendem a atrair
muitas moléculas de solvente para si.
Por exemplo, o íon sódio (Na+) é menor que o íon prata (Ag+). Por isso, o íon sódio (Na+) atrai mais
moléculas de água que o íon prata. Essa é uma das explicações para o fato de que os sais de sódio são
sempre mais solúveis que os correspondentes sais de prata.
Porém, no tange à condutividade, o íon sódio arrastará consigo uma quantidade maior de
moléculas do solvente, o que aumenta o seu tamanho efetivo em solução, dificultando a sua mobilidade.
Desse modo, embora haja uma diferença significativa entre os raios iônicos, a diferença nos raios
hidrodinâmicos é bem menor. Desse modo, a condutividade iônica dos dois íons é igual.
Um caso particularmente interessante são os sais orgânicos. Nesse caso, realmente pode-se dizer
que o comprimento do ânion afeta negativamente sua mobilidade. A explicação para isso é que, como a
cadeia carbônica é apolar, a solvatação acontece somente na extremidade carboxila.
Como todos os ácidos monocarboxílicos possuem praticamente a mesma carboxila, a solvatação
é a mesma para todos eles. Desse modo, o aumento da cadeia carbônica resultará igualmente em
aumento do raio iônico e aumento do raio hidrodinâmico.

Figura 24: Redução da Mobilidade provocada pelo aumento da cadeia carbônica

AULA 10 – SOLUÇÕES 51
Prof. Thiago Cardoso

(Estratégia Militares – TFC - Inédita)


Determine a condutividade de uma solução aquosa 0,01 mol/L de cloreto nitrato de potássio.
Dados: λ0 (K+) = 7,35 mS.m².mol–1; λ0 (NO3–) = 7,15 mS.m².mol–1

Comentários
A condutividade limite dessa solução é dada pela soma das contribuições parciais de cada íon, que
é expressa por:
Λ = λ+ [𝐾 + ] + 𝜆− [𝑁𝑂3− ]
Agora, basta substituir os dados fornecidos no enunciado. Porém, devemos tomar o cuidado,
porque as concentrações foram fornecidas em mol/L, enquanto as condutividades molares foram
fornecidas em metro. Por isso, devemos converter as concentrações para mol/m³.
0,1 𝑚𝑜𝑙 0,1 𝑚𝑜𝑙
[𝐾 + ] = [𝑁𝑂3− ] = = −3 3 = 10−1+3 𝑚𝑜𝑙. 𝑚−3 = 102 𝑚𝑜𝑙. 𝑚−3
1𝐿 10 𝑚
Agora, podemos substituir na expressão da condutividade limite.
Λ = λ+ [𝐾 + ] + 𝜆− [𝑁𝑂3− ]
Λ = 7,35.102 + 7,15.102 = 14,5.102 = 1450 𝑚𝑆. 𝑚−1 = 1,45 𝑆. 𝑚−1
Gabarito: 1,45 S.m–1

4.5.2. Condutividade de Soluções Concentradas


Em uma solução aquosa concentrada, a condutância molar da solução diminui em relação ao que
seria esperado pelas contribuições individuais de cada íon, porque começa a haver interações entre os
íons. Quando dois íons interagem, eles se atraem, dificultando que eles percorram a solução.
A redução da condutância real em relação à condutância limite (ou esperada) da solução é dada
pela Lei de Kohlrausch, que estabelece um decréscimo linear com a raiz quadrada da concentração:

Λreal = Λ𝑙𝑖𝑚𝑖𝑡𝑒 − 𝑏√𝐶


Vale ressaltar que o que diminui é a condutância molar da solução, isto é, a condutividade total da
solução dividida pela concentração em mol/L. Certamente, a condutividade propriamente dita da solução
aumenta com a concentração do soluto, tendo em vista que serão mais íons disponíveis para conduzir
corrente elétrica.
Vejamos dados sobre as condutividades molares em S.m².mol–1 para diversas soluções aquosas.

AULA 10 – SOLUÇÕES 52
Prof. Thiago Cardoso

Tabela 6: Condutividade Molar de Diversas Soluções Aquosas

Concentração / mol L–1 KCl HCl CH3COOH

𝟎, 𝟎𝟎𝟏𝟎 1,57.10−6 4,22.10−6 4,90.10−7

𝟎, 𝟎𝟏𝟎 1,41.10−6 4,12.10−6 1,70.10−7

𝟎, 𝟏𝟎 1,29.10−6 3,91.10−6 5,00.10−8

Esse efeito da redução de condutividade molar é geralmente ignorado nas questões. Porém, caso
seja fornecido o parâmetro b, fique atento, pois você deverá considerar esse efeito.

5. Lista de Questões Propostas


CONSTANTES

Constante de Avogadro (NA) = 6,02 x 1023 mol-1

Constante de Faraday (F) = 9,65 x 104 °C mol-1 = 9,65 x 104 A s mol-1 = 9,65 x 104 J V-1 mol-1

Volume molar de gás ideal = 22,4 L (CNTP)

Carga elementar = 1,602 x 10-19 C

Constante dos gases (R) = 8,21 x 10-2 atm L K-1 mol-1 = 8,31 J K-1 mol-1 = 1,98 cal K-1 mol-1

Constante gravitacional (g) = 9,81 m s-2

Constante de Planck (h) = 6,626 x 10-34 m2 kg s-1

Velocidade da luz no vácuo = 3,0 x 108 m s-1

Número de Euler (e) = 2,72

DEFINIÇÕES
Presão: 1 atm = 760 mmHg = 1,01325 x 105 N m-2 = 760 Torr = 1,01325 bar
Energia: 1 J = 1 N m = 1 kg m2 s-2
Condições normais de temperatura e pressão (CNTP): 0°C e 760 mmHg
Condições ambientes: 25 °C e 1 atm
Condições padrão: 1 bar; concentração das soluções = 1 mol L-1 (rigorosamente: atividade unitária das
espécies); sólido com estrutura cristalina mais estável nas condições de pressão e temperatura em
questão
(s) = sólido. (l) = líquido. (g) = gás. (aq) = aquoso. (CM) = circuito metálico. (conc) = concentrado.
(ua) = unidades arbitrárias. [X] = concentração da espécie química em mol L-1

AULA 10 – SOLUÇÕES 53
Prof. Thiago Cardoso

MASSAS MOLARES

Elemento Número Massa Molar (g Elemento Número Massa Molar (g


Químico Atômico mol-1) Químico Atômico mol-1)

H 1 1,01 Mn 25 54,94

Li 3 6,94 Fe 26 55,85

C 6 12,01 Co 27 58,93

N 7 14,01 Cu 29 63,55

O 8 16,00 Zn 30 65,39

F 9 19,00 As 33 74,92

Ne 10 20,18 Br 35 79,90

Na 11 22,99 Mo 42 95,94

Mg 12 24,30 Sb 51 121,76

Al 13 26,98 I 53 126,90

Si 14 28,08 Ba 56 137,33

S 16 32,07 Pt 78 195,08

Cl 17 35,45 Au 79 196,97

Ca 20 40,08 Hg 80 200,59

(Estratégia Militares – TFC - Inédita)

Sabendo-se que a solubilidade do cloreto de sódio em água é igual a 40 g/L, determine se as soluções formadas são
saturadas, insaturadas ou supersaturadas:

a) 3g de cloreto de sódio e 100 mL de água

b) 2g de cloreto de sódio e 50 mL de água

c) 2 mol de cloreto de sódio e 3 L de água

d) 1 mol de cloreto de sódio e 2 L de água

e) 5 g de cloreto de sódio e 100 mL de água

Dados: Na = 23 g/mol, C = 35,5 g/mol

(Estratégia Militares – TFC - Inédita)

AULA 10 – SOLUÇÕES 54
Prof. Thiago Cardoso

A solubilidade de um determinado sal em água é igual a 2 g/L. Dessa forma, ao se misturar 1,5 g de sal a 500 mL de água,
forma-se uma solução:

a) insaturada.

b) saturada sem corpo de fundo.

c) saturada com corpo de fundo.

d) supersaturada.

(Estratégia Militares – TFC - Inédita)

Converta as seguintes concentrações de g/L para mol/L

a) 53 mg/L de carbonato de sódio

b) 88 mg/L de dióxido de carbono

c) 1 g/L de nitrato de potássio

d) 11,7 g/L de cloreto de sódio

Dados de Massas Atômicas: Na = 23 g/mol, K = 39 g/mol, C = 12 g/mol, N = 14 g/mol, O = 16 g/mol, C = 35,5 g/mol.

(Estratégia Militares – TFC - Inédita)

Converta as seguintes concentrações de mol/L para g/L

a) 0,025 mol/L de hidrogenocarbonato de sódio

b) 4.10–3 mol/L de nitrito de cálcio

c) 5.10–5 mol/L de fosfato de cálcio

d) 0,4 mol/L de cloreto de potássio

Dados de Massas Atômicas: Na = 23 g/mol, K = 39 g/mol, C = 12 g/mol, N = 14 g/mol, O = 16 g/mol, C = 35,5 g/mol, H = 1
g/mol, Ca = 40 g/mol, P = 31 g/mol.

(Estratégia Militares – TFC - Inédita)

Deseja-se preparar uma solução 0,05 mg/L de cloreto de sódio a partir da diluição de 10 mL de uma solução 1 mg/L do sal
em água pura. Assinale a alternativa que indica o volume de água que deve ser adicionado para produzir a diluição
desejada:

a) 20 mL

b) 90 mL

c) 140 mL

d) 190 mL

e) 200 mL

(Estratégia Militares – TFC - Inédita)

AULA 10 – SOLUÇÕES 55
Prof. Thiago Cardoso

Considere que a solubilidade do cloreto de sódio em água é igual a 40 g/L e que uma amostra de água do mar tem uma
concentração do sal igual 10 g/L. Uma técnica muito utilizada para a obtenção do sal é a evaporação da água do mar. Uma
amostra de 20 L de água do mar é posta para evaporar, assinale a alternativa que indica a quantidade de água que precisa
ser evaporada até que o sal comece a cristalizar:

a) 2,5 L

b) 5 L

c) 10 L

d) 15 L

e) 17,5 L

(Estratégia Militares – TFC - Inédita)

Determine:

a) A temperatura de ebulição de uma solução aquosa 0,2 mol/kg de cloreto de cálcio.

b) A pressão osmótica de uma solução 0,03 mol/L de uma solução aquosa de sacarose a 27°C.

c) A temperatura de ebulição de uma solução 2 mol/kg de uma solução aquosa de cloreto de sódio.

Dados de Massas Atômicas: KE = 1,86 °C.molal –1; R = 0,082 atm.L/(mol.K) = 8,31 J/(mol.K)

(Estratégia Militares – TFC - Inédita)

A pressão osmótica de uma solução:

a) depende do tipo de soluto.

b) depende exclusivamente da concentração molar do soluto e de sua dissociação.

c) é invariante com a temperatura.

d) é a pressão necessária para promover a osmose.

e) 0,1 mol/L de glicose é igual à pressão osmótica de uma solução 0,1 mol/L de cloreto de sódio.

(Estratégia Militares – TFC - Inédita)

Determine:

a) A temperatura de ebulição de uma solução aquosa 0,2 mol/kg de cloreto de cálcio.

b) A pressão osmótica de uma solução 0,03 mol/L de uma solução aquosa de sacarose a 27°C.

c) A temperatura de ebulição de uma solução 2 mol/kg de uma solução aquosa de cloreto de sódio.

Dados de Massas Atômicas: KE = 1,86 °C.molal –1; R = 0,082 atm.L/(mol.K) = 8,31 J/(mol.K)

(Estratégia Militares – TFC - Inédita)

Determine a condutividade de uma solução aquosa 0,01 mol/L de cloreto nitrato de potássio.

AULA 10 – SOLUÇÕES 56
Prof. Thiago Cardoso

Dados: λ0 (K+) = 7,35 mS.m².mol–1; λ0 (NO3–) = 7,15 mS.m².mol–1

(Estratégia Militares – TFC – Inédita)

O soro fisiológico é uma solução isotônica em relação aos líquidos corporais, que é muito utilizada para limpeza de
ferimentos, nebulização, enxágue de lentos de contato e higienização nasal.

Ela consiste em uma solução aquosa 0,9% (massa em volume) de cloreto de sódio. Assinale a alternativa que apresenta a
concentração molar dessa solução:

a) 0,015 mol/L

b) 0,045 mol/L

c) 0,15 mol/L

d) 0,25 mol/L

e) 0,45 mol/L

(Estratégia Militares – TFC – Inédita)

No Brasil, existem duas formas de medir o teor de etanol em uma solução com a água: os graus INPM, que correspondem
ao teor em massa, e os graus Gay-Lussac, que correspondem ao teor em volume.

O álcool gel 70% em volume (ou 70 °GL) é bastante utilizado para a assepsia de partes do corpo e de objetos para a
prevenção de doenças.

Considerando que a densidade do etanol é igual a 0,8 g/cm³ e a densidade da água é igual a 1,0 g/cm³, o teor desse mesmo
álcool em graus INPM é:

a) 65%

b) 67%

c) 70%

d) 73%

e) 75%

(Estratégia Militares – TFC – Inédita)

Em uma aula prática de química, o professor forneceu a um grupo de alunos 100 mL de uma solução 1 mol.L –1 de sacarose.
Ele deseja que os alunos produzissem uma solução 0,2 mol.L–1 da substância por meio da diluição dessa solução
concentrada em água destilada.

Para isso, os alunos devem adicionar um volume de água igual a:

a) 40 mL

b) 90 mL

c) 400 mL

d) 900 mL

e) 1400 mL

AULA 10 – SOLUÇÕES 57
Prof. Thiago Cardoso

(Estratégia Militares – TFC – Inédita)

A osmose reversa é uma técnica muito utilizado no tratamento de água, que consiste em fazer passar a água ainda
contaminada com sais por uma membrana semipermeável. No interior da membrana, será deixada uma pequena
quantidade de água destilada.

A tendência natural da osmose seria que as moléculas de água atravessassem a membrana indo do meio hipotônico para o
meio hipertônico. Porém, quando se aplica uma pressão sobre a membrana, haverá um fluxo no sentido inverso, de modo
que a água seguirá para o meio hipotônico, aumentando a quantidade de água destilada.

O soro fisiológico é uma solução é uma solução 0,15 mol/L de cloreto de sódio (NaC). Assinale a alternativa que apresenta
a pressão necessária para realizar a osmose reversa nessa solução a 27 °C:

a) 3,7 atm

b) 4,5 atm

c) 5,2 atm

d) 6,5 atm

e) 7,4 atm

(Estratégia Militares – TFC – Inédita)

A água de um aquário marinho pode ser considerada como uma solução 35 gramas de diversos sais, cuja massa molar
média é igual a 70 g/mol e fator de van’t Hoff médio igual a 2,2, em 900 mL de água destilada.

Sabendo que a água pura tem pressão de vapor à temperatura ambiente igual a 25 mmHg, assinale a alternativa que
apresenta a pressão de vapor da água do aquário marinho:

a) 23,0 mmHg

b) 23,5 mmHg

c) 24,0 mmHg

d) 24,5 mmHg

e) 25,0 mmHg

(Estratégia Militares – TFC – Inédita)

Sabendo que o coeficiente de solubilidade do cloreto de sódio é de 36 g/100 g de água a 20 °C, assinale a alternativa que
apresenta a temperatura de ebulição de uma solução saturada desse sal.

Determine a temperatura de ebulição dessa solução:

Dado: Constante Ebuliscópica da Água = 0,52 °C.molal–1

a) 100 °C

b) 101,8 °C

c) 103,2 °C

d) 105 °C

AULA 10 – SOLUÇÕES 58
Prof. Thiago Cardoso

e) 106,4 °C

(Estratégia Militares – TFC – Inédita)

A água oxigenada é uma solução aquosa do peróxido de hidrogênio (H2O2), que possui várias aplicações práticas, como
antisséptico e também como agente branqueador para tecidos e cabelos.

Uma solução aquosa de água oxigenada é conhecida como 18 volumes, porque 1 litro dessa solução é capaz de produzir 18
litros de gás oxigênio, por meio da seguinte reação:

H2O2(aq) → H2O (l) + 1/2 O2 (g)

Sabendo disso, é possível afirmar que a concentração em massa dessa solução é aproximadamente:

Dado: Volume molar nas CNTP = 22,4 L.mol–1

a) 27 g/L

b) 36 g/L

c) 54 g/L

d) 72 g/L

e) 90 g/L

(Estratégia Militares – TFC – Inédita)

A pressão de vapor da água pura a 25 °C é igual a 24 mmHg. São dissolvidos 11,7 gramas de cloreto de sódio (NaC) em 900
mL de água. Assinale a opção que apresenta o valor aproximado do abaixamento da pressão de vapor da solução:

a) 0,10 mmHg

b) 0,19 mmHg.

c) 0,29 mmHg

d) 0,38 mmHg

e) 0,54 mmHg

(Estratégia Militares – TFC – Inédita)

A água oxigenada é uma solução aquosa de peróxido de hidrogênio que é bastante utilizada como antisséptico e
desinfectante.

Sabendo que o peróxido de hidrogênio se decompõe por meio da seguinte reação não balanceada:

H2O2 (aq) → H2O () + O2 (g)

A concentração de uma solução de água oxigenada é igual a n volumes, porque o volume de gás liberado é igual a n vezes
o volume inicial da solução.

Dessa forma, a concentração em massa de peróxido de hidrogênio correspondente a uma amostra de água oxigenada 9,76
volumes é igual a:

Dado: Volume Molar = 24,4 L.mol–1.

AULA 10 – SOLUÇÕES 59
Prof. Thiago Cardoso

a) 13,6 g/L.

b) 20,4 g/L

c) 27,2 g/L

d) 34,0 g/L

e) 54,4 g/L

(Estratégia Militares – TFC – Inédita)

Uma salmoura foi preparada com a dissolução de 5,85 gramas de cloreto de sódio em 100 mL de água. Assinale a
alternativa que indica o aumento da temperatura de ebulição do solvente provocado pela dissolução do soluto.

Dado: Constante Ebulioscópica da Água = 0,5 °C. kg. mol –1

a) 0,25 °C

b) 0,5 °C

c) 1,0 °C

d) 1,5 °C

e) 2,5 °C

(Estratégia Militares – TFC – Inédita)

Um professor de Química estava desconfiado de que sua aliança de casamento era falsificada. Segundo o vendedor, a joia
era constituída de uma mistura de 77,2% de ouro em massa e 22,8% de uma liga metálica, formada principalmente por
cobre e níquel.

Porém, devido à sua desconfiança, o professor resolveu medir a densidade da aliança. Para isso, ele utilizou uma balança
de precisão e registrou que a sua massa era de aproximadamente 10g. A seguir, ele mediu o volume, imergindo a aliança
em um béquer graduado.

Sabendo que a densidade do ouro é igual a 19,3 g/cm³ e que a densidade da liga metálica é igual a 7,6 g/cm³, o professor
atestou que sua aliança era legítima. Dessa forma, o volume da aliança medido pelo professor é aproximadamente igual a:

a) 0,4 mL

b) 0,5 mL

c) 0,6 mL

d) 0,7 mL

e) 0,8 mL

(Estratégia Militares – TFC – Inédita)

O titânio é um material muito útil, pois ele pode ser adicionado a ligas metálicas, como o aço, de modo a lhe conferir
propriedades muito interessantes, tornando o material mais leve e resistente.

Uma das propriedades mais interessantes desse material é a sua baixa densidade. Considere que seja feita uma liga
metálica a partir da mistura de 18 g de titânio a um volume de 6 cm³ de aço.

AULA 10 – SOLUÇÕES 60
Prof. Thiago Cardoso

Assinale a alternativa que indica o valor aproximado da densidade da liga metálica formada:

a) 5,0 g/cm³

b) 5,5 g/cm³

c) 6,0 g/cm³

d) 6,5 g/cm³

e) 7,0 g/cm³

(Estratégia Militares – TFC – Inédita)

A densidade é uma das características mais importantes de uma liga metálica. Quando uma liga metálica é disposta pela
proporção em massas dos metais constituintes, a densidade da liga metálica pode ser calculada como sendo
aproximadamente igual à média harmônica ponderada das densidades dos dois metais.

Dentre os metais, o ouro se destaca por apresentar uma densidade muito superior aos demais. Como são poucos os
materiais que apresentam uma densidade superior à do metal, é muito difícil fraudá-la.

Porém, um ladrão desejava falsificar uma barra de ouro e, para isso, projetou uma liga metálica formada por platina e
cobre, que devia ter a proporção exata dos dois elementos para imitar a densidade do ouro.

Assinale a alternativa que indica o teor de platina que deve ser utilizado na liga para fraudar a densidade do ouro:

a) 62,5%

b) 75%

c) 80,0%

d) 87,5%

e) 92,5%

(Estratégia Militares – TFC – Inédita)

Na Química, é muito importante conhecer os principais fatores limitantes da reação. Entre eles, está a quantidade dos
reagentes que foram adicionados.

Como os reagentes somente podem reagir na proporção estequiométrica, se um dos reagentes for colocado em uma
proporção inferior, ele será completamente consumido durante a reação e ela vai cessar.

Um químico promoveu a dissolução de uma rocha de 4 gramas de carbonato de cálcio. Para isso, ele utilizou 40 mL de uma
solução 1,0 mol/L de ácido clorídrico. Considerando as limitações de reagentes na reação e que o volume molar dos gases
nas condições da reação é igual a 22,4 L, assinale a alternativa que indica o volume de gás liberado.

Dado: CaCO3 (s) + 2 HCl → CaCl2 (aq) + H2O () + CO2 (g)

AULA 10 – SOLUÇÕES 61
Prof. Thiago Cardoso

a) 300 mL

b) 450 mL

c) 600 mL

d) 750 mL

e) 900 mL

(Estratégia Militares – TFC – Inédita)

Para a construção de um sal sintético a ser utilizado em aquário marinho, um fabricante, entre outros, alguns compostos
químicos. Supondo a dissolução de 39 gramas do sal em 1 litro de água destilada, a água apresentaria a seguinte
composição:

COMPOSIÇÃO QUÍMICA PROVÁVEL (percentual em massa): cloreto de sódio = 27%; cloreto de magnésio = 14,2%; sulfato
de potássio = 3,6%; cloreto de cálcio = 3,1%; bicarbonato de sódio = 0,4%.

CARACTERÍSTICAS FÍSICO-QUÍMICAS: pH medido a 25 °C = 8,3; temperatura da água = 25 °C; índice de refração a 25 °C =


1,025;

Considerando os dados da questão anterior e que o cloreto de magnésio seja a única fonte de magnésio presente no sal e
que a água do aquário tenha sido preparada com 1 litro de água destilada para 39 gramas do sal, podemos concluir que a
concentração final de íons magnésio em mg/L nessa água será:

a) 1200

b) 1250

c) 1300

d) 1350

e) 1400

(Estratégia Militares – TFC – Inédita)

No Brasil, existem duas unidades de medida para a concentração do álcool. Ele pode ser medida em percentual de massa
(°INPM) ou em percentual de volume (°GL).

O álcool gel utilizado para a assepsia apresenta a concentração em volume igual a 70% (ou seja, 70 °GL). No entanto, é
muito comum encontrá-lo também em embalagens que informam o percentual em massa.

a) 75%

b) 72%

c) 70%

d) 67%

AULA 10 – SOLUÇÕES 62
Prof. Thiago Cardoso

e) 65%

(Estratégia Militares – TFC – Inédita)

O balling é uma das técnicas mais importantes de manutenção dos níveis adequados de parâmetros, como o cálcio em um
aquário marinho. Os corais consomem continuamente o cálcio fornecido pelo balling para a construção dos seus
esqueletos de carbonato de cálcio.

Um aquário consome 11,1 mL por dia de uma solução aquosa 20 g/L de cloreto de cálcio. Supondo que 80% do cálcio
consumido no aquário seja pelos corais para a construção do seu esqueleto calcário, determine a quantidade de esqueleto
construída em um dia nesse aquário:

a) 80 mg

b) 120 mg

c) 160 mg

d) 200 mg

e) 240 mg

(Estratégia Militares – TFC – Inédita)

Um objeto de 2800 cm³ foi construído com 75% de ouro em massa e 25% de uma liga metálica. Assinale a alternativa que
indica a massa do objeto.

Dados: densidade do ouro = 20 g/cm³ e densidade da liga metálica = 5 g/cm³

a) 32,0 kg

b) 36,0 kg

c) 38,5 kg

d) 42,0 kg

e) 45,5 kg

(Estratégia Militares – TFC – Inédita)

Um objeto de 2800 cm³ foi construído com 75% de ouro em massa e 25% de uma liga metálica. Assinale a alternativa que
indica a massa do objeto.

Dados: densidade do ouro = 20 g/cm³ e densidade da liga metálica = 5 g/cm³

a) 32,0 kg

b) 36,0 kg

c) 38,5 kg

d) 42,0 kg

e) 45,5 kg

(Estratégia Militares – TFC – Inédita)

AULA 10 – SOLUÇÕES 63
Prof. Thiago Cardoso

Um aquário marinho apresenta 0,018 mg/L do elemento fósforo. Considerando que todo o fósforo esteja na forma de íons
fosfato, determine a concentração do íon fosfato (em mg/L) nesse aquário.

Dados: Massas Molares: P = 31 g/mol, O = 16 g/mol.

a) 0,018

b) 0,025

c) 0,036

d) 0,055

e) 0,078

(Estratégia Militares – TFC – Inédita)

Um aquário marinho apresenta 0,018 mg/L do elemento fósforo. Considerando que todo o fósforo esteja na forma de íons
fosfato, determine a concentração do íon fosfato (em mg/L) nesse aquário.

Dados: Massas Molares: P = 31 g/mol, O = 16 g/mol.

a) 0,018

b) 0,025

c) 0,036

d) 0,055

e) 0,078

(Estratégia Militares – TFC – Inédita)

Os ventos solares são o resultado da emissão contínua de partículas carregadas provenientes da coroa solar. Exemplos
desses efeitos são as caudas de cometas, que têm a sua orientação definida pela direção do vento solar. Na superfície da
Terra, os ventos solares carregam aproximadamente 500 milhões de átomos de hidrogênio por cm² por segundo.
Considerando que esse fluxo seja mantido constante e que a área da cidade de Campinas seja igual a 800 km², determine a
massa aproximada de hidrogênio que incidirá sobre essa cidade no intervalo de 1 ano:

Dado: NA = 6,0.1023

a) 1 kg

b) 2 kg

c) 3 kg

d) 4 kg

e) 5 kg

(Estratégia Militares – TFC – Inédita)

O soro fisiológico é uma solução isotônica de cloreto de sódio a 1% em massa. Considerando que a densidade da solução é
igual a 1,1 g/mL, assinale a alternativa que indica o valor aproximado da concentração molar dessa solução.

a) 0,09 mol/L.

AULA 10 – SOLUÇÕES 64
Prof. Thiago Cardoso

b) 0,12 mol/L.

c) 0,14 mol/L.

d) 0,19 mol/L.

e) 0,22 mol/L.

(Estratégia Militares – TFC – Inédita)

A pressão de vapor da água pura à temperatura ambiente é igual a 25 mmHg. Assinale a alternativa que apresenta a
pressão osmótica de uma solução aquosa 10,1 g/L de nitrato de potássio.

a) 2,4 atm

b) 3,7 atm

c) 4,9 atm

d) 6,1 atm

e) 7,3 atm

(Estratégia Militares – TFC – Inédita)

Uma amostra de 1.10–2 g de um determinado composto orgânico é dissolvida em 500 ml de benzeno a 87°C, resultando
numa solução de pressão osmótica de 0,082 atm. Determine o composto orgânico:

Dado: densidade do benzeno = 0,9 g/cm³

a) metano

b) butano

c) isopropano

d) hexano

e) neo-pentano

(Estratégia Militares – TFC – Inédita)

Sabendo que a água do aquário marinho deve ser preparada com a adição de 40 g de um sal marinho por litro de água
pronta e que o teor ideal de boro ideal na água é igual a 4,4 mg/L, determine o teor em massa do tetraborato de sódio
(Na2B4O7) no sal.

a) 0,05%

b) 0,10%

c) 0,15%

d) 0,20%

e) 0,25%

(ITA – 2001)

AULA 10 – SOLUÇÕES 65
Prof. Thiago Cardoso

Um litro de uma solução aquosa contém 0,30 mol de íons Na +, 0,28mol de Cl-, 0,10mol de ions SO42- e x mol de íons Fe3+. A
concentração de íons Fe3 (em mol/L) presentes nesta solução é:

a) 0,03

b) 0,06

c) 0,08

d) 0,18

e) 0,26

(Estratégia Militares – TFC – Inédita)

Uma amostra de 12 mL de uma solução aquosa de cloreto de ferro (II) foi tratada com uma solução aquosa 0,150 mol/L em
permanganato de potássio em meio ácido. Sabendo que o volume utilizado da solução de permanganato de potássio foi
igual a 4 mL, determine a concentração molar da solução original.

a) 0,05 mol/L

b) 0,10 mol/L

c) 0,15 mol/L

d) 0,20 mol/L

e) 0,25 mol/L

(Estratégia Militares – TFC – Inédita)

O esqueleto de um coral é formada essencial por uma mistura de carbonato de cálcio e magnésio. Sabendo que uma
amostra de 3,52 g desse esqueleto foi tratada com ácido clorídrico em excesso, liberando 0,984 L de um gás à temperatura
de 27 °C, determine o teor de cálcio no esqueleto.

(Estratégia Militares – TFC – Inédita)

A adição de certa massa de cloreto de sódio em água provoca uma redução na pressão parcial da água em 10%. Assinale a
alternativa que indica a porcentagem em massa do cloreto de sódio nessa mistura.

a) 7,5%

b) 12,0%

c) 15,3%

d) 22,5%

e) 26,5%

(Estratégia Militares – TFC – Inédita)

Uma amostra de platina está impregnada de ferro metálico como impureza. A fim de retirar o ferro da liga, adicionou-se
ácido nítrico concentrado, produzindo uma reação com liberação de dióxido de nitrogênio. Sabendo que uma amostra de
10g da liga metálica foi tratada com 400 mL de uma solução 0,8 mol/L de ácido nítrico, determine a pureza do metal:

a) 45%

AULA 10 – SOLUÇÕES 66
Prof. Thiago Cardoso

b) 55%

c) 65%

d) 75%

e) 85%

(Estratégia Militares – TFC – Inédita)

Uma amostra de 10 mL de uma solução aquosa de cloreto de ferro (II) foi tratada com uma solução aquosa 0,100 mol/L em
permanganato de potássio em meio ácido, de acordo com a seguinte reação não balanceada:

Fe2+ (aq) + MnO4 – (aq) + H+ (aq) → Fe3+ (aq) + Mn2+ (aq) + H2O ()

Sabendo que o volume utilizado da solução de permanganato de potássio foi igual a 2 mL, determine a concentração
molar da solução original.

a) 0,05 mol/L

b) 0,10 mol/L

c) 0,15 mol/L

d) 0,20 mol/L

e) 0,25 mol/L

(Estratégia Militares – TFC – Inédita)

Sendo dadas as condutividades molares dos íons bário e cloreto: σ(Ba2+) = 12,7 mS m² mol –1 e σ(Cl–) = 7,65 mS m² mol –1,
assinale a alternativa que apresenta a condutividade de uma solução aquosa 10,4 g/L de cloreto de bário.

(Estratégia Militares – TFC – Inédita)

A condutividade de uma solução aquosa 0,01 mol.L–1 de um ácido CH3COOH é igual a 16 mS.m–1. Conhecendo-se as
condutividades molares dos íons H+ (36 mS m² mol–1) e CH3COO– (4 mS m² mol–1), determine a constante de ionização do
ácido.

(ITA 2021 – 1ª Fase)

Sabe-se que a condutividade molar (Ʌ) de uma solução iônica é dada pela razão entre a condutividade dessa solução (k) e
sua concentração molar. Considere soluções diluídas de CaC2, NaC e KC com iguais concentrações em massa, para as
quais são observadas as seguintes razões entre condutividades molares e entre massas molares (MM):

ꓥ(CaCl2)/ ꓥ(NaCl) = 1,9; ꓥ(CaCl2)/ ꓥ(KCl) = 1,8;

MM(CaCl2)/MM(NaCl) = 1,9; MM(CaCl2)/MM(KCl) = 1,5.

Com base nessas informações, assinale a opção CORRETA entre as condutividades das soluções.

a) kCaC2 = kNaC = kkC

b) kCaC2 = kNaC > kkC

c) kCaC2 > kNaC > kkC

AULA 10 – SOLUÇÕES 67
Prof. Thiago Cardoso

d) kCaC2 < kNaC = kkC

e) kCaC2 < kNaC < kkC

(ITA 2021 – 1ª Fase)

Considere as curvas de solubilidade de sais inorgânicos mostradas na figura. A respeito de alguns destes sias são feitas as
seguintes afirmações.

I. Dissolvendo-se 130g de KNO3 em 200 g de água, a 40 oC, obteremos uma solução saturada com depósito de 70g desta
substância que não será dissolvida.

II. Se dissolvermos 20g de Ce2(SO4)3 em 300g de água a 10 oC e, posteriormente, aquecermos esta solução a 90 oC, haverá
gradativa precipitação da substância.

III. A menor quantidade de água necessária para dissolver completamente 140g de K 2Cr2O7 a 90 oC é, aproximadamente,
150g.

IV. NaNO3 é a substância mais solúvel a 30 oC.

Das afirmações acima, está(ão) CORRETA(S)

a) apenas I, II e IV.

b) apenas I e III.

c) apenas II.

d) apenas III e IV.

e) nenhuma.

𝟔𝟎𝟏𝟎𝟎 = 𝒙𝟐𝟎𝟎 → 𝒙 = 𝟏𝟐𝟎𝒈

Portanto, como a solubilidade do sal é de 120g, teremos como resultado uma solução saturada com depósito
de 10g de KNO3 não dissolvido. Afirmação falsa.

II – Observe que a solubilidade do Ce2(SO4)3 a 10 oC é de 10g/100g H2O

AULA 10 – SOLUÇÕES 68
Prof. Thiago Cardoso

Por regra de 3, obtém-se que a solubilidade para 300g de água será de 30g. Como misturam-se apenas 20g de
sal, todo o Ce2(SO4)3 será dissolvido. Quando se aquece a mistura a 90 oC, a solubilidade do sal reduz para um
valor menor que 5g/100g H2O, de modo que, efetivamente, menos de 15 gramas de Ce2(SO4)3 poderiam estar
dissolvidos. Assim, ocorre precipitação de parte do sal. Afirmação verdadeira.

III – A solubilidade do K2Cr2O7 a 90 oC é pouco menor do que 70g/100g H2O.

Podemos calcular a quantidade mínima de água necessária 𝒚 para dissolver 140g do sal por uma regra de três:

𝟕𝟎𝟏𝟎𝟎 = 𝟏𝟒𝟎𝒚 → 𝒚 ≈ 𝟐𝟎𝟎𝒈

Portanto, a quantia de água necessária destoa muito de 150g. Afirmação falsa.

IV – É possível observar no gráfico que a solubilidade do CaCl2 é caracterizado por uma curva crescente, e já
passa a superar a solubilidade do NaNO3 por volta de 25 oC. A 30 oC, portanto, é esperado que a curva do CaCl2
esteja acima da do NaNO3 , sendo, então, o CaCl2 o sal mais solúvel a 30 oC. Afirmação falsa.

AULA 10 – SOLUÇÕES 69
Prof. Thiago Cardoso

Na temperatura de 30 °C, a solubilidade do cloreto de cálcio é tão elevada que nem aparece no gráfico.

(ITA 2021 – 1ª Fase)

Numa titulação de oxirredução, 50,00 mL de uma solução ácida de Fe(NO 3)2 a 0,38 mol-L-1 foi titulada com uma solução
padronizada de permanganato de potássio a 4,2 x 10-2 mol-L-1, até que a solução resultante adquirisse leve coloração
rósea. Sobre esta titulação, são feitas as seguintes afirmações:

(1) O volume da solução de permanganato de potássio gasto na titulação foi de 100 mL.

(2) O menor coeficiente estequiométrico inteiro para Fe2+ na reação redox balanceada é 7.

(3) No ponto final, o volume total da solução será de 120 mL.

(4) Um precipitado sólido de cor esverdeada será observado como produto dessa reação.

(5) O número total de elétrons envolvidos na reação redox é 22 milimols.

(6) A razão entre os volumes do titulante e do titulado no ponto final é 2,1.

A soma dos números associados às afirmações CORRETAS é igual a

a) 0

b) 1

c) 3

d) 6

e) 11

(IME – 2020 – 1 Fase)

AULA 10 – SOLUÇÕES 70
Prof. Thiago Cardoso

Na figura abaixo, é mostrado o diagrama de fases Temperatura versus Composição (fração molar) de dois líquidos voláteis,
hexano (P.E. = 69 °C) e octano (P.E. = 126 °C), para a pressão de 1 atm.

Considere uma mistura binária líquida ideal de hexano e octano, contendo 20% de hexano. Quando essa mistura é
aquecida, ela entra em ebulição, possibilitando a marcação do ponto A, que representa o líquido  em ebulição e o ponto
B, que representa o vapor  gerado pela vaporização do líquido . Considere, agora, que o vapor  seja condensado e em
seguida vaporizado, gerando o vapor . Com base nessas informações, determine a:

a) composição no ponto B;

b) temperatura aproximada de ebulição da mistura líquida de partida que contém 20% de hexano;

c) composição do líquido formado pela condensação do vapor ;

d) composição do vapor .

(IME – 2020 – 1 Fase)

Um minério de ferro, contendo Fe3O4, foi analisado a partir da dissolução de uma amostra de massa 1,161 g em ácido. Na
dissolução, todo o ferro proveniente do Fe 3O4 foi reduzido a Fe2+. A seguir, a amostra foi titulada com 40 mL de uma
solução 0,025 mol/L de KMnO4, tendo como produtos Mn2+ e Fe3+.

Diante do exposto:

a) escreva a equação iônica global simplificada de oxirredução, balanceada, ocorrida na titulação;

b) determine a porcentagem em massa de Fe 3O4 no minério.

(IME – 2020 – 1 Fase)

Em um experimento em laboratório, tomaram-se duas amostras de 0,177 g de um composto de fórmula C aHbOcNd. Uma
das amostras foi completamente consumida por combustão, gerando 0,264 g de CO 2 e 0,135 g de vapor de água. A outra
reagiu totalmente com compostos não nitrogenados, gerando amônia como único produto nitrogenado, a qual necessitou
de 3 cm3 de uma solução 0,5 mol/L de ácido sulfúrico para sua completa neutralização. Determine a fórmula empírica do
composto.

(IME – 2019 – 1 Fase)

É requerido que fazendas produtoras de leite bovino controlem a acidez do leite que está aguardando o processamento.
Essa acidez é resultante da conversão da lactose em ácido lático (ácido 2-hidroxipropanoico) por ação de microrganismos:

AULA 10 – SOLUÇÕES 71
Prof. Thiago Cardoso

C12H22O11 + H2O → 4C3H6O3

Um fazendeiro decidiu fazer um experimento para determinar a taxa de geração de ácido lático no leite armazenado:
retirou uma amostra de 50 cm3 de leite, cuja concentração de ácido lático é de 1,8 g/L, e, depois de três horas, utilizou 40
cm3 de uma solução 0,1 molar de NaOH para neutralizá-la.

Conclui-se que a taxa média de produção de ácido lático por litro de leite é:

a) 0,25 mg/L.s

b) 0,33 mg/L.s

c) 0,50 mg/L.s

d) 0,67 mg/L.s

e) 1,00 mg/L.s

(ITA – 2018)

São feitas as seguintes proposições a respeito de propriedades coligativas:

I. A pressão osmótica depende do tipo de solvente para um dado soluto.

II. A criometria usa o abaixamento do ponto de congelamento do solvente para medir a massa molar do soluto.

III. Na ebuliometria, a variação da temperatura de ebulição depende da concentração molal de soluto não volátil utilizado.

IV. Na tonometria, ocorre abaixamento da pressão de vapor de uma solução que contém um soluto não volátil, em relação
ao solvente puro.

Das proposições acima é(são) CORRETA(S)

a) apenas I.

b) apenas I e III.

c) apenas II, III e IV.

d) apenas II e IV.

e) todas.

(IME – 2019 – 1ª Fase – adaptada)

Admita que uma solução aquosa 0,0400 molar de ácido tricloroacético congele a –0,1395 ºC. Considere, ainda, que a
constante de abaixamento do ponto de congelamento (Kc) da água seja 1,860 ºC.kg.mol–1 e que 1,00 L de solução contenha
1,00 kg de solvente. Calcule o grau de ionização ácido tricloroacético nessa solução.

(ITA – 2017)

A adição de certa massa de etanol em água diminui a temperatura de congelamento do solvente em 18,6 °C. Sabendo que
a constante crioscópica da água é de 1,86 °C·kg·mol–1, assinale a porcentagem em massa do etanol nesta mistura.

a) 10,0 %.

b) 18,6 %.

AULA 10 – SOLUÇÕES 72
Prof. Thiago Cardoso

c) 25,0 %.

d) 31,5 %.

e) 46,0 %.

(ITA – 2017)

A pressão de vapor da água pura é de 23,8 torr a 25°C. São dissolvidos 10,0 g de cloreto de sódio em 100,0 g de água pura a
25°C. Assinale a opção que indica o valor do abaixamento da pressão de vapor da solução, em torr.

a) 22,4

b) 11,2

c) 5,6

d) 2,8

e) 1,4

(ITA – 2017)

A reação do mercúrio metálico com excesso de ácido sulfúrico concentrado a quente produz um gás mais denso do que o
ar. Dois terços deste gás são absorvidos e reagem completamente com uma solução aquosa de hidróxido de sódio,
formando 12,6 g de um sal. A solução de ácido sulfúrico utilizada tem massa específica igual a 1,75 g·cm –3 e concentração
de 80 % em massa. Assinale a alternativa que apresenta o volume consumido da solução de ácido sulfúrico, em cm 3.

a) 11

b) 21

c) 31

d) 41

e) 51

(ITA – 2017)

Considere duas soluções, X e Y, de um mesmo soluto genérico. A solução X tem 49% em massa do soluto, enquanto a
solução Y possui 8% em massa do mesmo soluto. Quer-se obter uma terceira solução, que tenha 20% em massa deste
soluto, a partir da mistura de um volume VX da solução X com um volume VY da solução Y. Considerando que todas as
soluções envolvidas exibem comportamento ideal, assinale a opção que apresenta a razão VX/VY CORRETA.

a) 12/29.

b) 29/12.

c) 19/12.

d) 12/19.

e) 8/49.

(IME – 2013)

AULA 10 – SOLUÇÕES 73
Prof. Thiago Cardoso

Um erlenmeyer contém 10,0 mL de uma solução de ácido clorídrico, juntamente com algumas gotas de uma solução de
fenolftaleína. De uma bureta, foi-se gotejando uma solução 0,100 M de hidróxido de sódio até o aparecimento de leve
coloração rósea. Nesse momento, observou-se um consumo de 20,0 mL da solução alcalina. Pode-se afirmar que a
concentração de HC na solução ácida original era de:

Dados:

Massas atômicas: H = 1,00 u, O = 16,0 u, Na = 23,0 u, Cl = 35,5 u

a) 3,65 x 10–3 g/cm3

b) 7,30 x 10–3 g/cm3

c) 4,00 x 10–3 g/cm3

d) 3,20 x 10–3 g/cm3

e) 2,00 x 10–3 g/cm3

(ITA – 2015)

Considere uma solução saturada do sal MX que é pouco solúvel em água destilada a 25 °C. Seja y a condutância da água
destilada é (y + 2,0.10-7) ohm-1 cm-1 a condutância da solução. Sabendo que as condutividades iônicas molares dos íons M+
e X– são, respectivamente, 60 ohm-1cm² mol-1 e 40 ohm-1cm² mol-1, determine a solubilidade do MX em água em mol dm -3.

(ITA SP/2013)

Nas condições ambientes, 0,500g de um resíduo sólido foi dissolvido completamente em aproximadamente 13 mL de uma
mistura dos ácidos nítrico e fluorídrico (HNO3 : HF = 10 : 3). A solução aquosa ácida obtida foi quantitativamente
transferida para um balão volumétrico com capacidade de 250 mL e o volume do balão completado com água
desmineralizada. A análise quantitativa dos íons de ferro na solução do balão revelou que a quantidade de ferro nesta
solução era igual a 40,0 mgL–1. Respeitando o número de algarismos significativos, determine a quantidade de ferro (em %
em massa) presente no resíduo sólido. Mostre o raciocínio e os cálculos realizados para chegar à sua resposta.

(ITA SP/2014)

A pressão de vapor de uma solução ideal contendo um soluto não-volátil dissolvido é diretamente proporcional à

a) fração molar do soluto.

b) fração molar do solvente.

c) pressão osmótica do soluto.

d) molaridade, em mol  L–1, do solvente.

e) molalidade, em mol  kg–1, do solvente.

(ITA – 2013)

Uma solução líquida constituída por dois componentes A e B e apresentando comportamento ideal, conforme Lei de
Rauolt, está em equilíbrio com seu vapor. Utilizando a notação:

xA e xB para as respectivas frações em mol para as respectivas frações em A e B na solução líquida,

pA e pB para as respectivas pressões de vapor de A e B no vapor em equilíbrio com a solução líquida, e

AULA 10 – SOLUÇÕES 74
Prof. Thiago Cardoso

p 0A e p 0B para as respectivas pressões de vapor de A puro e B puro numa mesma temperatura,

assinale a opção que apresenta a relação CORRETA para a pressão de vapor de A (p A) em equilíbrio com a solução líquida.

a) pA = p 0A (1 – xA)

b) PA = p 0B (1 – xB)

c) pA = p 0B xA

d) pA = p 0A xA

e) pA = p 0B xB

(ITA – 2015)

Assinale a opção que apresenta os instrumentos de medição de volume mais indicados para a realização de uma titulação.

a) Bureta e erlenmeyer

b) Proveta e erlenmeyer

c) Pipeta volumétrica e erlenmeyer

d) Proveta e béquer

e) Pipeta volumétrica e béquer

(ITA – 2011)

A solução aquosa 6% em massa de água oxigenada (H 2O2) é geralmente empregada como agente branqueador para
tecidos e cabelos. Pode-se afirmar que a concentração aproximada dessa solução aquosa, expressa em volumes, é

a) 24.

b) 20.

c) 12.

d) 10.

e) 6.

Obs.: Considere o volume molar de gases nas CNTP é igual a 22,4 L/mol.

(ITA – 2011)

A 25 °C, as massas específicas do etanol e da água, ambos puros, são 0,8 g cm –3 e 1,0 g cm–3, respectivamente. Adicionando
72 g de água pura a 928 g de etanol puro, obteve-se uma solução com 1208 cm3 de volume.

Assinale a opção que expressa a concentração desta solução em graus Gay-Lussac (°GL).

a) 98

b) 96

AULA 10 – SOLUÇÕES 75
Prof. Thiago Cardoso

c) 94

d) 93

e) 72

(Estratégia Militares – TFC - Inédita)

Considere uma solução de um soluto X em um solvente Y. A soma dos títulos do soluto e do solvente em número de mols,
em massa e em volume devem ser necessariamente iguais a 100%? Justifique a sua resposta.

(ITA – 2008)

Qual das substâncias abaixo não é empregada na fabricação da pólvora negra?

a) trinitrotolueno

b) enxofre

c) carvão

d) nitrato de sódio

e) nitrato de potássio

(IME – 2008)

Dispõe-se de uma mistura sulfonítrica de composição mássica igual a 60% de H 2SO4, 11,2% de HNO3 e 28,8% de H2O. A
1000 kg desta mistura são adicionados 100 kg de solução de HNO3 88% (m/m) e 200 kg de H2SO4 60% (m/m). Indique a
composição mássica da mistura sulfonítrica final.

a) 55,4% de H2SO4; 15,4% de HNO3; 29,2% de H2O.

b) 59,6% de H2SO4; 16,6% de HNO3; 23,8% de H2O.

c) 59,0% de H2SO4; 16,4% de HNO3; 24,6% de H2O.

d) 55,9% de H2SO4; 15,5% de HNO3; 28,6% de H2O.

e) 64,3% de H2SO4; 15,1% de HNO3; 20,6% de H2O.

(IME RJ/2007)

Oleum, ou ácido sulfúrico fumegante, é obtido através da absorção do trióxido de enxofre por ácido sulfúrico. Ao se
misturar oleum com água obtém-se ácido sulfúrico concentrado. Supondo que uma indústria tenha comprado 1.000 kg de
oleum com concentração em peso de trióxido de enxofre de 20% e de ácido sulfúrico de 80%, calcule a quantidade de água
que deve ser adicionada para que seja obtido ácido sulfúrico com concentração de 95% em peso.

Dados:

Massas atômicas (u.m.a): S = 32; O = 16; H = 1

a) 42 kg

b) 300 kg

c) 100 kg

AULA 10 – SOLUÇÕES 76
Prof. Thiago Cardoso

d) 45 kg

e) 104,5 kg

(ITA – 2006)

São fornecidas as seguintes informações a respeito de titulação ácido-base:

I – A figura mostra as curvas de titulação de 30,0 mL de diferentes ácidos (I, II, III, IV e V), todos a 0,10 mol L –1, com uma
solução aquosa 0,10 mol L–1 em NaOH.

II – O indicador fenolftaleína apresenta o intervalo de mudança de cor entre pH 8,0 a 10,0, e o indicador vermelho de
metila, entre pH 4,0 a 6,0.

Considerando estas informações, é CORRETO afirmar que:

a) o indicador vermelho de metila é mais adequado que a fenolftaleína para ser utilizado na titulação do ácido IV.

b) o indicador vermelho de metila é mais adequado que a fenolftaleína para ser utilizado na titulação do ácido V.

c) o ácido III é mais forte que o ácido II.

d) os dois indicadores (fenolftaleína e vermelho de metila) são adequados para a titulação do ácido I.

e) os dois indicadores (fenolftaleína e vermelho de metila) são adequados para a titulação do ácido III.

(ITA – 2005)

Esta tabela apresenta a solubilidade de algumas substâncias em água, a 15°C:

Quando 50 mL de uma solução aquosa 0,10 moI L–1 em sulfato de zinco são misturados a 50 mL de uma solução aquosa
0,010 moI L–1 em sulfito de sódio, à temperatura de 15°C, espera-se observar:

AULA 10 – SOLUÇÕES 77
Prof. Thiago Cardoso

a) a formação de uma solução não saturada constituída pela mistura das duas substâncias.

b) a precipitação de um sólido constituído por sulfeto de zinco.

c) a precipitação de um sólido constituído por sulfito de zinco.

d) a precipitação de um sólido constituído por sulfato de zinco.

e) a precipitação de um sólido constituído por sulfeto de sódio.

(ITA – 2004)

Deseja-se preparar 57 gramas de sulfato de alumínio[Al 2(SO4)3] a partir de alumínio sólido (Al) praticamente puro, e ácido
sulfúrico (H2SO4). O ácido sulfúrico disponível é uma solução aquosa 96 % (m/m), com massa específica de 1,4gcm -1.

a) Qual a massa, em gramas, de alumínio necessária para preparar a quantidade de Al 2(SO4)3 especificada? Mostre os
cálculos realizados.

b) Qual a massa, em gramas, de ácido sulfúrico necessária para preparar a quantidade de Al 2(SO4)3 especificada? Mostre os
cálculos realizados.

c) Nas condições normais de temperatura e pressão (CNTP), qual é o volume, em litros, de gás formado durante a
preparação da quantidade de Al2(SO4)3 especificada? Mostre os cálculos realizados.

d) Caso a quantidade especificada de Al2(SO4)3 seja dissolvida em água acidulada, formando 1 L de solução, qual a
concentração de íons Al3+ e de íons SO42– existentes nesta solução?

(ITA – 1999)

Para preparar 80 L de uma solução aquosa 12% (massa/massa) de KOH (massa específica da solução = 1,10 g/cm 3) foram
adicionados x litros de uma solução aquosa 44% (massa/massa) de KOH (massa específica da solução = 1,50 g/cm 3) e y
litros de água deionizada (massa específica = 1,00 g/cm 3). Os valores de x e y são respectivamente:

a) 12 L e 68 L

b) 16 L e 64 L

c) 30 L e 50 L

d) 36 L e 44 L

e) 44 L e 36 L

(ITA – 1998)

Fazendo-se borbulhar gás cloro através de 1,0 litro de uma solução de hidróxido de sódio, verificou-se ao final do
experimento que todo hidróxido de sódio foi consumido, e que na solução resultante foram formados 2,5 mols de cloreto
de sódio. Considerando que o volume não foi alterado durante todo o processo, e que na temperatura em questão tenha
ocorrido apenas a reação correspondente à seguinte equação química, não balanceada:

OH-(aq) + Cl2(g) → Cl-(aq) + ClO3-(aq) + H2O(L)

Qual deve ser a concentração inicial do hidróxido de sódio?

a) 6,0 mol/L

b) 5,0 mol/L

AULA 10 – SOLUÇÕES 78
Prof. Thiago Cardoso

c) 3,0 mol/L

d) 2,5 mol/L

e) 2,0 mol/L

(ITA – 2012 – adaptada)

Uma amostra de 210–2 g de um determinado composto orgânico é dissolvida em 300 mL de água a 25 °C, resultando
numa solução de pressão osmótica 0,027 atm. Pode-se afirmar, então, que o composto orgânico é o(a) :

a) ácido acético (C2H4O2).

b) etileno glicol (C2H6O2).

c) etanol (C2H6O).

d) uréia (CH4N2O).

e) tri-fluor-carbono (CHF3).

(ITA SP/2003)

O abaixamento da temperatura de congelamento da água numa solução aquosa com concentração molal de soluto igual a
0,100 mol kg–1 é

0,55oC. Sabe-se que a constante crioscópica da água é igual a 1,86oC kg mol–1. Qual das opções abaixo contém a fórmula
molecular CORRETA do soluto?

a) [Ag(NH3)2]Cl

b) [Pt(NH3)4Cl2]Cl2

c) Na[Al(OH)4]

d) K3[Fe(CN)6 ]

e) K4[Fe(CN)6]

(ITA – 2005)

Dois frascos abertos, um contendo água pura líquida (frasco A) e o outro contendo o mesmo volume de uma solução
aquosa concentrada em sacarose (frasco B), são colocados em um recipiente que, a seguir, é devidamente fechado. É
CORRETO afirmar, então, que, decorrido um longo período de tempo,

a) os volumes dos líquidos nos frascos A e B não apresentam alterações visíveis.

b) o volume do líquido no frasco A aumenta, enquanto que o do frasco B diminui.

c) o volume do líquido no frasco A diminui, enquanto que o do frasco B aumenta.

d) o volume do líquido no frasco A permanece o mesmo, enquanto que o do frasco B diminui.

e) o volume do líquido no frasco A diminui, enquanto que o do frasco B permanece o mesmo.

(ITA – 2012 – adaptada)

AULA 10 – SOLUÇÕES 79
Prof. Thiago Cardoso

Considere as seguintes misturas (soluto/solvente) na concentração de 10 % em mol de soluto:

I. acetona/clorofórmio

II. água/etanol

III. água/metanol

IV. benzeno/tolueno

V. n-hexano/n-heptano

Assinale a opção que apresenta a(s) mistura(s) para a(s) qual(is) a pressão de vapor do solvente na mistura é
aproximadamente igual à sua pressão de vapor quando puro multiplicada pela sua respectiva fração molar.

a) Apenas I

b) Apenas I, II e III

c) Apenas II e III

d) Apenas IV e V

e) Apenas V

Dica: Caso você não conheça as fórmulas estruturais dos compostos citados, por ainda não ter estudado Química Orgânica,
elas estão transcritas a seguir.

(IME – 2010)

Calcule a massa de 1 L de uma solução aquosa de nitrato de zinco cuja concentração é expressa por 0,643 molar e por
0,653 molal.

(ITA – 2007)

Dois béqueres, X e Y, contêm, respectivamente, volumes iguais de soluções aquosas: concentrada e diluída de cloreto de
sódio na mesma temperatura. Dois recipientes hermeticamente fechados, mantidos à mesma temperatura constante, são

AULA 10 – SOLUÇÕES 80
Prof. Thiago Cardoso

interconectados por uma válvula, inicialmente fechada, cada qual contendo um dos béqueres. Aberta a válvula, após o
restabelecimento do equilíbrio químico, verifica-se que a pressão de vapor nos dois recipientes é Pf .

Assinale a opção que indica, respectivamente, as comparações CORRETAS entre os volumes inicial (VX i) e final (VXf ), da
solução no béquer X e entre as pressões de vapor inicial (PY i) e final (Pf) no recipiente que contém o béquer Y.

a) VXi < VXf e PYi = PYf

b) VXi < VXf e PYi > PYf

c) VXi < VXf e PYi < PYf

d) VXi > VXf e PYi > PYf

e) VXi > VXf e PYi < PYf

(ITA – 2007 – adaptada)

Prepara-se, a 25oC, uma solução por meio da mistura de 25 mL de n-pentano (C5H12) e 45 mL de n-hexano (C6H14).

Dados:

massa específica do n-pentano = 0,63 g/mL;

massa específica do n-hexano = 0,66 g/mL;

pressão de vapor do n-pentano = 511 torr;

pressão de vapor do n-hexano = 150 torr.

Determine os seguintes valores, mostrando os cálculos efetuados:

a) Fração molar do n-pentano na solução.

b) Pressão de vapor da solução.

c) Fração molar do n-pentano no vapor em equilíbrio com a solução.

(ITA – 2005)

Considere as afirmações abaixo, todas relativas à pressão de 1 atm:

I. A temperatura de fusão do ácido benzóico puro é 122°C, enquanto que a da água pura é 0°C.

II. A temperatura de ebulição de uma solução aquosa 1,00 moI L –1 de sulfato de cobre é maior do que a de uma solução
aquosa 0,10 mol L–1 deste mesmo sal.

III. A temperatura de ebulição de uma solução aquosa saturada em cloreto de sódio é maior do que a da água pura.

IV. A temperatura de ebulição do etanol puro é 78,4°C, enquanto que a de uma solução alcoólica 10% (m/m) em água é
78,2°C.

Das diferenças apresentadas em cada uma das afirmações acima, está(ão) relacionada(s) com propriedades coligativas:

a) apenas I e III.

b) apenas l.

AULA 10 – SOLUÇÕES 81
Prof. Thiago Cardoso

c) apenas II e III.

d) apenas II e IV.

e) apenas III e IV.

(ITA – 2006)

Considere soluções de SiCl4/CCl4 de frações molares variáveis, todas a 25°C. Sabendo que a pressão de vapor do CCl 4 a 25°C
é igual a 114,9 mmHg, assinale a opção que mostra o gráfico que melhor representa a pressão de vapor de CCl 4 (PCCl4) em
função da fração molar de SiCl4 no líquido ( XSiCl4 )

a) b) c)

b) e)

(IME – 2004)

Um calcário composto por MgCO3 e CaCO3 foi aquecido para produzir MgO e CaO. Uma amostra de 2,00 gramas desta
mistura de óxidos foi tratada com 100 cm3 de ácido clorídrico 1,00 molar. Sabendo-se que o excesso de ácido clorídrico
necessitou de 20,0 cm3 de solução de NaOH 1,00 molar para ser neutralizado, determine a composição percentual, em
massa, de MgCO3 e CaCO3 na amostra original desse calcário.

(IME – 2005)

Determine o abaixamento relativo da pressão de vapor do solvente quando 3,04 g de cânfora (C10H16O) são dissolvidos em
117,2 mL de etanol (C a 25oC.

Dado: massa específica do etanol = 0,785 g/mL

(IME – 2004)

Na produção de uma solução de cloreto de sódio em água a 0,90% (p/p), as quantidades de solvente e soluto são pesadas
separadamente e, posteriormente, promove-se a solubilização. Certo dia, suspeitou-se que a balança de soluto estivesse
descalibrada. Por este motivo, a temperatura de ebulição de uma amostra da solução foi medida, obtendo-se 100,14ºC.
Considerando o sal totalmente dissociado, determine a massa de soluto a ser acrescentada de modo a produzir um lote de
1000kg com a concentração correta.

Dado: Constante Ebulioscópica da Água = 0,52 °C/molal

(IME – 2001)

AULA 10 – SOLUÇÕES 82
Prof. Thiago Cardoso

Uma solução contendo 0,994g de um polímero, de fórmula geral (C 2H4)n, em 5,00g de benzeno, tem ponto de
congelamento 0,51ºC mais baixo que o do solvente puro. Determine o valor de n.

Dado: Constante crioscópia do benzeno = 5,10ºC/molal

(IME – 2015 – 1ª Fase)

Um pesquisador verificou, em uma determinada posição geográfica, por meio da análise de amostras de água do mar
extraídas do local, que a massa específica média da água do mar era 1,05 g/mL, a concentração média de espécies
dissolvidas era 0,80 mol/L e a temperatura média era de 290 K. O mesmo pesquisador, com o objetivo de colher água doce
em seu estudo, planeja envolver, com uma membrana semipermeável ideal, uma das extremidades abertas de um longo
tubo, a qual será imersa na água do mar. A que profundidade mínima, em metros, o tubo deveria ser imerso?

a) 1930,0.

b) 183,4.

c) 73,7.

d) 19,4.

e) 9,7.

Dados: R = 0,08 atm.L/(K.mol) = 8,3 J/(K.mol) = 62,3 mmHg.L/(K.mol) ; g = 10 m/s²; 1 atm = 103750 Pa.

Obs.: Essa questão requer o conhecimento de Hidrostática, assunto da Física

(Estratégia Militares – TFC – Inédita)

Uma solução aquosa de 1 L de HC 0,1 mol/L foi titulada com uma solução aquosa de NaOH de mesma concentração
molar. Assinale a alternativa que apresenta a curva de condutância da solução em função do volume de NaOH adicionado.

a)

b)

AULA 10 – SOLUÇÕES 83
Prof. Thiago Cardoso

c)

d)

e)

(Estratégia Militares – TFC – Inédita)

O cloreto de potássio é utilizado como sal light, pois partilha muitas propriedades do íon sódio, inclusive o sabor. A
respeito desses dois importantes cátions, assinale a alternativa correta:

a) Quando uma solução de cloreto de potássio é aquecida, libera uma luz violeta, enquanto o íon sódio libera uma luz
amarela.

b) Os sais do potássio geralmente são mais solúveis que os sais de sódio, pois o potássio apresenta maior raio iônico.

c) É de se esperar que uma solução 1 mol/L de cloreto de potássio seja melhor condutora que uma solução 1 mol/L de
cloreto de sódio, devido às diferenças de mobilidades dos íons.

d) O sódio e o potássio são importantes no mecanismo das contrações musculares, por meio da bomba de sódio e
potássio, mecanismo pelo qual ambos os íons são transferidos por osmose através das membranas celulares.

e) O íon potássio pode ser facilmente precipitado, borbulhando-se H2S em solução aquosa.

AULA 10 – SOLUÇÕES 84
Prof. Thiago Cardoso

(Estratégia Militares – TFC – Inédita)

A fim de determinar o número de unidades estruturais de um polímero derivado do polietileno (CH 2)n, uma amostra de 5,6
g foi dissolvida em 100 mL benzeno. Admitindo que a constante crioscópica do benzeno seja igual 5,10 °C.molal –1 e que o
ponto de congelamento da solução foi superior em 0,51 °C ao ponto de congelamento do benzeno puro, assinale a opção
que indica o número de unidades estruturais do polímero.

Dado: Densidade do benzeno = 0,88 g/cm³

a) 18

b) 30

c) 45

d) 50

e) 60

(Estratégia Militares – TFC – Inédita)

A temperatura de ebulição de uma solução aquosa 0,1 mol/L de um ácido orgânico não-volátil HX é 0,026 °C superior à
temperatura do benzeno puro. Sabendo que a constante ebulioscópica do solvente é igual a 0,52 °C.molal -1, determine o
valor aproximado da constante de dissociação do ácido HX.

a) 6,0.10-4

b) 9,2.10-4

c) 1,1.10-3

d) 4,4.10-3

e) 7,5.10-3

(Estratégia Militares – TFC – Inédita)

A temperatura de ebulição de uma solução aquosa 0,1 mol/L de um ácido orgânico não-volátil HX é 0,026 °C superior à
temperatura do benzeno puro. Sabendo que a constante ebulioscópica do solvente é igual a 0,52 °C.molal-1, determine o
valor aproximado da constante de dissociação do ácido HX.

a) 6,0.10-4

b) 9,2.10-4

c) 1,1.10-3

d) 4,4.10-3

e) 7,5.10-3

(Estratégia Militares – TFC – Inédita)

O nitrogênio em um aquário marinho pode se encontrar na forma de amônia, nitrito, nitrato ou de moléculas orgânicas.
Supondo que o aquário tenha 6,2 mg/L de nitrato e 11 mg/L do elemento nitrogênio no total, determine o percentual de
nitrogênio que se encontra na forma de nitrato:

a) 12,7%

AULA 10 – SOLUÇÕES 85
Prof. Thiago Cardoso

b) 25,4%

c) 32,5%

d) 40,0%

e) 56,3%

(Estratégia Militares – TFC – Inédita)

Uma solução de 20,0 g de qual sal hidratado dissolvido em 200 g H 2O terá o menor ponto de congelamento?

a) CuSO4 . 5 H2O (M = 250)

b) NiSO4 . 6 H2O (M = 263)

c) MgSO4 . 7 H2O (M = 246)

d) Na2SO4 . 10 H2O (M = 286)

e) Todas as soluções apresentam o mesmo ponto de congelamento.

(Estratégia Militares – TFC – Inédita)

Uma luminária caiu em um aquário marinho liberando íons estanhosos em solução. A fim de detectar a quantidade desses
íons que foram dissolvidos, é possível utilizar uma mistura permanganato de potássio e ácido sulfúrico como agente
titulante.

Sabendo o manganês se reduz a íons Mn2+ incolores e que para neutralizar uma amostra de 100 mL da solução aquosa foi
utilizado 1 mL de uma solução aquosa 0,001 mol/L de permanganato de potássio, assinale a alternativa que indica a
concentração aproximada em massa de íons Sn2+ presentes na solução.

a) 1,2 mg/L

b) 1,8 mg/L

c) 2,0 mg/L

d) 2,4 mg/L

e) 3,0 mg/L

(Estratégia Militares – TFC – Inédita)

Determine a condutividade de uma solução aquosa 0,2 mol/L de sulfato de sódio em S.m –1.

Dados: σ (SO42–) = 160 S cm² mol–1; σ (Na+) = 50 S cm² mol–1

a) 4,2

b) 5,2

c) 6,2

d) 52

e) 62

AULA 10 – SOLUÇÕES 86
Prof. Thiago Cardoso

(Estratégia Militares – TFC – Inédita)

Uma amostra de prata metálica foi tratada por excesso de ácido sulfúrico concentrado a quente, consumindo 24,5 cm³ de
uma solução aquosa, cuja densidade é igual a 1,75 g/cm³ e concentração de 80% em massa. Assinale a alternativa que
indica a massa de gás liberada nessa reação.

a) 0,7 g

b) 1,4 g

c) 2,8 g

d) 11,2 g

e) 22,4 g

(Estratégia Militares – TFC – Inédita)

Os corais do gênero Acropora são extremamente sensíveis à presença de íons nitrato, resultantes do ciclo do nitrogênio na
água. Para a manutenção da saúde desses animais, é necessário que a água tenha um teor limite de até 5 mg/L, sendo
mais adequado níveis abaixo de 1 mg/L

Uma forma simples de se controlar o teor de nitrato em aquários marinhos é com o uso da vodka, que pode ser
considerada como uma solução aquosa com teor de 40% de etanol em volume.

O etanol é capaz de provocar a redução dos íons nitrato a nitrogênio gasoso. Sabendo que um aquário de 400 litros tinha
inicialmente um teor de 4,96 mg/L de nitrato e o que aquarista deseja reduzir a 1,24 mg/L, determine o volume de vodka a
ser adicionado.

Dado: densidade do etanol = 0,8 g/mL.

(Estratégia Militares – TFC – Inédita)

O iodeto de potássio é um sal incolor e bastante solúvel em água, comercializado na forma de uma solução aquosa,
vendida comercialmente como lugol. Ele pode ser utilizado para a detecção de íons chumbo (Pb 2+).

Uma forma interessante de visualizar essa reação é utilizando um recipiente cilíndrico raso, como uma placa de Petri, em
que foi colocada uma fina lâmina de água suficiente apenas para cobrir o fundo do recipiente. Nesse recipiente, coloca–se
em dois pontos diametralmente opostos da placa uma pequena quantidade de soluções bastante concentradas de nitrato
de chumbo e de iodeto de potássio.

Nessa situação, forma-se uma listra amarela. A respeito dessa situação:

a) escreva a reação química que explica a formação da substância amarela citada.

b) a listra amarela se formará mais perto do ponto em que foi gotejada a solução de nitrato de chumbo ou do ponto em
que foi gotejada a solução de iodeto de potássio? Faça um esboço de como se apresentará o sistema final.

(Estratégia Militares – TFC – Inédita)

Em um experimento de laboratório, conectou-se um led (diodo emissor de luz) a uma bateria e dois eletrodos imersos em
100 mL de uma solução aquosa de cloreto de potássio 0,1 mol/L.

A intensidade da luz emitida pelo LED é aproximadamente proporcional à raiz quadrada da condutividade dessa solução
aquosa. Por isso, pode-se utilizar uma proveta para adicionar um certo volume de solução 0,8 mol/L de sulfato de prata.

AULA 10 – SOLUÇÕES 87
Prof. Thiago Cardoso

Desenhe um gráfico da intensidade da luz emitida pelo LED em função da quantidade de solução de hidróxido de cálcio
adicionado, respondendo aos seguintes itens:

a) Qual o volume adicionado de Ag2SO4 quando a intensidade luminosa do led é mínima?

b) Qual a relação aproximadamente entre a intensidade do led quando o volume acrescentado da solução de Ag2SO4 for
muito grande e a intensidade inicial do led?

AULA 10 – SOLUÇÕES 88
Prof. Thiago Cardoso

6. Gabarito

1. 1, 3 e 4 insaturadas; 2 e 5 saturadas 30. D


2. C 31. D
3. a) 5.10–4 mol/L; b) 2.10–3 mol/L; c) 9,9.10–3 32. B
mol/L; d) 0,2 mol/L
33. D
4. a) 2,1 g/L; b) 528 mg/L; c) 15,5 mg/L; d) 29,8
g/L 34. C

5. D 35. E

6. D 36. A

7. a) 101,116 °C; b) 0,738 atm; c) 15,5 mg/L; d) 37. B


107,44 °C 38. E
8. B 39. 11,4%
9. a) 101,116 °C; b) 0,738 atm; c) 15,5 mg/L; d) 40. C
107,44 °C
41. B
10. 1,45 S.m–1
42. B
11. C
43. 1,4 S.m–1
12. A
44. 1,7.10–5
13. C
45. B
14. E
46. C
15. D
47. A
16. E
48. a) xoct = 0,5 e xhex = 0,5; b) 104 °C; c) xoct = 0,5 e
17. C xhex = 0,5; d) xoct = 0,2 e xhex = 0,8
18. B 49. a) 5 Fe2+ + MnO4– + 8 H+ → Mn2+ + 5Fe3+ + 4
19. C H2O; b) 33,33%

20. C 50. C2H5NO

21. D 51. C

22. D 52. C

23. D 53. 87,5%

24. B 54. D

25. E 55. E

26. E 56. B

27. C 57. A

28. D 58. B

29. D 59. 2.10–6

AULA 10 – SOLUÇÕES 89
Prof. Thiago Cardoso

60. 2% 81. a) 39% b) 290,8 torr; c) 65%


61. B 82. C
62. D 83. E
63. A 84. CaCO3 = 66,5%; MgCO3 = 33,5%
64. B 85. 1%
65. B 86. 1,17kg
66. sim; sim; não 87. 71
67. A 88. B
68. A 89. A
69. C 90. A
70. D 91. C
71. A 92. C
72. a) 9 g; b) 98 g; c) 11,2 L; d) [Al3+] = 0,333 mol/L; 93. C
SO42-= 0,50 mol/L
94. A
73. B
95. D
74. C
96. E
75. D
97. B
76. B
98. D
77. C
99. 8,625 mL
78. D
100. discursiva
79. 1,1062 kg
101. E
80. B

AULA 10 – SOLUÇÕES 90
Prof. Thiago Cardoso

7. Lista de Questões Comentadas


Essa lista de exercícios foi dividida em duas seções: uma primeira parte com questões inéditas de
nível I e uma segunda parte com questões mais avançadas, inclusive uma bateria bem extensa de questões
do ITA e IME.

7.1. Lista de Questões Nível I


(Estratégia Militares – TFC – Inédita)
O soro fisiológico é uma solução isotônica em relação aos líquidos corporais, que é muito utilizada para
limpeza de ferimentos, nebulização, enxágue de lentos de contato e higienização nasal.
Ela consiste em uma solução aquosa 0,9% (massa em volume) de cloreto de sódio. Assinale a alternativa
que apresenta a concentração molar dessa solução:
a) 0,015 mol/L
b) 0,045 mol/L
c) 0,15 mol/L
d) 0,25 mol/L
e) 0,45 mol/L

Comentários
A proporção massa em volume indica que, para cada 100 mL de solução, encontram-se 0,9 g de
cloreto de sódio. Dessa forma, podemos calcular a concentração comum dessa solução:
𝑚 0,9
𝐶= = = 9 𝑔/𝐿
𝑉 0,100
Podemos converter a concentração comum em molar dividindo-se pela massa molar do soluto.
𝑀𝑁𝑎𝐶𝑙 = 23 + 35,5 = 58,5𝑔/𝑚𝑜𝑙
Por fim, basta calcular
𝐶 9
[𝑁𝑎𝐶𝑙] = = ≅ 0,154 𝑚𝑜𝑙/𝐿
𝑀 58,5
Gabarito: C

(Estratégia Militares – TFC – Inédita)


No Brasil, existem duas formas de medir o teor de etanol em uma solução com a água: os graus INPM,
que correspondem ao teor em massa, e os graus Gay-Lussac, que correspondem ao teor em volume.
O álcool gel 70% em volume (ou 70 °GL) é bastante utilizado para a assepsia de partes do corpo e de
objetos para a prevenção de doenças.
Considerando que a densidade do etanol é igual a 0,8 g/cm³ e a densidade da água é igual a 1,0 g/cm³,
o teor desse mesmo álcool em graus INPM é:

AULA 10 – SOLUÇÕES 91
Prof. Thiago Cardoso

a) 65%
b) 67%
c) 70%
d) 73%
e) 75%

Comentários
Uma amostra de 1 litro dessa solução terá 700 mL de etanol e 300 mL de água. Considerando as
densidades das duas substâncias, podemos calcular as massas de etanol e de água presentes na mistura.
𝑚
𝑑 = ∴ 𝑚 = 𝑑𝑉
𝑉
Agora, apliquemos a expressão acima.
𝑚𝐶2 𝐻6𝑂 = 𝑑𝐶2 𝐻6𝑂 . 𝑉𝐶2𝐻6𝑂 = 0,8.700 = 560 𝑔
𝑚𝐻2𝑂 = 𝑑𝐻2𝑂 . 𝑉𝐻2𝑂 = 1,0.300 = 300 𝑔
Dessa forma, a massa total de 1 litro da solução será:
𝑚 = 𝑚𝐶2𝐻6 𝑂 + 𝑚𝐻2 𝑂 = 300 + 560 = 860 𝑔
Por fim, o título em massa do etanol corresponde à razão entre a massa de etanol e a massa
total da solução:
560
𝜏= ≅ 0,63 = 65%
860
Gabarito: A

(Estratégia Militares – TFC – Inédita)


Em uma aula prática de química, o professor forneceu a um grupo de alunos 100 mL de uma solução 1
mol.L–1 de sacarose. Ele deseja que os alunos produzissem uma solução 0,2 mol.L –1 da substância por
meio da diluição dessa solução concentrada em água destilada.
Para isso, os alunos devem adicionar um volume de água igual a:
a) 40 mL
b) 90 mL
c) 400 mL
d) 900 mL
e) 1400 mL

Comentários
Vamos utilizar a Lei da Diluição.
𝑀1 𝑉1 = 𝑀0 𝑉0
Substituindo os valores fornecidos no enunciado:

AULA 10 – SOLUÇÕES 92
Prof. Thiago Cardoso

0,20. 𝑉1 = 1.100
100
∴ 𝑉1 = = 500 𝑚𝐿
0,20
Dessa forma, o volume a ser completado na solução é equivalente à diferença entre o volume
final e o volume inicial:
Δ𝑉 = 500 − 100 = 400 𝑚𝐿
Gabarito: C

(Estratégia Militares – TFC – Inédita)


A osmose reversa é uma técnica muito utilizado no tratamento de água, que consiste em fazer passar a
água ainda contaminada com sais por uma membrana semipermeável. No interior da membrana, será
deixada uma pequena quantidade de água destilada.
A tendência natural da osmose seria que as moléculas de água atravessassem a membrana indo do
meio hipotônico para o meio hipertônico. Porém, quando se aplica uma pressão sobre a membrana,
haverá um fluxo no sentido inverso, de modo que a água seguirá para o meio hipotônico, aumentando
a quantidade de água destilada.
O soro fisiológico é uma solução é uma solução 0,15 mol/L de cloreto de sódio (NaC). Assinale a
alternativa que apresenta a pressão necessária para realizar a osmose reversa nessa solução a 27 °C:
a) 3,7 atm
b) 4,5 atm
c) 5,2 atm
d) 6,5 atm
e) 7,4 atm

Comentários
A pressão osmótica de uma solução é dada pela expressão:
𝑃 = 𝑀𝑅𝑇
Devemos notar, porém, que o cloreto de sódio se dissocia, de acordo com a seguinte equação:
𝑁𝑎𝐶𝑙 (𝑠) → 𝑁𝑎+ (𝑎𝑞) + 𝐶𝑙 − (𝑎𝑞)
Dessa forma, a concentração molar da solução é igual a 0,30 mol/L em íons, tendo em vista que
são 0,15 mol/L de íons Na+ e 0,15 mol/L de íons C–.
𝑃 = 𝑀𝑅𝑇 = 2.0,15.0,082.300 = 7,38 𝑎𝑡𝑚
Gabarito: E

AULA 10 – SOLUÇÕES 93
Prof. Thiago Cardoso

(Estratégia Militares – TFC – Inédita)


A água de um aquário marinho pode ser considerada como uma solução 35 gramas de diversos sais,
cuja massa molar média é igual a 70 g/mol e fator de van’t Hoff médio igual a 2,2, em 900 mL de água
destilada.
Sabendo que a água pura tem pressão de vapor à temperatura ambiente igual a 25 mmHg, assinale a
alternativa que apresenta a pressão de vapor da água do aquário marinho:
a) 23,0 mmHg
b) 23,5 mmHg
c) 24,0 mmHg
d) 24,5 mmHg
e) 25,0 mmHg

Comentários
O primeiro ponto é calcular o número de mols presentes na solução de 900 mL.
𝑚1 35
𝑛1 = = = 0,5 𝑚𝑜𝑙
𝑀1 70
Agora, devemos considerar o fator de van’t Hoff do sal. Um fator de van’t Hoff igual a 2,2 significa
que cada mol de fórmulas do sal produzir 2,2 mols de íons em solução aquosa.
𝑛1 = 0,5.2,2 = 1,1 𝑚𝑜𝑙
O próximo passo é calcular o número de mols do solvente presentes na solução. Para isso,
seguimos o mesmo procedimento: o número de mols é igual à massa presente na solução dividida pela
massa molar.
𝑀𝐻2𝑂 = 2.1 + 1.16 = 18 𝑔/𝑚𝑜𝑙
900
𝑛2 = = 50 𝑚𝑜𝑙
18
Pela Lei de Raoult, o abaixamento relativo é igual à fração molar do soluto:
Δ𝑃
= 𝑥1
𝑃0
Δ𝑃 𝑛1 1,1 1,1
= 𝑥1 = = =
𝑃0 𝑛1 + 𝑛2 50 + 1,1 51,1
Portanto, o abaixamento da pressão de vapor da solução é:
1,1 1,1
∴ Δ𝑃 = ⋅ 𝑃0 = ⋅ 25 = 0,43 𝑚𝑚𝐻𝑔
51,1 51,1
Dessa forma, a pressão de vapor da solução é:
𝑃 = 𝑃0 − 𝛥𝑃 = 25 − 0,43 = 24,57 𝑚𝑚𝐻𝑔
Gabarito: D

AULA 10 – SOLUÇÕES 94
Prof. Thiago Cardoso

(Estratégia Militares – TFC – Inédita)


Sabendo que o coeficiente de solubilidade do cloreto de sódio é de 36 g/100 g de água a 20 °C, assinale
a alternativa que apresenta a temperatura de ebulição de uma solução saturada desse sal.
Determine a temperatura de ebulição dessa solução:
Dado: Constante Ebuliscópica da Água = 0,52 °C.molal–1
a) 100 °C
b) 101,8 °C
c) 103,2 °C
d) 105 °C
e) 106,4 °C

Comentários
Primeiramente, vamos calcular a molalidade da solução. O primeiro passo é obter o número de
mols de NaCl.
𝑚1 36
𝑛1 = = ≅ 0,615 𝑚𝑜𝑙
𝑀1 58,5
Agora, calculemos a molalidade:
𝑛1 0,615
𝑊= = = 6,15 𝑚𝑜𝑙𝑎𝑙
𝑚2 0,100
Por fim, devemos nos lembrar que o cloreto de sódio se dissocia liberando 2 íons em solução
aquosa, de acordo com a seguinte equação:
𝑁𝑎𝐶𝑙 (𝑠) → 𝑁𝑎+ (𝑎𝑞) + 𝐶𝑙 − (𝑎𝑞)
Portanto, o fator de van’t Hoff associado ao cloreto de sódio é igual a 2. Logo, a temperatura de
ebulição da solução dada pela equação do Efeito Ebulioscópico é:
Δ𝑇𝐸 = 𝐾𝐸 . 𝑊. 𝑖 = 0,52.6,15.2 = 6,4 °𝐶
Gabarito: E

(Estratégia Militares – TFC – Inédita)


A água oxigenada é uma solução aquosa do peróxido de hidrogênio (H2O2), que possui várias aplicações
práticas, como antisséptico e também como agente branqueador para tecidos e cabelos.
Uma solução aquosa de água oxigenada é conhecida como 18 volumes, porque 1 litro dessa solução é
capaz de produzir 18 litros de gás oxigênio, por meio da seguinte reação:
H2O2(aq) → H2O (l) + 1/2 O2 (g)
Sabendo disso, é possível afirmar que a concentração em massa dessa solução é aproximadamente:
Dado: Volume molar nas CNTP = 22,4 L.mol–1
a) 27 g/L
b) 36 g/L

AULA 10 – SOLUÇÕES 95
Prof. Thiago Cardoso

c) 54 g/L
d) 72 g/L
e) 90 g/L

Comentários
Primeiramente, vamos calcular o número de mols de oxigênio que foram produzidos a partir de 1
litro de água destilada. Podemos utilizar o volume molar:
𝑉 𝑂2 18
𝑛𝑂2 = = ≅ 0,8 𝑚𝑜𝑙
𝑉𝑀 22,4
Agora, vamos escrever a equação de decomposição do peróxido de hidrogênio.
1
𝐻2 𝑂2 (𝑎𝑞) → 𝐻2 𝑂 (𝑙) + 𝑂2 (𝑔)
2
Podemos calcular o número de mols de peróxido de hidrogênio presentes na solução pela
proporção estequiométrica.
𝑛𝐻2𝑂2 𝑛𝑂
= 2
1 1/2
∴ 𝑛𝐻2 𝑂2 = 2. 𝑛𝑂2 = 2.0,8 = 1,6 𝑚𝑜𝑙
Assim, em 1 litro de solução, temos 1,6 mol de H2O2. Como a questão pediu a concentração em
grama por litro, basta multiplicar pela massa molar.
𝑀𝐻2𝑂2 = 2.1 + 2.16 = 34 𝑔/𝑚𝑜𝑙
𝐶 = [𝐻2 𝑂2 ]. 𝑀𝐻2 𝑂2 = 1,6.34 = 54,4 𝑔/𝐿
Gabarito: C

(Estratégia Militares – TFC – Inédita)


A pressão de vapor da água pura a 25 °C é igual a 24 mmHg. São dissolvidos 11,7 gramas de cloreto de
sódio (NaC) em 900 mL de água. Assinale a opção que apresenta o valor aproximado do abaixamento
da pressão de vapor da solução:
a) 0,10 mmHg
b) 0,19 mmHg.
c) 0,29 mmHg
d) 0,38 mmHg
e) 0,54 mmHg

Comentários

Vamos usar a lei de Raoult para resolver a questão:

Δ𝑃 = 𝑋𝑠𝑜𝑙𝑢𝑡𝑜 ⋅ 𝑃°𝑆𝑜𝑙𝑣𝑒𝑛𝑡𝑒

AULA 10 – SOLUÇÕES 96
Prof. Thiago Cardoso

Para calcular a fração molar do soluto, devemos considerar o fator de Van’t Hoff, o qual contabiliza
o número efetivo de partículas na solução (lembre-se que as propriedades coligativas são propriedades
diretamente relacionadas com o número de partículas do soluto).

Daí, perceba que o nosso i é igual a 2, visto que o NaCse dissocia completamente em Na+ e C-,
de acordo com a seguinte reação de dissolução.

𝑁𝑎𝐶𝑙(𝑠) → 𝑁𝑎+ (𝑎𝑞) + 𝐶𝑙 − (𝑎𝑞)

Podemos calcular as massas molares da água (H2O) e do cloreto de sódio (NaC). Para isso, basta
somar as massas dos elementos químicos, que foram disponibilizados na tabela periódica.

𝑀𝐻2 𝑂 = 2.1 + 1.16 = 18 𝑔/𝑚𝑜𝑙

𝑀𝑁𝑎𝐶𝑙 = 1.23 + 1.35,5 = 58,5 𝑔/𝑚𝑜𝑙

Admitindo a densidade da água 1 g/mL, a massa corresponde a 900 mL é igual a 900 g. Dessa
forma, o número de mols de água e de NaCl presentes em solução podem ser obtidos como a razão entre
a massa da amostra e a massa molar.

900
𝑛𝐻2 𝑂 = = 50 𝑚𝑜𝑙𝑠 𝑑𝑒 𝐻2 𝑂
18
11,7
𝑛𝑁𝑎𝐶𝑙 = = 0,2 𝑚𝑜𝑙𝑠 𝑑𝑒 𝑁𝑎𝐶𝑙
58,5

Portanto, o abaixamento da pressão de vapor, calculado pela fórmula mostrada no início da


questão é:

𝑛𝑁𝑎𝐶𝑙 0,4 0,4


Δ𝑃 = ⋅ 𝑃°𝑆𝑜𝑙𝑣𝑒𝑛𝑡𝑒 = ⋅ 24 = ⋅ 48 ≅ 0,19 𝑚𝑚𝐻𝑔
𝑛𝑁𝑎𝐶𝑙 + 𝑛𝐻2 𝑂 50 + 0,4 50,4

Gabarito: B

(Estratégia Militares – TFC – Inédita)


A água oxigenada é uma solução aquosa de peróxido de hidrogênio que é bastante utilizada como
antisséptico e desinfectante.
Sabendo que o peróxido de hidrogênio se decompõe por meio da seguinte reação não balanceada:
H2O2 (aq) → H2O () + O2 (g)
A concentração de uma solução de água oxigenada é igual a n volumes, porque o volume de gás liberado
é igual a n vezes o volume inicial da solução.
Dessa forma, a concentração em massa de peróxido de hidrogênio correspondente a uma amostra de
água oxigenada 9,76 volumes é igual a:
Dado: Volume Molar = 24,4 L.mol–1.
a) 13,6 g/L.
b) 20,4 g/L
c) 27,2 g/L

AULA 10 – SOLUÇÕES 97
Prof. Thiago Cardoso

d) 34,0 g/L
e) 54,4 g/L

Comentários

Primeiramente, vamos balancear a reação.

2 𝐻2 𝑂2 (𝑎𝑞) → 2 𝐻2 𝑂 (𝑙) + 𝑂2 (𝑔)

Considerando 1 litro de solução, com as informações fornecidas no enunciado, essa mesma


solução produzirá 9,76 litros de oxigênio. Podemos calcular o número de mols de oxigênio como a razão
entre o volume produzido e o volume molar.

𝑉𝑂2 9,76
𝑛𝑂2 = = = 0,4 𝑚𝑜𝑙
𝑉𝑀 24,4

Agora, podemos calcular o número de mols de peróxido de hidrogênio pela proporção


estequiométrica.
𝑛𝐻2 𝑂2 𝑛𝑂2
= ∴ 𝑛𝐻2 𝑂2 = 2. 𝑛𝑂2 = 2.0,4 = 0,8 𝑚𝑜𝑙
2 1
Dessa forma, encontram-se 0,8 mol em 1 litro de solução de água oxigenada. Podemos calcular a
massa de peróxido de hidrogênio.

𝑀𝐻2 𝑂2 = 2.1 + 2.16 = 2 + 32 = 34 𝑔/𝑚𝑜𝑙

𝑚𝐻2 𝑂2 = 0,8.34 = 27,2 𝑔

Dessa forma, a solução tem 27,2 g por litro de solução.

Gabarito: C

(Estratégia Militares – TFC – Inédita)


Uma salmoura foi preparada com a dissolução de 5,85 gramas de cloreto de sódio em 100 mL de água.
Assinale a alternativa que indica o aumento da temperatura de ebulição do solvente provocado pela
dissolução do soluto.
Dado: Constante Ebulioscópica da Água = 0,5 °C. kg. mol –1
a) 0,25 °C
b) 0,5 °C
c) 1,0 °C
d) 1,5 °C
e) 2,5 °C

AULA 10 – SOLUÇÕES 98
Prof. Thiago Cardoso

Comentários

Primeiramente, vamos calcular o número de mols de NaCl.

𝑀𝑁𝑎𝐶𝑙 = 1.23 + 1.35,5 = 58,5 𝑔/𝑚𝑜𝑙

O número de mols pode ser obtido dividindo-se a massa da amostra pela massa molar.

𝑚𝑁𝑎𝐶𝑙 5,85
𝑛𝑁𝑎𝐶𝑙 = = = 0,1 𝑚𝑜𝑙
𝑀𝑁𝑎𝐶𝑙 58,5

A concentração molal pode ser obtida como a razão entre o número de mols do soluto e a massa
do solvente.

𝑛1 0,1
𝑊= = = 1 𝑚𝑜𝑙
𝑚2 0,1

Como o cloreto se dissocia em dois íons, é preciso levar em conta o fator de Van’t Hoff para o sal.

𝑁𝑎𝐶𝑙 (𝑠) → 𝑁𝑎+ (𝑎𝑞) + 𝐶𝑙 − (𝑎𝑞)

𝑖=2

Dessa forma, o efeito ebulioscópico se torna:

Δ𝑇𝐸 = 𝐾𝐸 . 𝑊. 𝑖

Δ𝑇𝐸 = (0,5). 1.2 = 1 °𝐶

Gabarito: C

(Estratégia Militares – TFC – Inédita)


Um professor de Química estava desconfiado de que sua aliança de casamento era falsificada. Segundo
o vendedor, a joia era constituída de uma mistura de 77,2% de ouro em massa e 22,8% de uma liga
metálica, formada principalmente por cobre e níquel.
Porém, devido à sua desconfiança, o professor resolveu medir a densidade da aliança. Para isso, ele
utilizou uma balança de precisão e registrou que a sua massa era de aproximadamente 10g. A seguir,
ele mediu o volume, imergindo a aliança em um béquer graduado.
Sabendo que a densidade do ouro é igual a 19,3 g/cm³ e que a densidade da liga metálica é igual a 7,6
g/cm³, o professor atestou que sua aliança era legítima. Dessa forma, o volume da aliança medido pelo
professor é aproximadamente igual a:
a) 0,4 mL
b) 0,5 mL
c) 0,6 mL
d) 0,7 mL
e) 0,8 mL

AULA 10 – SOLUÇÕES 99
Prof. Thiago Cardoso

Comentários

Observe que a massa de ouro e da liga metálica podem ser calculadas pelo teor fornecido no
enunciado.

𝑚𝐴𝑢 = 0,772.10 = 7,72 𝑔

𝑚𝑙𝑖𝑔𝑎 = 0,228.10 = 2,28 𝑔

A seguir, o nosso próximo passo é calcular o volume total da aliança. Para isso, devemos considerar
os volumes parciais de ouro e da liga metálica.

7,72 g Au
10 g
2,28 g liga

Os volumes parciais se relacionam com a massa específica dos metais.


𝑚 𝑚
𝜌= ∴𝑉=
𝑉 𝜌

Calculemos os volumes parciais pela razão

𝑚𝐴𝑢 7,72
𝑉𝐴𝑢 = = ≅ 0,4 𝑚𝐿
𝜌𝐴𝑢 19,3

𝑚𝑙𝑖𝑔𝑎 2,28
𝑉𝑙𝑖𝑔𝑎 = = ≅ 0,3 𝑚𝐿
𝜌𝑙𝑖𝑔𝑎 7,6

Portanto, o volume total da aliança é igual a:

𝑉 = 𝑉𝐴𝑢 + 𝑉𝑙𝑖𝑔𝑎 = 0,4 + 0,3 = 0,7 𝑚𝐿

Vamos esquematizar a aliança.

7,72 g Au 0,4 mL
10 g
2,28 g liga 0,3 mL

A densidade da aliança, portanto, será igual a:

AULA 10 – SOLUÇÕES 100


Prof. Thiago Cardoso

𝑚 10
𝜌= = ≅ 14,28
𝑉 0,7
Gabarito: D

(Estratégia Militares – TFC – Inédita)


O titânio é um material muito útil, pois ele pode ser adicionado a ligas metálicas, como o aço, de modo
a lhe conferir propriedades muito interessantes, tornando o material mais leve e resistente.
Uma das propriedades mais interessantes desse material é a sua baixa densidade. Considere que seja
feita uma liga metálica a partir da mistura de 18 g de titânio a um volume de 6 cm³ de aço.

Assinale a alternativa que indica o valor aproximado da densidade da liga metálica formada:
a) 5,0 g/cm³
b) 5,5 g/cm³
c) 6,0 g/cm³
d) 6,5 g/cm³
e) 7,0 g/cm³

Comentários

Vamos nos lembrar sobre a expressão da densidade e obter as expressões da massa e do volume
por meio da densidade.
𝑚 𝑚
𝑑= ∴ 𝑚 = 𝑑𝑉 ∴ 𝑉 =
𝑉 𝑑
Primeiramente, vamos calcular o volume de titânio presentes na liga.

𝑚𝑇𝑖 18
𝑉𝑇𝑖 = = = 4 𝑐𝑚³
𝑑𝑇𝑖 4,5

Como obtivemos o volume de titânio e o volume de aço foi fornecido no enunciado. Dessa forma,
o volume total da liga é:

𝑉 = 𝑉𝑇𝑖 + 𝑉𝐹𝑒 = 4 + 6 = 10 𝑐𝑚³

Podemos, agora, calcular a massa de ferro.

𝑚𝐹𝑒 = 𝑑𝐹𝑒 𝑉𝐹𝑒

𝑚𝐹𝑒 = 7,8.6 = 46,8 𝑔

Dessa forma, a massa total da liga pode ser obtida como a soma das massas de titânio e de aço
presentes na liga:

AULA 10 – SOLUÇÕES 101


Prof. Thiago Cardoso

𝑚 = 𝑚 𝑇𝑖 + 𝑚𝐹𝑒 = 18 + 46,8 = 64,8 𝑔

Por fim, podemos calcular a densidade da liga como a razão entre a sua massa e o seu volume.
Ambos foram calculados anteriormente.

𝑚 64,8
𝑑= = = 6,48 𝑔/𝑐𝑚³
𝑉 10
Gabarito: D

(Estratégia Militares – TFC – Inédita)


A densidade é uma das características mais importantes de uma liga metálica. Quando uma liga
metálica é disposta pela proporção em massas dos metais constituintes, a densidade da liga metálica
pode ser calculada como sendo aproximadamente igual à média harmônica ponderada das densidades
dos dois metais.
Dentre os metais, o ouro se destaca por apresentar uma densidade muito superior aos demais. Como
são poucos os materiais que apresentam uma densidade superior à do metal, é muito difícil fraudá-la.
Porém, um ladrão desejava falsificar uma barra de ouro e, para isso, projetou uma liga metálica formada
por platina e cobre, que devia ter a proporção exata dos dois elementos para imitar a densidade do
ouro.

Assinale a alternativa que indica o teor de platina que deve ser utilizado na liga para fraudar a densidade
do ouro:
a) 62,5%
b) 75%
c) 80,0%
d) 87,5%
e) 92,5%

Comentários

Preste atenção na dica fornecida pelo enunciado: a densidade de uma liga metálica pode ser
calculada como a média harmônica ponderada das densidades dos metais presentes na liga. Sendo a e b
as proporções respectivas de platina e cobre presentes na liga, temos:

𝑎 𝑏 1
+ =
21 9 18
Podemos multiplicar a equação acima por 63.

(𝐼) 3𝑎 + 7𝑏 = 3,5

AULA 10 – SOLUÇÕES 102


Prof. Thiago Cardoso

Além disso, a soma das proporções em massa de cada metal presente da liga é igual a 1 (ou 100%).

(𝐼𝐼) 𝑎 + 𝑏 = 1

Podemos multiplicar a segunda equação por 3.

(𝐼) 3𝑎 + 7𝑏 = 3,5
(𝐼𝐼) 3𝑎 + 3𝑏 = 3

Podemos obter a diferença (I) – (II).

7𝑏 − 3𝑏 = 3,5 − 3

Fazendo as contas, temos:

4𝑏 = 0,5

0,5
∴𝑏= = 0,125 = 12,5%
4
Dessa forma, a proporção de cobre na liga é igual a 12,5%. Logo, a proporção em massa da platina
é a = 87,5%.

Gabarito: D

(Estratégia Militares – TFC – Inédita)


Na Química, é muito importante conhecer os principais fatores limitantes da reação. Entre eles, está a
quantidade dos reagentes que foram adicionados.
Como os reagentes somente podem reagir na proporção estequiométrica, se um dos reagentes for
colocado em uma proporção inferior, ele será completamente consumido durante a reação e ela vai
cessar.
Um químico promoveu a dissolução de uma rocha de 4 gramas de carbonato de cálcio. Para isso, ele
utilizou 40 mL de uma solução 1,0 mol/L de ácido clorídrico. Considerando as limitações de reagentes
na reação e que o volume molar dos gases nas condições da reação é igual a 22,4 L, assinale a alternativa
que indica o volume de gás liberado.
Dado: CaCO3 (s) + 2 HCl → CaCl2 (aq) + H2O () + CO2 (g)
a) 300 mL
b) 450 mL
c) 600 mL
d) 750 mL
e) 900 mL

AULA 10 – SOLUÇÕES 103


Prof. Thiago Cardoso

Comentários

Vamos calcular o número de mols de carbonato. Para isso, devemos dividir a massa da amostra
pela massa molar do sal.

𝑀𝐶𝑎𝐶𝑂3 = 1.40 + 1.12 + 3.16 = 40 + 12 + 48 = 100 𝑔/𝑚𝑜𝑙

Já o número de mols de carbonato de cálcio pode ser obtido pela relação entre a massa da amostra
e a massa molar.

𝑚𝐶𝑎𝐶𝑂3 4
𝑛𝐶𝑎𝐶𝑂3 = = = 0,04 𝑚𝑜𝑙
𝑀𝐶𝑎𝐶𝑂3 100

Por sua vez, o número de mols de HCl pode ser obtido como o produto:

𝑛𝐻𝐶𝑙 = [𝐻𝐶𝑙]. 𝑉𝐻𝐶𝑙 = 1.0,04 = 0,04 𝑚𝑜𝑙

Como a reação ocorre na proporção 1:2, temos que o HCl é o reagente limitante. Vejamos na
tabela estequiométrica.

𝑪𝒂𝑪𝑶𝟑 (𝒔) + 𝟐 𝑯𝑪𝒍 (𝒂𝒒) → 𝑪𝒂𝑪𝒍𝟐 (𝒂𝒒) + 𝑯𝟐 𝑶 + 𝑪𝑶𝟐

início 0,04 𝑚𝑜𝑙 0,04 𝑚𝑜𝑙 − − −

reage 0,02 𝑚𝑜𝑙 0,04 𝑚𝑜𝑙 0,02 𝑚𝑜𝑙 0,02 𝑚𝑜𝑙 0,02 𝑚𝑜𝑙

final 0,02 𝑚𝑜𝑙 0 𝑚𝑜𝑙 0,02 𝑚𝑜𝑙 0,02 𝑚𝑜𝑙 0,02 𝑚𝑜𝑙

Dessa forma, são produzidos 0,02 mol de CO2. Como a questão informou o volume molar, temos:

𝑉𝐶𝑂2 = 0,02.22,5 = 0,45 𝐿 = 450 𝑚𝐿

Gabarito: B

(Estratégia Militares – TFC – Inédita)


Para a construção de um sal sintético a ser utilizado em aquário marinho, um fabricante, entre outros,
alguns compostos químicos. Supondo a dissolução de 39 gramas do sal em 1 litro de água destilada, a
água apresentaria a seguinte composição:
COMPOSIÇÃO QUÍMICA PROVÁVEL (percentual em massa): cloreto de sódio = 27%; cloreto de magnésio
= 14,2%; sulfato de potássio = 3,6%; cloreto de cálcio = 3,1%; bicarbonato de sódio = 0,4%.
CARACTERÍSTICAS FÍSICO-QUÍMICAS: pH medido a 25 °C = 8,3; temperatura da água = 25 °C; índice de
refração a 25 °C = 1,025;
Considerando os dados da questão anterior e que o cloreto de magnésio seja a única fonte de magnésio
presente no sal e que a água do aquário tenha sido preparada com 1 litro de água destilada para 39
gramas do sal, podemos concluir que a concentração final de íons magnésio em mg/L nessa água será:
a) 1200
b) 1250

AULA 10 – SOLUÇÕES 104


Prof. Thiago Cardoso

c) 1300
d) 1350
e) 1400

Comentários

Vamos calcular a massa de magnésio que pode ser encontrada em 1 litro de água. Primeiramente,
vamos obter a massa de cloreto de magnésio, que pode ser obtida a partir do percentual em massa desse
composto na formulação inicial do sal.

𝑚𝑀𝑔𝐶𝑙2 = 0,142.39 = 5,538 𝑔 = 5538 𝑚𝑔

Podemos, agora, calcular a massa de magnésio utilizando o teor do elemento em MgCl2. O teor do
elemento químico pode ser obtido como a razão entre a massa de magnésio presente na fórmula e a
massa total da fórmula.

1.24 24 24
(%𝑀𝑔) = = =
1.24 + 2.35,5 24 + 71 95

Dessa forma, a massa de magnésio encontrada em 1 litro de água é:

24
𝑚𝑀𝑔 = (%𝑀𝑔). 𝑚𝑀𝑔𝐶𝑙2 = ⋅ 5538 ≅ 1400 𝑚𝑔
95
Portanto, encontramos 1400 mg de magnésio em 1 L de água.

Gabarito: E

(Estratégia Militares – TFC – Inédita)


No Brasil, existem duas unidades de medida para a concentração do álcool. Ele pode ser medida em
percentual de massa (°INPM) ou em percentual de volume (°GL).
O álcool gel utilizado para a assepsia apresenta a concentração em volume igual a 70% (ou seja, 70 °GL).
No entanto, é muito comum encontrá-lo também em embalagens que informam o percentual em
massa.

a) 75%
b) 72%
c) 70%
d) 67%
e) 65%

AULA 10 – SOLUÇÕES 105


Prof. Thiago Cardoso

Comentários
Considerando 1 litro de uma mistura de álcool e água com 70% de volume em álcool, temos que
essa amostra contém 700 mL de etanol e 300 mL de água.
As massas correspondentes podem ser calculadas pela densidade.
𝑚
𝑑 = ∴ 𝑚 = 𝑑𝑉
𝑉
Agora, calculemos as massas de etanol e de água presentes na mistura.
𝑚𝑒𝑡𝑎𝑛𝑜𝑙 = 𝑑𝑒𝑡𝑎𝑛𝑜𝑙 . 𝑉𝑒𝑡𝑎𝑛𝑜𝑙 = 0,8.700 = 560 𝑔
𝑚𝐻2 𝑂 = 𝑑𝐻2 𝑂 . 𝑉𝐻2𝑂 = 1,0.300 = 300 𝑔
Logo, a massa total de 1 litro de álcool 70% será a soma das massas de etanol e de água presentes
na mistura.
𝑚 = 𝑚𝑒𝑡𝑎𝑛𝑜𝑙 + 𝑚𝐻2 𝑂 = 560 + 300 = 860 𝑔
Por fim, o título em massa de etanol pode ser obtido como a razão entre a massa de etanol e a
massa total da mistura.
𝑚𝑒𝑡𝑎𝑛𝑜𝑙 560
𝜏= = ≅ 0,65 = 65%
𝑚 860
Gabarito: E

(Estratégia Militares – TFC – Inédita)


O balling é uma das técnicas mais importantes de manutenção dos níveis adequados de parâmetros,
como o cálcio em um aquário marinho. Os corais consomem continuamente o cálcio fornecido pelo
balling para a construção dos seus esqueletos de carbonato de cálcio.
Um aquário consome 11,1 mL por dia de uma solução aquosa 20 g/L de cloreto de cálcio. Supondo que
80% do cálcio consumido no aquário seja pelos corais para a construção do seu esqueleto calcário,
determine a quantidade de esqueleto construída em um dia nesse aquário:
a) 80 mg
b) 120 mg
c) 160 mg
d) 200 mg
e) 240 mg

Comentários

Questão muito interessante. Vamos analisar as informações. Para calcular a quantidade de cálcio,
temos que achar a quantidade de cloreto consumida por dia. Daí, vem:
𝑚
𝐶=
𝑉

AULA 10 – SOLUÇÕES 106


Prof. Thiago Cardoso

𝑚
20 =
11,1 ⋅ 10−3

𝑚 = 20 ⋅ 11,1 ⋅ 10−3 = 0,222 𝑔 𝑑𝑒 𝐶𝑎𝐶𝑙2

Contudo, CaC2: 111 g/mol e Ca: 40 g/mol. Logo, podemos calcular a porcentagem de cálcio no
cloreto. Temos:

𝑀𝐶𝑎 40
%= =
𝑀𝐶𝑎𝐶𝑙2 111

Daí:

40 𝑚𝐶𝑎
=
111 𝑚
40 𝑚𝐶𝑎
=
111 0,222
40
𝑚𝐶𝑎 = 0,222 ⋅ = 0,08 𝑔 𝑑𝑒 𝐶𝑎
111
Dessa massa de cálcio, 80% foi, efetivamente, convertida em esqueleto calcário. Logo:

𝑚𝑟𝑒𝑎𝑔𝑒𝑛𝑡𝑒 = 0,8 ⋅ 0,08 = 0,064 𝑔

Perceba que todo esse cálcio reagente aparecerá na forma de CaCO3 a fim de formar o esqueleto.
Logo, podemos calcular a porcentagem de cálcio no carbonato (CaCO3: 100 g/mol):

𝑀𝐶𝑎 40
= = 0,4
𝑀𝐶𝑎𝐶𝑂3 100

Logo, a massa de esqueleto formada em um dia é:


𝑚𝑟𝑒𝑎𝑔𝑒𝑛𝑡𝑒
= 0,4
𝑚𝑒𝑠𝑞𝑢𝑒𝑙𝑒𝑡𝑜

𝑚𝑟𝑒𝑎𝑔𝑒𝑛𝑡𝑒 0,064
𝑚𝑒𝑠𝑞𝑢𝑒𝑙𝑒𝑡𝑜 = = = 0,16 𝑔 = 160 𝑚𝑔
0,4 0,4
Gabarito: C

AULA 10 – SOLUÇÕES 107


Prof. Thiago Cardoso

(Estratégia Militares – TFC – Inédita)


Um objeto de 2800 cm³ foi construído com 75% de ouro em massa e 25% de uma liga metálica. Assinale
a alternativa que indica a massa do objeto.
Dados: densidade do ouro = 20 g/cm³ e densidade da liga metálica = 5 g/cm³
a) 32,0 kg
b) 36,0 kg
c) 38,5 kg
d) 42,0 kg
e) 45,5 kg

Comentários

Considere uma peça de 4g com a mesma constituição do objeto. Essa peça terá 3 g de ouro e 1 g
da liga metálica. Podemos calcular o seu volume.

O volume de ouro na peça será igual à massa de ouro dividido pela densidade do metal.

𝑚𝐴𝑢 3
𝑉𝐴𝑢 = = ≅ 0,15 𝑐𝑚³
𝑑𝐴𝑢 20

𝑚𝑙𝑖𝑔𝑎 1
𝑉𝑙𝑖𝑔𝑎 = = ≅ 0,2 𝑐𝑚³
𝑑𝑙𝑖𝑔𝑎 5

Portanto, o volume total de 4 gramas da peça é igual a:

V4 = 𝑉𝐴𝑢 + 𝑉𝑙𝑖𝑔𝑎 = 0,15 + 0,2 = 0,35 𝑐𝑚³

Podemos calcular a massa da peça de 2800 cm³ por uma Regra de Três.

𝑚𝑝𝑒ç𝑎 4𝑔
=
2800 0,35
2800 2800
∴ 𝑚𝑝𝑒ç𝑎 = ⋅4= ⋅ 4 = 32000 𝑔 = 32 𝑘𝑔
0,35 0,35
Gabarito: D

AULA 10 – SOLUÇÕES 108


Prof. Thiago Cardoso

(Estratégia Militares – TFC – Inédita)


Um objeto de 2800 cm³ foi construído com 75% de ouro em massa e 25% de uma liga metálica. Assinale
a alternativa que indica a massa do objeto.
Dados: densidade do ouro = 20 g/cm³ e densidade da liga metálica = 5 g/cm³
a) 32,0 kg
b) 36,0 kg
c) 38,5 kg
d) 42,0 kg
e) 45,5 kg

Comentários

Considere uma peça de 4g com a mesma constituição do objeto. Essa peça terá 3 g de ouro e 1 g
da liga metálica. Podemos calcular o seu volume.

O volume de ouro na peça será igual à massa de ouro dividido pela densidade do metal.

𝑚𝐴𝑢 3
𝑉𝐴𝑢 = = ≅ 0,15 𝑐𝑚³
𝑑𝐴𝑢 20

𝑚𝑙𝑖𝑔𝑎 1
𝑉𝑙𝑖𝑔𝑎 = = ≅ 0,2 𝑐𝑚³
𝑑𝑙𝑖𝑔𝑎 5

Portanto, o volume total de 4 gramas da peça é igual a:

V4 = 𝑉𝐴𝑢 + 𝑉𝑙𝑖𝑔𝑎 = 0,15 + 0,2 = 0,35 𝑐𝑚³

Podemos calcular a massa da peça de 2800 cm³ por uma Regra de Três.

𝑚𝑝𝑒ç𝑎 4𝑔
=
2800 0,35
2800 2800
∴ 𝑚𝑝𝑒ç𝑎 = ⋅4= ⋅ 4 = 32000 𝑔 = 32 𝑘𝑔
0,35 0,35
Gabarito: D

AULA 10 – SOLUÇÕES 109


Prof. Thiago Cardoso

(Estratégia Militares – TFC – Inédita)


Um aquário marinho apresenta 0,018 mg/L do elemento fósforo. Considerando que todo o fósforo
esteja na forma de íons fosfato, determine a concentração do íon fosfato (em mg/L) nesse aquário.
Dados: Massas Molares: P = 31 g/mol, O = 16 g/mol.
a) 0,018
b) 0,025
c) 0,036
d) 0,055
e) 0,078

Comentários

Considerando que a fórmula do íon fosfato é 𝑃𝑂3−


4 , temos que a sua massa molar é:

𝑀 = 1.31 + 4.16 = 95 𝑔/𝑚𝑜𝑙

Considerando que todo o fósforo esteja presente na forma de fosfato, a concentração de fosfato
presente na solução é diretamente proporcional à sua massa molar.

[𝑃𝑂43− ] [𝑃]
=
95 31
[𝑃𝑂43− ] 0,018
=
95 31
95.0,018
∴ [𝑃𝑂43− ] = ≅ 0,055
31
Gabarito: D

(Estratégia Militares – TFC – Inédita)


Um aquário marinho apresenta 0,018 mg/L do elemento fósforo. Considerando que todo o fósforo
esteja na forma de íons fosfato, determine a concentração do íon fosfato (em mg/L) nesse aquário.
Dados: Massas Molares: P = 31 g/mol, O = 16 g/mol.
a) 0,018
b) 0,025
c) 0,036
d) 0,055
e) 0,078

AULA 10 – SOLUÇÕES 110


Prof. Thiago Cardoso

Comentários

Considerando que a fórmula do íon fosfato é 𝑃𝑂3−


4 , temos que a sua massa molar é:

𝑀 = 1.31 + 4.16 = 95 𝑔/𝑚𝑜𝑙

Considerando que todo o fósforo esteja presente na forma de fosfato, a concentração de fosfato
presente na solução é diretamente proporcional à sua massa molar.

[𝑃𝑂43− ] [𝑃]
=
95 31
[𝑃𝑂43− ] 0,018
=
95 31
95.0,018
∴ [𝑃𝑂43− ] = ≅ 0,055
31
Gabarito: D

(Estratégia Militares – TFC – Inédita)


Os ventos solares são o resultado da emissão contínua de partículas carregadas provenientes da coroa
solar. Exemplos desses efeitos são as caudas de cometas, que têm a sua orientação definida pela direção
do vento solar. Na superfície da Terra, os ventos solares carregam aproximadamente 500 milhões de
átomos de hidrogênio por cm² por segundo. Considerando que esse fluxo seja mantido constante e que
a área da cidade de Campinas seja igual a 800 km², determine a massa aproximada de hidrogênio que
incidirá sobre essa cidade no intervalo de 1 ano:
Dado: NA = 6,0.1023
a) 1 kg
b) 2 kg
c) 3 kg
d) 4 kg
e) 5 kg

Comentários:
O enunciado forneceu a taxa de átomos de hidrogênio que atingem a cidade por unidade de área
e de tempo (500 milhões de átomos por cm² por segundo). Chamemos essa taxa de I. O número total de
átomos de hidrogênio que chegariam à cidade pode ser obrigado como o produto da taxa pela área e pelo
tempo de 1 ano.
Observe que a área foi dada em km². Como a taxa foi dada em cm², podemos converter 1 km = 104
cm, portanto, 1 km = 108 cm².
Para o tempo de 1 ano, podemos convertê-lo em segundos, considerando que 1 ano tem 365 dias.
Cada dia tem 24 horas. Cada hora tem 60 minutos. E cada minuto tem 60 segundos. Portanto, 1 ano seria
igual a 365.24.60.60 segundos.

AULA 10 – SOLUÇÕES 111


Prof. Thiago Cardoso

𝑁 = 𝐼𝐴𝑡 = (500.106 ). (800.108 ). (365.24.60.60)


𝑁 = (5.108 ). (8.1010 ). (365.24.60.60)
Podemos organizar os termos:
𝑁 = 𝐼𝐴𝑡 = (5.8.365.24.60.60). 108+10
𝑁 = 𝐼𝐴𝑡 = (1261440000). 108+10
𝑁 = 𝐼𝐴𝑡 = (1,26.109 ). 108+10
𝑁 = 1,26.109+8+10 = 1,26.1027 = 12,6.1026
Agora, vamos calcular o número de mols de hidrogênio que atingirão a cidade de Campinas.
12,6.1026
𝑛= = 2,1.103 𝑚𝑜𝑙
6.1023
A massa de hidrogênio obtida será igual ao produto do número de mols pela massa molar.
𝑚 = 𝑛. 𝑀 = 2,1.103 . 1 𝑔 = 2,1 𝑘𝑔
Gabarito: B

(Estratégia Militares – TFC – Inédita)


O soro fisiológico é uma solução isotônica de cloreto de sódio a 1% em massa. Considerando que a
densidade da solução é igual a 1,1 g/mL, assinale a alternativa que indica o valor aproximado da
concentração molar dessa solução.
a) 0,09 mol/L.
b) 0,12 mol/L.
c) 0,14 mol/L.
d) 0,19 mol/L.
e) 0,22 mol/L.

Comentários
Vamos calcular a massa de 1 L da solução a partir de sua densidade, notando que 1 L = 1000 cm³.
𝑚
𝑑 = ∴ 𝑚 = 𝑑𝑉 = 1,1.1000 = 1100 𝑔
𝑉
Como a solução tem 1% em massa de cloreto de sódio, podemos obter a massa de cloreto de sódio
presente na solução:
𝑚𝑁𝑎𝐶𝑙 = 0,01. 𝑚 = 0,01.1100 = 11 𝑔
Agora, vamos converter essa massa para número de mols. Para isso, devemos dividir pela massa
molar, que pode ser obtida somando as massas dos elementos químicos.
𝑀𝑁𝑎𝐶𝑙 = 23 + 35,5 = 58,5 𝑔/𝑚𝑜𝑙
𝑚𝑁𝑎𝐶𝑙 11
𝑛𝑁𝑎𝐶𝑙 = = ≅ 0,19 𝑚𝑜𝑙
𝑀𝑁𝑎𝐶𝑙 58,5

AULA 10 – SOLUÇÕES 112


Prof. Thiago Cardoso

Portanto, em 1 litro de solução, encontramos 0,19 mol de cloreto de sódio. Portanto, a


concentração molar é 0,19 mol/L.
Gabarito: D

(Estratégia Militares – TFC – Inédita)


A pressão de vapor da água pura à temperatura ambiente é igual a 25 mmHg. Assinale a alternativa que
apresenta a pressão osmótica de uma solução aquosa 10,1 g/L de nitrato de potássio.
a) 2,4 atm
b) 3,7 atm
c) 4,9 atm
d) 6,1 atm
e) 7,3 atm

Comentários
Vamos calcular o número de mols de nitrato de potássio dissolvidos. Para isso, basta dividir a
concentração em massa pela massa molar.
𝑀𝐾𝑁𝑂3 = 39 + 14 + 3.16 = 101 𝑔/𝑚𝑜𝑙
10,1
[𝐾𝑁𝑂3 ] = = 0,1 𝑚𝑜𝑙
101
Observe que o nitrato de potássio se dissocia em água, liberando dois íons. Logo, haverá 0,2 mol/L
de íons.
Em 1 litro de água, temos 1000 gramas da substância. Podemos calcular o número de mols do
solvente.
𝑚𝐻2 𝑂 1000
𝑛𝐻2 𝑂 = = ≅ 55,56 𝑚𝑜𝑙
𝑀𝐻2 𝑂 18
Dessa forma, a pressão osmótica é dada pela expressão:
𝑃 = 𝑀𝑅𝑇 = 0,2.0,082.298 = 4,89 ≅ 4,9 𝑎𝑡𝑚
Gabarito: C

AULA 10 – SOLUÇÕES 113


Prof. Thiago Cardoso

(Estratégia Militares – TFC – Inédita)


Uma amostra de 1.10–2 g de um determinado composto orgânico é dissolvida em 500 ml de benzeno a
87°C, resultando numa solução de pressão osmótica de 0,082 atm. Determine o composto orgânico:
Dado: densidade do benzeno = 0,9 g/cm³
a) metano
b) butano
c) isopropano
d) hexano
e) neo-pentano

Comentários:
Da expressão da propriedade coligativa, pressão osmótica, temos:
𝜋 = 𝑀. 𝑅. 𝑇. 𝑖
Como o composto é orgânico, temos:
𝑖=1
Assim:
𝑚
→𝜋= . 𝑅. 𝑇
𝑉. 𝑀𝑀
Substituindo os valores, temos:
10−2
→ 0,082 = . 0,082.360 → 𝑀𝑀 = 72 𝑔
0,5. 𝑀𝑀
Por fim, como o composto é um alcano, podemos concluir que sua fórmula molecular é CnH2n+2.
Assim, podemos calcular o valor de n:
𝑛. 12 + (2𝑛 + 2). 1 = 72
14𝑛 + 2 = 72
14𝑛 = 72 − 2 = 70
70
∴𝑛= =5
14
Logo, o composto é o neopentano.
Gabarito: E

AULA 10 – SOLUÇÕES 114


Prof. Thiago Cardoso

(Estratégia Militares – TFC – Inédita)


Sabendo que a água do aquário marinho deve ser preparada com a adição de 40 g de um sal marinho
por litro de água pronta e que o teor ideal de boro ideal na água é igual a 4,4 mg/L, determine o teor
em massa do tetraborato de sódio (Na2B4O7) no sal.
a) 0,05%
b) 0,10%
c) 0,15%
d) 0,20%
e) 0,25%

Comentários
Vamos calcular a massa molar do tetraborato de sódio.
𝑀𝑁𝑎2𝐵4 𝑂7 = 2.23 + 4.11 + 7.16 = 46 + 44 + 112 = 202 𝑔/𝑚𝑜𝑙
A concentração em massa do boro pode ser obtida como o teor de boro em Na2B4O7
[𝑏𝑜𝑟𝑜] = (%𝑏𝑜𝑟𝑜). [𝑁𝑎4 𝐵2 𝑂7 ]
2.11
4,4.10−3 = ( ) . [𝑁𝑎4 𝐵2 𝑂7 ]
202
4,4.10−3
∴ [𝑁𝑎4 𝐵2 𝑂7 ] = ⋅ 202 = 20,2.10−3 𝑔/𝐿
4.11
Concluímos que 1 litro da água pronta terá 20,2.10–3 g do sal. Dessa forma, 20,2.10–3 é a massa de
tetraborato de sódio presente no total de 40 g do sal marinho.
20,2.10−3
%𝑁𝑎4 𝐵2 𝑂7 = = 0,5.10−3 = 0,05.10−2 = 0,05%
40
Gabarito: A

(ITA – 2001)
Um litro de uma solução aquosa contém 0,30 mol de íons Na+, 0,28mol de Cl-, 0,10mol de ions SO42- e x
mol de íons Fe3+. A concentração de íons Fe3 (em mol/L) presentes nesta solução é:
a) 0,03
b) 0,06
c) 0,08
d) 0,18
e) 0,26

Comentários
A solução deve ser eletricamente neutra. Portanto, a soma das cargas positivas deve ser igual à
soma das cargas negativas.

AULA 10 – SOLUÇÕES 115


Prof. Thiago Cardoso

0,30 + 0,28. (−1) + 0,10. (−2) + 𝑥. (3) = 0


0,30 − 0,28 − 0,20 + 3𝑥 = 0
0,30 − 0,48 + 3𝑥 = 0
0,18
3𝑥 = 0,48 − 0,30 = 0,18 ∴ 𝑥 = = 0,06
3

Gabarito: B

(Estratégia Militares – TFC – Inédita)


Uma amostra de 12 mL de uma solução aquosa de cloreto de ferro (II) foi tratada com uma solução
aquosa 0,150 mol/L em permanganato de potássio em meio ácido. Sabendo que o volume utilizado da
solução de permanganato de potássio foi igual a 4 mL, determine a concentração molar da solução
original.
a) 0,05 mol/L
b) 0,10 mol/L
c) 0,15 mol/L
d) 0,20 mol/L
e) 0,25 mol/L

Comentários
Em meio ácido, o manganês presente no permanganato se reduz a Mn2+. Os íons H+ provenientes
do ácido
𝐹𝑒 2+ (𝑎𝑞) + 𝑀𝑛𝑂4− (𝑎𝑞) + 𝐻 + (𝑎𝑞) → 𝐹𝑒 3+ (𝑎𝑞) + 𝑀𝑛2+ (𝑎𝑞) + 𝐻2 𝑂(𝑙)
Podemos deixar em evidência os elementos, cujos números de oxidação sofreram variação.

Agora, vamos usar a técnica de inverter os coeficientes.

Nos já temos uma proporção entre os íons Fe2+ presentes na amostra inicial e os íons MnO4–
presentes na solução titulante. Pela Proporção Estequiométrica, temos:
𝑛𝐹𝑒 2+ 𝑛𝑀𝑛𝑂4−
=
5 1

AULA 10 – SOLUÇÕES 116


Prof. Thiago Cardoso

O número de mols de íons Fe2+ pode ser obtido como o produto da concentração molar pelo
volume da solução. O mesmo válido para o permanganato.
[𝐹𝑒 2+ ]. 𝑉 [𝑀𝑛𝑂4− ]. 𝑉𝑀𝑛𝑂4−
=
5 1
Substituindo os valores fornecidos no enunciado, temos:
[𝐹𝑒 2+ ]. 12 0,15.4
=
5 1
12
⋅ [𝐹𝑒 2+ ] = 0,6
5
0,6.5
∴ [𝐹𝑒 2+ ] = = 0,25
12
Gabarito: E

(Estratégia Militares – TFC – Inédita)


O esqueleto de um coral é formada essencial por uma mistura de carbonato de cálcio e magnésio.
Sabendo que uma amostra de 3,52 g desse esqueleto foi tratada com ácido clorídrico em excesso,
liberando 0,984 L de um gás à temperatura de 27 °C, determine o teor de cálcio no esqueleto.

Comentários
Primeiramente, vamos calcular as massas molares dos sais presentes no coral: o carbonato de
cálcio (CaCO3) e o de magnésio (MgCO3).
𝑀𝐶𝑎𝐶𝑂3 = 1.40 + 1.12 + 3.16 = 100 𝑔/𝑚𝑜𝑙
𝑀𝑀𝑔𝐶𝑂3 = 1.24 + 1.12 + 3.16 = 84 𝑔/𝑚𝑜𝑙
Sejam n1 o número de mols de CaCO3 presentes na amostra e n2 o número de mols de MgCO3
presentes na amostra, podemos escrever que a massa total do esqueleto é a soma das massas dos dois
sais, que podem ser obtidas como o produto do número de mols de fórmulas pela massa molar.
𝑚 = 𝑚𝐶𝑎𝐶𝑂3 + 𝑚𝑀𝑔𝐶𝑂3

(𝐼) 100. 𝑛1 + 84. 𝑛2 = 3,52

Agora, vamos escrever as reações desses carbonatos com o ácido clorídrico. Essas reações liberam
ácido carbônico (H2CO3), que se decompõe em água e gás carbônico.
𝑪𝒂𝑪𝑶𝟑 (𝒔) + 𝟐 𝑯𝑪𝒍 → 𝑪𝒂𝑪𝒍𝟐 (𝒂𝒒) + 𝑯𝟐 𝑶 (𝒈) + 𝑪𝑶𝟐 (𝒈)
𝑴𝒈𝑪𝑶𝟑 (𝒔) + 𝟐 𝑯𝑪𝒍 → 𝑴𝒈𝑪𝒍𝟐 (𝒂𝒒) + 𝑯𝟐 𝑶 (𝒈) + 𝑪𝑶𝟐 (𝒈)
Dessa forma, o número de mols de gás desprendidos na reação é igual à soma do número de mols
de cada sal. Podemos obter esse número de mols de CO2 pela Equação de Clapeyron a partir dos dados
fornecidos no enunciado.
𝑃𝑉 1.0,984
𝑃𝑉 = 𝑛𝑅𝑇 ∴ 𝑛𝐶𝑂2 = = = 0,04 𝑚𝑜𝑙
𝑅𝑇 0,082.300
Assim, podemos escrever:

AULA 10 – SOLUÇÕES 117


Prof. Thiago Cardoso

(𝐼𝐼) 𝑛1 + 𝑛2 = 0,04

Note que chegamos a um sistema com duas equações e duas incógnitas.

(𝐼) 100. 𝑛1 + 84. 𝑛2 = 3,52

(𝐼𝐼) 𝑛1 + 𝑛2 = 0,04

Podemos resolver pelo método da adição. Para isso, multiplicamos a equação (II) por 100 e
subtraímos (II) – (I).

(𝐼𝐼) 100. 𝑛1 + 100. 𝑛2 = 4

(𝐼) 100. 𝑛1 + 84. 𝑛2 = 3,52

(𝐼𝐼) − (𝐼) 100𝑛2 − 84𝑛2 = 4 − 3,52

Agora, façamos as contas:


16. 𝑛2 = 0,48
0,48
∴ 𝑛2 = = 0,03 𝑚𝑜𝑙
16
Podemos extrair n1 da equação (II):
𝑛1 + 𝑛2 = 0,04 ∴ 𝑛1 = 0,04 − 0,03 = 0,01 𝑚𝑜𝑙
Como uma fórmula de CaCO3 contém 1 átomo de cálcio, temos que a massa de cálcio presente no
esqueleto é igual ao número de mols multiplicado pela massa molar do cálcio.
𝑚𝐶𝑎 = 0,01.40 = 0,4 𝑔
Logo, o teor de cálcio no esqueleto é obtido como a razão entre a massa de cálcio e a massa total
do esqueleto.
0,4
%𝐶𝑎 = ≅ 0,114 = 11,4%
3,52
Gabarito: 11,4%

(Estratégia Militares – TFC – Inédita)


A adição de certa massa de cloreto de sódio em água provoca uma redução na pressão parcial da água
em 10%. Assinale a alternativa que indica a porcentagem em massa do cloreto de sódio nessa mistura.
a) 7,5%
b) 12,0%
c) 15,3%
d) 22,5%
e) 26,5%

AULA 10 – SOLUÇÕES 118


Prof. Thiago Cardoso

Comentários
Primeiramente, devemos considerar que, como expresso pela Lei de Raoult, o abaixamento
relativo da pressão de vapor da água é igual à fração molar do soluto.
Δ𝑃
= 𝑥1 = 0,10
𝑃0
Dessa forma, supondo uma solução com o total de 1 mol, podemos concluir que 0,1 mol vieram
do soluto e 0,9 mol vieram do solvente.
Agora, devemos notar que o soluto se dissocia.
𝑁𝑎𝐶𝑙 (𝑠) → 𝑁𝑎+ (𝑎𝑞) + 𝐶𝑙 − (𝑎𝑞)
Dessa forma, como o NaCl se dissocia liberando 2 mols de íons, podemos concluir que basta 0,05
mol do soluto para que se produzam 0,1 mol de íons.
Precisamos calcular as massas de cloreto de sódio (NaCl) e água (H2O) presentes na solução. Para
isso, precisamos das massas molares.
𝑀𝑁𝑎𝐶𝑙 = 23 + 35,5 = 58,5 𝑔/𝑚𝑜𝑙
𝑀𝐻2 𝑂 = 2.1 + 1.16 = 18 𝑔/𝑚𝑜𝑙
As massas dos compostos podem ser obtidas como o produto do número de mols pela massa
molar.
𝑚𝑁𝑎𝐶𝑙 = 𝑛𝑁𝑎𝐶𝑙 . 𝑀𝑁𝑎𝐶𝑙 = 0,05.58,5 = 2,925 𝑔
𝑚𝐻2 𝑂 = 𝑛𝐻2 𝑂 . 𝑀𝐻2 𝑂 = 0,90.18 = 16,2 𝑔
Assim, a massa de solução é igual à soma das massas do soluto e do solvente.
𝑚𝑠𝑜𝑙𝑢çã𝑜 = 𝑚𝑁𝑎𝐶𝑙 + 𝑚𝐻2 𝑂 = 2,925 + 16,2 = 19,125 𝑔
Por fim, o percentual em massa de NaCl na solução é:
𝑚𝑁𝑎𝐶𝑙 2,925
𝜏= = ≅ 0,153 = 15,3%
𝑚𝑠𝑜𝑙𝑢çã𝑜 19,125
Gabarito: C

(Estratégia Militares – TFC – Inédita)


Uma amostra de platina está impregnada de ferro metálico como impureza. A fim de retirar o ferro da
liga, adicionou-se ácido nítrico concentrado, produzindo uma reação com liberação de dióxido de
nitrogênio. Sabendo que uma amostra de 10g da liga metálica foi tratada com 400 mL de uma solução
0,8 mol/L de ácido nítrico, determine a pureza do metal:
a) 45%
b) 55%
c) 65%
d) 75%
e) 85%

AULA 10 – SOLUÇÕES 119


Prof. Thiago Cardoso

Comentários
A reação do ferro com ácido nítrico concentrado balanceada é a seguinte:
𝐹𝑒 (𝑠) + 4 𝐻𝑁𝑂3 (𝑎𝑞) → 𝐹𝑒(𝑁𝑂3 )2 + 2 𝑁𝑂2 (𝑔) + 2 𝐻2 𝑂 (𝑙)
O número de mols de ácido nítrico pode ser obtido a partir da concentração molar da solução,
como o produto da concentração molar pelo volume.
𝑛𝐻𝑁𝑂3 = [𝐻𝑁𝑂3 ]. 𝑉 = 0,4.0,8 = 0,32 𝑚𝑜𝑙
O número de mols de ferro presentes na liga pode ser obtido pela proporção estequiométrica.
𝑛𝐹𝑒 𝑛𝐻𝑁𝑂3
=
1 4
0,32
∴ 𝑛𝐹𝑒 = = 0,08 𝑚𝑜𝑙
4
Logo, a massa de ferro presente na amostra pode ser obtido como o produto entre o número de
mols e a massa molar do ferro.
𝑚𝐹𝑒 = 0,08.56 ≅ 4,5 𝑔
Dessa forma, a massa de platina presente no material é 5,5 g e a sua pureza é:
5,5
𝑝𝑢𝑟𝑒𝑧𝑎 = = 0,55 = 55%
10
Gabarito: B

(Estratégia Militares – TFC – Inédita)


Uma amostra de 10 mL de uma solução aquosa de cloreto de ferro (II) foi tratada com uma solução
aquosa 0,100 mol/L em permanganato de potássio em meio ácido, de acordo com a seguinte reação
não balanceada:
Fe2+ (aq) + MnO4 – (aq) + H+ (aq) → Fe3+ (aq) + Mn2+ (aq) + H2O ()
Sabendo que o volume utilizado da solução de permanganato de potássio foi igual a 2 mL, determine a
concentração molar da solução original.
a) 0,05 mol/L
b) 0,10 mol/L
c) 0,15 mol/L
d) 0,20 mol/L
e) 0,25 mol/L

Comentários

Como informado pelo enunciado, a reação é:

𝐹𝑒 2+ (𝑎𝑞) + 𝑀𝑛𝑂4− (𝑎𝑞) + 𝐻 + (𝑎𝑞) → 𝐹𝑒 3+ (𝑎𝑞) + 𝑀𝑛2+ (𝑎𝑞) + 𝐻2 𝑂(𝑙)

Podemos deixar em evidência os elementos, cujos números de oxidação sofreram variação.

AULA 10 – SOLUÇÕES 120


Prof. Thiago Cardoso

Agora, vamos usar a técnica de inverter os coeficientes.

Nós já temos uma proporção entre os íons Fe2+ presentes na amostra inicial e os íons MnO4–
presentes na solução titulante. Pela Proporção Estequiométrica, temos:
𝑛𝐹𝑒 2+ 𝑛𝑀𝑛𝑂4−
=
5 1

O número de mols de íons Fe2+ pode ser obtido como o produto da concentração molar pelo

volume da solução. O mesmo válido para o permanganato.

[𝐹𝑒 2+ ]. 𝑉 [𝑀𝑛𝑂4− ]. 𝑉𝑀𝑛𝑂4−


=
5 1

Substituindo os valores fornecidos no enunciado, temos:

[𝐹𝑒 2+ ]. 10 0,1.2
=
5 1

2[𝐹𝑒 2+ ] = 0,2

0,2
∴ [𝐹𝑒 2+ ] = = 0,1
2

Gabarito: B

AULA 10 – SOLUÇÕES 121


Prof. Thiago Cardoso

(Estratégia Militares – TFC – Inédita)


Sendo dadas as condutividades molares dos íons bário e cloreto: σ(Ba2+) = 12,7 mS m² mol –1 e σ(Cl–) =
7,65 mS m² mol –1, assinale a alternativa que apresenta a condutividade de uma solução aquosa 10,4
g/L de cloreto de bário.

Comentários
Primeiramente, vamos calcular a concentração molar dessa solução citada. Para isso, basta dividir
a concentração em massa pela massa molar do sal.
𝑀𝐵𝑎𝐶𝑙2 = 137 + 2.35,5 = 208 𝑔/𝑚𝑜𝑙
𝐶 10,4
∴ [𝐵𝑎𝐶𝑙2 ] = = = 0,05 𝑚𝑜𝑙/𝐿
𝑀 208
Observe que, por força da estequiometria do sal, teremos:
𝑚𝑜𝑙 0,05𝑚𝑜𝑙
[𝐵𝑎2+ ] = 0,05 = −3 3 = 5.10−2+3 = 50 𝑚𝑜𝑙. 𝑚−3
𝐿 10 𝑚
𝑚𝑜𝑙 0,10 𝑚𝑜𝑙
[𝐶𝑙 − ] = 0,10 = = 100 𝑚𝑜𝑙. 𝑚−3
𝐿 10−3
Como a condutividade foi fornecida em mol/L, faz sentido obter as condutividades em mol/m³,
que são as unidades do SI.
𝜎 = 50.12,7 + 100.7,65 = 635 + 765 = 1400 𝑚𝑆. 𝑚−1 = 1,4 𝑆. 𝑚−1
Gabarito: 1,4 S.m–1

(Estratégia Militares – TFC – Inédita)


A condutividade de uma solução aquosa 0,01 mol.L–1 de um ácido CH3COOH é igual a 16 mS.m–1.
Conhecendo-se as condutividades molares dos íons H+ (36 mS m² mol–1) e CH3COO– (4 mS m² mol–1),
determine a constante de ionização do ácido.

Comentários
Se o ácido acético fosse um ácido forte, a condutividade molar da solução seria:
𝜎 = [𝐻 + ]𝜎(𝐻 + ) + [𝐶𝐻3 𝐶𝑂𝑂− ]𝜎(𝐶𝐻3 𝐶𝑂𝑂− )
Nesse caso, se o ácido fosse forte, teríamos:
0,01 𝑚𝑜𝑙 0,01 𝑚𝑜𝑙
[𝐻 + ] = = −3 3 = 10 𝑚𝑜𝑙. 𝑚−3
1𝐿 10 𝑚
0,01 𝑚𝑜𝑙 0,01 𝑚𝑜𝑙
[𝐶𝐻3 𝐶𝑂𝑂− ] = = −3 3 = 10 𝑚𝑜𝑙. 𝑚−3
1𝐿 10 𝑚
Dessa forma, a condutividade molar da solução seria:
𝜎 = 10.36 + 10.4 = 360 + 40 = 400 𝑚𝑆. 𝑚−1
Podemos calcular o grau de ionização como a razão entre a condutividade real e a condutividade
máxima, ou seja, aquela que seria obtida se o ácido fosse forte.

AULA 10 – SOLUÇÕES 122


Prof. Thiago Cardoso

𝜎𝑟𝑒𝑎𝑙 16
𝛼= = = 0,04 = 4.10−2
𝜎𝑚𝑎𝑥 400
Por fim, pela Lei de Ostwald, temos:
𝛼2𝑀 (4.10−2 )2 . (10−2 ) 16.10−4 . 10−2
𝐾𝑎 = = = = 16,7.10−6 ≅ 1,7.10−5
1−𝛼 1 − 0,04 0,96
Gabarito: 1,7.10–5

7.2. Lista de Questões Nível II

(ITA 2021 – 1ª Fase)


Sabe-se que a condutividade molar (Ʌ) de uma solução iônica é dada pela razão entre a condutividade
dessa solução (k) e sua concentração molar. Considere soluções diluídas de CaC2, NaC e KC com iguais
concentrações em massa, para as quais são observadas as seguintes razões entre condutividades
molares e entre massas molares (MM):
ꓥ(CaCl2)/ ꓥ(NaCl) = 1,9; ꓥ(CaCl2)/ ꓥ(KCl) = 1,8;
MM(CaCl2)/MM(NaCl) = 1,9; MM(CaCl2)/MM(KCl) = 1,5.
Com base nessas informações, assinale a opção CORRETA entre as condutividades das soluções.
a) kCaC2 = kNaC = kkC
b) kCaC2 = kNaC > kkC
c) kCaC2 > kNaC > kkC
d) kCaC2 < kNaC = kkC
e) kCaC2 < kNaC < kkC

Comentários
Aqui, o ITA definiu o conceito de condutividade molar, o que ajuda bastante na resolução. Assim,
podemos escrever:
𝜅
𝐴=
𝑚
Mas
𝑛 𝑚𝑎𝑠𝑠𝑎 𝑚𝑎𝑠𝑠𝑎 1
𝑚= = = ⋅
𝑉 𝑀𝑀 ⋅ 𝑉 𝑉 𝑀𝑀
O enunciado cita que as concentrações em massa são iguais. Desse modo, seja “C” igual a razão
da massa pelo volume, temos:
𝐶
𝑚=
𝑀𝑀
Logo
𝜅 𝜅
𝐴= = ⋅ 𝑀𝑀
𝐶 𝐶
𝑀𝑀

AULA 10 – SOLUÇÕES 123


Prof. Thiago Cardoso

Então, usando os dados do enunciado, vem:


𝜅𝐶𝑎𝐶𝑙2
𝐴𝐶𝑎𝐶𝑙2 ⋅ 𝑀𝑀𝐶𝑎𝐶𝑙2 𝜅𝐶𝑎𝐶𝑙2 ⋅ 𝑀𝑀𝐶𝑎𝐶𝑙2
= 𝜅𝐶 =
𝐴𝑁𝑎𝐶𝑙 𝑁𝑎𝐶𝑙 𝜅𝑁𝑎𝐶𝑙 ⋅ 𝑀𝑀𝑁𝑎𝐶𝑙
𝐶 ⋅ 𝑀𝑀𝑁𝑎𝐶𝑙
𝜅𝐶𝑎𝐶𝑙2 𝐴𝐶𝑎𝐶𝑙2 𝑀𝑀𝑁𝑎𝐶𝑙
= ⋅
𝜅𝑁𝑎𝐶𝑙 𝐴𝑁𝑎𝐶𝑙 𝑀𝑀𝐶𝑎𝐶𝑙2
𝜅𝐶𝑎𝐶𝑙2 1
= 1,9 ⋅ =1
𝜅𝑁𝑎𝐶𝑙 1,9
𝜅𝐶𝑎𝐶𝑙2 = 𝜅𝑁𝑎𝐶𝑙
Analogamente, temos:
𝜅𝐶𝑎𝐶𝑙2
𝐴𝐶𝑎𝐶𝑙2
= 𝐶 ⋅ 𝑀𝑀𝐶𝑎𝐶𝑙2 = 𝜅𝐶𝑎𝐶𝑙2 ⋅ 𝑀𝑀𝐶𝑎𝐶𝑙2
𝐴𝐾𝐶𝑙 𝜅𝐾𝐶𝑙 𝜅𝐾𝐶𝑙 ⋅ 𝑀𝑀𝐾𝐶𝑙
𝐶 ⋅ 𝑀𝑀𝐾𝐶𝑙
𝜅𝐶𝑎𝐶𝑙2 𝐴𝐶𝑎𝐶𝑙2 𝑀𝑀𝐾𝐶𝑙
= ⋅
𝜅𝐾𝐶𝑙 𝐴𝐾𝐶𝑙 𝑀𝑀𝐶𝑎𝐶𝑙2
𝜅𝐶𝑎𝐶𝑙2 1
= 1,8 ⋅
𝜅𝐾𝐶𝑙 1,5
𝜅𝐶𝑎𝐶𝑙2
= 1,2
𝜅𝐾𝐶𝑙
Como a fração resultante é maior que 1, concluímos:
𝜅𝐶𝑎𝐶𝑙2 > 𝜅𝐾𝐶𝑙
Logo, temos a ordem:
𝜅𝐶𝑎𝐶𝑙2 = 𝜅𝑁𝑎𝐶𝑙 > 𝜅𝐾𝐶𝑙
Gabarito: B

(ITA 2021 – 1ª Fase)


Considere as curvas de solubilidade de sais inorgânicos mostradas na figura. A respeito de alguns destes
sias são feitas as seguintes afirmações.

AULA 10 – SOLUÇÕES 124


Prof. Thiago Cardoso

I. Dissolvendo-se 130g de KNO3 em 200 g de água, a 40 oC, obteremos uma solução saturada com
depósito de 70g desta substância que não será dissolvida.
II. Se dissolvermos 20g de Ce2(SO4)3 em 300g de água a 10 oC e, posteriormente, aquecermos esta
solução a 90 oC, haverá gradativa precipitação da substância.
III. A menor quantidade de água necessária para dissolver completamente 140g de K 2Cr2O7 a 90 oC é,
aproximadamente, 150g.
IV. NaNO3 é a substância mais solúvel a 30 oC.
Das afirmações acima, está(ão) CORRETA(S)
a) apenas I, II e IV.
b) apenas I e III.
c) apenas II.
d) apenas III e IV.
e) nenhuma.

Comentários

I – Pelo gráfico, a solubilidade do KNO3 a 40 oC é de 60g KNO3/100g H2O.

Podemos fazer uma regra de 3 para obter a solubilidade máxima 𝒙 do sal para 200g de água:

𝟔𝟎 𝒙
= → 𝒙 = 𝟏𝟐𝟎𝒈
𝟏𝟎𝟎 𝟐𝟎𝟎

Portanto, como a solubilidade do sal é de 120g, teremos como resultado uma solução saturada
com depósito de 10g de KNO3 não dissolvido. Afirmação falsa.

II – Observe que a solubilidade do Ce2(SO4)3 a 10 oC é de 10g/100g H2O

AULA 10 – SOLUÇÕES 125


Prof. Thiago Cardoso

Por regra de 3, obtém-se que a solubilidade para 300g de água será de 30g. Como misturam-se
apenas 20g de sal, todo o Ce2(SO4)3 será dissolvido. Quando se aquece a mistura a 90 oC, a solubilidade
do sal reduz para um valor menor que 5g/100g H2O, de modo que, efetivamente, menos de 15 gramas de
Ce2(SO4)3 poderiam estar dissolvidos. Assim, ocorre precipitação de parte do sal. Afirmação verdadeira.

III – A solubilidade do K2Cr2O7 a 90 oC é pouco menor do que 70g/100g H2O.

Podemos calcular a quantidade mínima de água necessária 𝒚 para dissolver 140g do sal por uma
regra de três:

AULA 10 – SOLUÇÕES 126


Prof. Thiago Cardoso

𝟕𝟎 𝟏𝟒𝟎
= → 𝒚 ≈ 𝟐𝟎𝟎𝒈
𝟏𝟎𝟎 𝒚

Portanto, a quantia de água necessária destoa muito de 150g. Afirmação falsa.

IV – É possível observar no gráfico que a solubilidade do CaCl2 é caracterizado por uma curva
crescente, e já passa a superar a solubilidade do NaNO3 por volta de 25 oC. A 30 oC, portanto, é esperado
que a curva do CaCl2 esteja acima da do NaNO3 , sendo, então, o CaCl2 o sal mais solúvel a 30 oC.
Afirmação falsa.

Na temperatura de 30 °C, a solubilidade do cloreto de cálcio é tão elevada que nem aparece no
gráfico.
Gabarito: C

(ITA 2021 – 1ª Fase)


Numa titulação de oxirredução, 50,00 mL de uma solução ácida de Fe(NO 3)2 a 0,38 mol-L-1 foi titulada
com uma solução padronizada de permanganato de potássio a 4,2 x 10 -2 mol-L-1, até que a solução
resultante adquirisse leve coloração rósea. Sobre esta titulação, são feitas as seguintes afirmações:
(1) O volume da solução de permanganato de potássio gasto na titulação foi de 100 mL.
(2) O menor coeficiente estequiométrico inteiro para Fe2+ na reação redox balanceada é 7.
(3) No ponto final, o volume total da solução será de 120 mL.
(4) Um precipitado sólido de cor esverdeada será observado como produto dessa reação.
(5) O número total de elétrons envolvidos na reação redox é 22 milimols.
(6) A razão entre os volumes do titulante e do titulado no ponto final é 2,1.
A soma dos números associados às afirmações CORRETAS é igual a
a) 0

AULA 10 – SOLUÇÕES 127


Prof. Thiago Cardoso

b) 1
c) 3
d) 6
e) 11

Comentários
O permanganato de potássio em meio ácido é um poderoso agente oxidante.

𝐹𝑒 2+ (𝑎𝑞) + 𝑀𝑛𝑂4− (𝑎𝑞) + 𝐻 + (𝑎𝑞) → 𝐹𝑒 3+ (𝑎𝑞) + 𝑀𝑛2+ (𝑎𝑞) + 𝐻2 𝑂(𝑙)

verde violeta amarelo incolor

Precisamos agora balancear a equação. Para isso, devemos notar que o ferro perde 1 elétron,
enquanto o manganês ganha 5. Portanto, podemos usar coeficientes cruzados – ou seja, o coeficiente do
ferro será 5 e o do manganês será 1.
5 𝐹𝑒 2+ + 1 𝑀𝑛𝑂4− (𝑎𝑞) + 𝐻 + (𝑎𝑞) → 5 𝐹𝑒 3+ + 1 𝑀𝑛2+ (𝑎𝑞) + 𝐻2 𝑂
Note que temos 17 cargas positivas do lado dos produtos. Portanto, precisamos de 8 H + para
balancear a carga global no lado dos reagentes.
5 𝐹𝑒 2+ + 1 𝑀𝑛𝑂4− (𝑎𝑞) + 8 𝐻 + (𝑎𝑞) → 5 𝐹𝑒 3+ + 1 𝑀𝑛2+ (𝑎𝑞) + 𝐻2 𝑂
Para balancear os hidrogênios, precisamos do coeficiente 4 H2O.
5 𝐹𝑒 2+ + 1 𝑀𝑛𝑂4− (𝑎𝑞) + 8 𝐻 + (𝑎𝑞) → 5 𝐹𝑒 3+ + 1 𝑀𝑛2+ (𝑎𝑞) + 4 𝐻2 𝑂
Observe que temos 4 átomos de oxigênio de cada lado da reação, portanto, ela está, de fato,
balanceada.
Podemos utilizar a proporção estequiométrica para os íons Fe2+ e MnO4–.
𝑛𝐹𝑒 2+ 𝑛𝑀𝑛𝑂4−
=
5 1
(1) O volume de permanganato pode ser obtido usando a proporção estequiométrica acima.
Devemos nos lembrar que o número de mols é igual ao produto da concentração molar do íon pelo
volume empregado da solução.
[𝐹𝑒 2+ ]. 𝑉𝐹𝑒 2+ [𝑀𝑛𝑂4− ]. 𝑉𝑀𝑛𝑂4−
=
5 1
2+
[𝐹𝑒 ]. 𝑉𝐹𝑒 2+ (0,38). 50 19
∴ 𝑉𝑀𝑛𝑂4− = − = −2
= ≅ 90 𝑚𝐿
5. [𝑀𝑛𝑂4 ] 5.4,2.10 21.10−2
Afirmação incorreta.
(2) Como vimos, o coeficiente é igual a 5. Afirmação incorreta.
(3) O volume utilizado do titulante é igual a 90 mL e volume inicial da solução é igual a 50 mL,
portanto, o volume final será igual a 140 mL. Afirmação incorreta.
(4) Na reação, não há a produção de nenhum precipitado. Todos os produtos envolvidos estão
dissolvidos em meio aquoso. Afirmação incorreta.
(5) Como cada átomo de Fe2+ perde 1 elétron, podemos estabelecer que o número de elétrons
envolvidos na oxirredução é igual ao número de íons Fe2+.

AULA 10 – SOLUÇÕES 128


Prof. Thiago Cardoso

𝑛𝑒𝑙é𝑡𝑟𝑜𝑛𝑠 = 1. 𝑛𝐹𝑒 2+ = [𝐹𝑒 2+ ]. 𝑉𝐹𝑒 2+ = 0,38.50.10−3 = 19.10−3 𝑚𝑜𝑙


Afirmação incorreta.
(6) Vamos calcular a razão pedida.
90
𝑝= = 1,8
50
Afirmação incorreta.
Logo, não há nenhuma afirmação correta. Portanto, a soma é igual a 0.
Gabarito: A

(IME – 2020 – 1 Fase)


Na figura abaixo, é mostrado o diagrama de fases Temperatura versus Composição (fração molar) de
dois líquidos voláteis, hexano (P.E. = 69 °C) e octano (P.E. = 126 °C), para a pressão de 1 atm.

Considere uma mistura binária líquida ideal de hexano e octano, contendo 20% de hexano. Quando
essa mistura é aquecida, ela entra em ebulição, possibilitando a marcação do ponto A, que representa
o líquido  em ebulição e o ponto B, que representa o vapor  gerado pela vaporização do líquido .
Considere, agora, que o vapor  seja condensado e em seguida vaporizado, gerando o vapor . Com
base nessas informações, determine a:
a) composição no ponto B;
b) temperatura aproximada de ebulição da mistura líquida de partida que contém 20% de hexano;
c) composição do líquido formado pela condensação do vapor ;
d) composição do vapor .

Comentários:
Essa questão trata do processo de destilação fracionada de uma solução. Devemos notar que a
destilação é composta de duas etapas:
• Ebulição: que acontece a temperatura constante;
• Condensação: que preserva a composição da mistura. Desse modo, o líquido condensado tem
sempre a mesma composição do vapor.

AULA 10 – SOLUÇÕES 129


Prof. Thiago Cardoso

Inicialmente, temos:
𝑥0 ℎ𝑒𝑥𝑎𝑛𝑜 = 0,2
{
𝑥0 𝑜𝑐𝑡𝑎𝑛𝑜 = 0,8
De acordo com o gráfico, podemos destacar os seguintes pontos:

Então:
i. O processo é constituído de 3 etapas.
ii. A curva inferior representa a composição do líquido, a curva superior representa a
composição do vapor e a região entre eles representa a mistura: vapor + líquido.
iii. ̅̅̅̅) . Portanto, pelo diagrama:
(2) representa a vaporização do líquido inicial (𝐴𝐵
𝑥𝑜𝑐𝑡𝑎𝑛𝑜,𝑣𝑎𝑝𝑜𝑟 = 0,5 ⇒ 𝑥ℎ𝑒𝑥𝑎𝑛𝑜,𝑣𝑎𝑝𝑜𝑟 = 0,5 (𝑎) 𝑒 𝑇𝑒𝑚𝑝. 𝑒𝑏. = 104 °𝐶 (𝑏)
(3) representa a condensação do vapor 𝛽 (𝐵𝐶 ̅̅̅̅ ). Portanto:
𝑥𝑜𝑐𝑡𝑎𝑛𝑜,𝑙í𝑞𝑢𝑖𝑑𝑜 = 0,5 ⇒ 𝑥ℎ𝑒𝑥𝑎𝑛𝑜,𝑙í𝑞𝑢𝑖𝑑𝑜 = 0,5 (𝑐)
̅̅̅̅ ). Portanto:
(4) representa a vaporização do líquido 𝛽 (𝐶𝐷
𝑥𝑜𝑐𝑡𝑎𝑛𝑜,𝑣𝑎𝑝𝑜𝑟 = 0,2 ⇒ 𝑥ℎ𝑒𝑥𝑎𝑛𝑜,𝑣𝑎𝑝𝑜𝑟 = 0,8 (𝑑)
Gabarito: a) xoct = 0,5 e xhex = 0,5; b) 104 °C; c) xoct = 0,5 e xhex = 0,5; d) xoct = 0,2 e xhex = 0,8

(IME – 2020 – 1 Fase)


Um minério de ferro, contendo Fe3O4, foi analisado a partir da dissolução de uma amostra de massa
1,161 g em ácido. Na dissolução, todo o ferro proveniente do Fe3O4 foi reduzido a Fe2+. A seguir, a
amostra foi titulada com 40 mL de uma solução 0,025 mol/L de KMnO4, tendo como produtos Mn2+ e
Fe3+.
Diante do exposto:
a) escreva a equação iônica global simplificada de oxirredução, balanceada, ocorrida na titulação;
b) determine a porcentagem em massa de Fe3O4 no minério.

Comentários e Resolução:
a) De acordo com enunciado e aplicando o método do íon-elétron, temos:
5𝑒 − + 8𝐻 + + 𝑀𝑛𝑂4− → 𝑀𝑛2+ + 4𝐻2 𝑂

AULA 10 – SOLUÇÕES 130


Prof. Thiago Cardoso

𝐹𝑒 2+ → 𝐹𝑒 3+ + 1𝑒 − (𝑥5)
Portanto:
5𝐹𝑒 2+ + 𝑀𝑛𝑂4− + 8𝐻 + → 𝑀𝑛2+ + 5𝐹𝑒 3+ + 4𝐻2 𝑂
b) Agora, calcula-se a quantidade de 𝐾𝑀𝑛𝑂4.
𝑛𝐾𝑀𝑛𝑂4 = 𝑉𝐾𝑀𝑛𝑂4 ∙ 𝑀𝐾𝑀𝑛𝑂4 = 4 ∙ 10−2 ∙ 25 ∙ 10−3 = 10−3 𝑚𝑜𝑙
Pela proporção estequiométrica da reação, tem-se que:
𝑛𝐹𝑒 2+ = 5 ∙ 10−3
Note que todo 𝐹𝑒 2+ veio de 𝐹𝑒3 𝑂4. Portanto, temos que:
5
∙ 10−3 𝑚𝑜𝑙 𝑑𝑒 𝐹𝑒3 𝑂4
3
De acordo com os dados fornecidos no início da prova, a massa molar do 𝐹𝑒3 𝑂4 é igual a
232 𝑔/𝑚𝑜𝑙. Logo:
5
𝑚𝐹𝑒3𝑂4 = 232 ∙ ∙ 10−3 𝑔 = 0,386 𝑔
3
Portanto:
0,386
%𝐹𝑒3 𝑂4 = = 33,3 %
1,161
Gabarito: a) 5 Fe2+ + MnO4– + 8 H+ → Mn2+ + 5Fe3+ + 4 H2O; b) 33,33%

(IME – 2020 – 1 Fase)


Em um experimento em laboratório, tomaram-se duas amostras de 0,177 g de um composto de fórmula
CaHbOcNd. Uma das amostras foi completamente consumida por combustão, gerando 0,264 g de CO 2 e
0,135 g de vapor de água. A outra reagiu totalmente com compostos não nitrogenados, gerando amônia
como único produto nitrogenado, a qual necessitou de 3 cm 3 de uma solução 0,5 mol/L de ácido
sulfúrico para sua completa neutralização. Determine a fórmula empírica do composto.

Comentários:
Para a primeira amostra, podemos escrever que:
1 𝑏 𝑏 𝑑
𝐶𝑎 𝐻𝑏 𝑂𝑐 𝑁𝑑 + (2𝑎 + − 𝑐)𝑂2(g) ⟶ 𝑎𝐶𝑂2(g) + 𝐻2 𝑂(g) + 𝑁2(g)
2 2 2 2
0,264
𝑛𝐶𝑂2 = = 0,006
44
0,135
𝑛𝐻2 𝑂 = = 0,0075
18
Como todo o carbono e todo o hidrogênio vieram do composto orgânico, temos que:
𝑎 0,006 𝑎 2
= ⟹ = (𝐼)
𝑏 0,0075 𝑏 5
2
Para a segunda amostra:
𝐶𝑎 𝐻𝑏 𝑂𝑐 𝑁𝑑 + 𝑅𝑒𝑎𝑔𝑒𝑛𝑡𝑒𝑠𝑛ã𝑜𝑛𝑖𝑡𝑟𝑜𝑔𝑒𝑛𝑎𝑑𝑜𝑠 ⟶ 𝑑𝑁𝐻3(g) + 𝑃𝑟𝑜𝑑𝑢𝑡𝑜𝑠
2𝑁𝐻3(g) + 𝐻2 𝑆𝑂4(aq) ⟶ (𝑁𝐻4 )2 𝑆𝑂4(aq)
Pelas informações do enunciado e pela estequiometria do problema, temos:

AULA 10 – SOLUÇÕES 131


Prof. Thiago Cardoso

𝑛𝑁𝐻3 = 2 ⋅ 0,5 ⋅ 3 ⋅ 10−3 𝑚𝑜𝑙 = 3 ⋅ 10−3 𝑚𝑜𝑙


Como todo o nitrogênio veio do composto orgânico e massa das duas amostras é igual, ganhamos
que:
𝑎 0,006 𝑎 2
= ⟹ = (𝐼𝐼)
𝑑 0,003 𝑑 1
Agora, precisamos determinar a relação entre 𝑎 e 𝑐.
Da questão, temos que a massa do composto orgânico na primeira amostra é 𝑚 = 0,177𝑔
Podemos escrever a seguinte relação:
0,177 0,006
= (𝐼𝐼𝐼)
12𝑎 + 𝑏 + 16𝑐 + 14𝑑 a

De (𝐼), (𝐼𝐼) e (𝐼𝐼𝐼), temos que:


𝑎 2
=
𝑐 1
Reescrevendo a fórmula do composto, obtemos:
𝐶𝑎 𝐻5𝑎 𝑂𝑎 𝑁𝑎 , o qual possui fórmula empírica (mínima) 𝐶2 𝐻5 𝑂1 𝑁1
2 2 2

Assim, a formula empírica é: 𝐶2 𝐻5 ON


Gabarito: C2H5NO

(IME – 2019 – 1 Fase)


É requerido que fazendas produtoras de leite bovino controlem a acidez do leite que está aguardando
o processamento. Essa acidez é resultante da conversão da lactose em ácido lático (ácido 2-
hidroxipropanoico) por ação de microrganismos:
C12H22O11 + H2O → 4C3H6O3
Um fazendeiro decidiu fazer um experimento para determinar a taxa de geração de ácido lático no leite
armazenado: retirou uma amostra de 50 cm3 de leite, cuja concentração de ácido lático é de 1,8 g/L, e,
depois de três horas, utilizou 40 cm3 de uma solução 0,1 molar de NaOH para neutralizá-la.
Conclui-se que a taxa média de produção de ácido lático por litro de leite é:
a) 0,25 mg/L.s
b) 0,33 mg/L.s
c) 0,50 mg/L.s
d) 0,67 mg/L.s
e) 1,00 mg/L.s

Comentários
O ácido láctico é um monoácido, como podemos evidenciar na sua estrutura.

AULA 10 – SOLUÇÕES 132


Prof. Thiago Cardoso

Sendo assim, a neutralização ocorre na proporção 1:1, já que o hidróxido de sódio (NaOH) é
também uma monobase.
Dessa maneira, podemos calcular a quantidade de mols de ácido lático presentes na solução ao
final das três horas.
𝑛𝐻 + = 𝑛𝑂𝐻 −
1. [𝑙á𝑡𝑖𝑐𝑜]. 𝑉𝑙á𝑡𝑖𝑐𝑜 = 1. [𝑁𝑎𝑂𝐻]. 𝑉𝑁𝑎𝑂𝐻
1. [𝑙á𝑡𝑖𝑐𝑜]. 50 = 1.0,1.40
0,1.40 4
∴ [𝑙á𝑡𝑖𝑐𝑜] = = = 0,08 𝑚𝑜𝑙/𝐿
50 50
Podemos converter a concentração de mol/L para g/L como pedido pelo enunciado aplicando a
massa molar.
𝑀𝑙á𝑡𝑖𝑐𝑜 = 3.12 + 6.1 + 3.16 = 36 + 6 + 48 = 90 𝑔/𝑚𝑜𝑙
Basta, portanto, multiplicar.
𝐶𝑓𝑖𝑛𝑎𝑙 = 0,08.90 = 7,2 𝑔/𝐿
Sendo assim, no período de 3 horas, a quantidade de ácido lático produzida pela amostra de 50
mL leite foi:
Δ𝐶 = 𝐶𝑓𝑖𝑛𝑎𝑙 − 𝐶𝑖𝑛𝑖𝑐𝑖𝑎𝑙 = 7,2 − 1,8 = 5,4 𝑔/𝐿
Essa concentração foi produzida em 3 horas. Portanto, a velocidade de produção pode ser
calculada dividindo-se a concentração produzida pelo tempo. Como foi pedido o tempo em segundos, é
útil lembrar que 1 hora tem 60 minutos e que cada minuto tem 60 segundos. Portanto, 1 hora é
equivalente a 60x60 = 3600 segundos.
Δ𝐶 5,4 𝑔/𝐿 5,4 𝑔/𝐿 5,4 1,8
𝑣= = = = =
Δ𝑡 3ℎ 3. 3600 𝑠 3.3600 3600
1,8 1 1
𝑣= = = . 10−3 = 0,5.10−3 𝑔/𝐿. 𝑠 = 0,5 𝑚𝑔/𝐿. 𝑠
3600 2000 2

Gabarito: C

AULA 10 – SOLUÇÕES 133


Prof. Thiago Cardoso

(ITA – 2018)
São feitas as seguintes proposições a respeito de propriedades coligativas:
I. A pressão osmótica depende do tipo de solvente para um dado soluto.
II. A criometria usa o abaixamento do ponto de congelamento do solvente para medir a massa molar
do soluto.
III. Na ebuliometria, a variação da temperatura de ebulição depende da concentração molal de soluto
não volátil utilizado.
IV. Na tonometria, ocorre abaixamento da pressão de vapor de uma solução que contém um soluto não
volátil, em relação ao solvente puro.

Das proposições acima é(são) CORRETA(S)

a) apenas I.
b) apenas I e III.
c) apenas II, III e IV.
d) apenas II e IV.
e) todas.

Comentários
Vamos analisar as proposições individualmente.
I – É muito interessante notar que a pressão osmótica é expressa unicamente em função da
concentração molar do soluto.
𝜋 = [𝑋]𝑅𝑇
Na expressão da pressão osmótica, portanto, não existe nenhum termo que dependa do solvente.
Sendo assim, não importa qual o solvente, pois a pressão osmótica depende apenas da concentração
molar do soluto e da temperatura absoluta. Proposição errada.
II – É possível sim determinar a massa molar do soluto a partir do abaixamento do ponto de
congelamento do solvente. Proposição correta.
III – Perfeito. A concentração molal do soluto é o fator a ser utilizado no estudo do abaixamento
do ponto de congelamento e do aumento do ponto de ebulição. Proposição correta.
IV – Se o soluto for não volátil, de fato, ele provocará redução na pressão de vapor do solvente.
Proposição correta.
Portanto, somente as proposições II, III e IV estão corretas.

Gabarito: C

AULA 10 – SOLUÇÕES 134


Prof. Thiago Cardoso

(IME – 2019 – 1ª Fase – adaptada)


Admita que uma solução aquosa 0,0400 molar de ácido tricloroacético congele a –0,1395 ºC. Considere,
ainda, que a constante de abaixamento do ponto de congelamento (K c) da água seja 1,860 ºC.kg.mol–1
e que 1,00 L de solução contenha 1,00 kg de solvente. Calcule o grau de ionização ácido tricloroacético
nessa solução.

Comentários
A temperatura de congelamento pode ser calculada a partir da equação do efeito crioscópico. A
suposição de que 1,00 L de solução contenha 1,00 kg de solvente implica que a molaridade seja igual à
molalidade.
Δ𝑇𝐶 = 𝐾𝐶 . 𝑊. 𝑖
0,1395 = 1,86.0,04. 𝑖
0,1395
∴𝑖= = 1,875
1,86.0,04
Já vimos que o fator de van’t Hoff pode ser expresso diretamente em função do grau de ionização.
𝑖 =1+𝛼
∴ 𝛼 = 𝑖 − 1 = 1,875 − 1 = 0,875 = 87,5%
Caso você não se lembrasse dessa relação, o fator de van’t Hoff é expresso como a razão entre o
número de partículas produzidas ao final da dissolução e o número de moléculas do soluto inicialmente
dissolvidas.
O ácido tricloroacético se ioniza, pois é um monoácido moderado.
𝐶𝐶𝑙3 𝐶𝑂𝑂𝐻 → 𝐶𝐶𝑙3 𝐶𝑂𝑂− (𝑎𝑞) + 𝐻 + (𝑎𝑞)
Vamos escrever essa ionização na Tabela Estequiométrica.

𝑪𝑪𝒍𝟑 𝑪𝑶𝑶𝑯 → 𝑪𝑪𝒍𝟑 𝑪𝑶𝑶− (𝒂𝒒) + 𝑯+ (𝒂𝒒)

Início 1 0 0

Reage/forma 𝛼 𝛼 𝛼

Final 1−𝛼 𝛼 𝛼

𝑇𝑜𝑡𝑎𝑙 𝑑𝑒 𝑃𝑎𝑟𝑡í𝑐𝑢𝑙𝑎𝑠 𝐹𝑖𝑛𝑎𝑖𝑠 = (1 − 𝛼) + 𝛼 + 𝛼 = 1 + 𝛼


𝑃𝑎𝑟𝑡í𝑐𝑢𝑙𝑎𝑠 𝐹𝑖𝑛𝑎𝑖𝑠 1+𝛼
𝑖= = =1+𝛼
𝑃𝑎𝑟𝑡í𝑐𝑢𝑙𝑎𝑠 𝐼𝑛𝑖𝑐𝑖𝑎𝑖𝑠 1
𝑖 =1+𝛼
∴ 𝛼 = 𝑖 − 1 = 1,875 − 1 = 0,875 = 87,5%
Gabarito: 87,5%

AULA 10 – SOLUÇÕES 135


Prof. Thiago Cardoso

(ITA – 2017)
A adição de certa massa de etanol em água diminui a temperatura de congelamento do solvente em
18,6 °C. Sabendo que a constante crioscópica da água é de 1,86 °C·kg·mol–1, assinale a porcentagem em
massa do etanol nesta mistura.
a) 10,0 %.
b) 18,6 %.
c) 25,0 %.
d) 31,5 %.
e) 46,0 %.

Comentários
O aluno atento pode questionar se seria possível aplicar o Efeito Crioscópico ao etanol, que é um
soluto volátil. Em primeiro lugar, além de não brigar com o enunciado – muito importante, pois você
sempre será perdendo. Se dá para resolver a questão com as equações que você conhece, vá lá e resolva.
Além disso, nas proximidades da temperatura de congelamento da água, a pressão de vapor do
etanol é muito baixa, portanto, ele poderia sim ser considerado um soluto não volátil. O efeito da sua
volatilidade seria bem mais sentido no caso da ebulição da mistura.
Outro fato interessante é que o ponto de fusão do etanol é de -114,3 °C, muito mais baixa do que
a temperatura de congelamento da água. Por isso, nas proximidades da fusão da mistura, o etanol
permanece dissolvido no estado líquido, não havendo nenhum movimento de congelamento do próprio
soluto que pudesse influenciar nas suas propriedades coligativas.
O etanol é um soluto que não se dissocia, portanto, o seu fator de van’t Hoff é igual a 1. Sendo
assim, o efeito crioscópico pode ser calculado diretamente pela concentração molal.
Δ𝑇 = 𝐾𝐶 . 𝑊. 𝑖
18,6
18,6 = 1,86. 𝑊. 1 ∴ 𝑊 = = 10 𝑚𝑜𝑙/𝑘𝑔
1,86
Essa concentração molal significa que, para cada 1 kg de solvente, encontram-se 10 mol do soluto.
A massa correspondente de soluto pode ser calculada pelo produto do seu número de mols pela massa
molar.
𝑀𝐶2 𝐻6 𝑂 = 2.12 + 6.1 + 1.16 = 24 + 6 + 16 = 46 𝑔/𝑚𝑜𝑙
𝑚1 = 𝑛𝐶2𝐻6 𝑂 . 𝑀𝐶2𝐻6 𝑂 = 10.46 = 460 𝑔
Sendo assim, para cada 1 kg de solvente, encontram-se 460 g de soluto. Dessa forma, o título em
massa do etanol é dado por:
𝑚1 460 460
𝜏= = = ≅ 0,315 = 31,5%
𝑚1 + 𝑚2 1000 + 460 1460
Gabarito: D

AULA 10 – SOLUÇÕES 136


Prof. Thiago Cardoso

(ITA – 2017)
A pressão de vapor da água pura é de 23,8 torr a 25°C. São dissolvidos 10,0 g de cloreto de sódio em
100,0 g de água pura a 25°C. Assinale a opção que indica o valor do abaixamento da pressão de vapor
da solução, em torr.
a) 22,4
b) 11,2
c) 5,6
d) 2,8
e) 1,4

Comentários
De acordo com a Lei de Raoult, o abaixamento relativo da pressão de vapor do solvente é igual à
fração molar do soluto.
Δ𝑃
= 𝑥1
𝑃0
A fração molar do soluto deve ser obtida a partir dos números de mols do soluto e do solvente,
que podem ser calculados dividindo-se a massa informada pela massa molar.
𝑀𝑁𝑎𝐶𝑙 = 1.23 + 1.35,5 = 58,5 𝑔/𝑚𝑜𝑙
𝑚𝑁𝑎𝐶𝑙 10
𝑛𝑁𝑎𝐶𝑙 = = = 0,17 𝑚𝑜𝑙
𝑀𝑁𝑎𝐶𝑙 58,5
É importante, observar, no entanto, que o cloreto de sódio se dissocia completamente em água
liberando dois íons Na+ e Cl– para cada fórmula dissolvida. Sendo assim, o número de íons dissolvidos é
igual ao dobro.
𝑛1 = 2.0,17 = 0,34 𝑚𝑜𝑙
Façamos o mesmo procedimento para calcular o número de mols de água.
𝑀𝐻2 𝑂 = 2.1 + 1.16 = 18 𝑔/𝑚𝑜𝑙
𝑚𝐻2 𝑂 100
𝑛2 = 𝑛𝐻2 𝑂 = = = 5,56 𝑚𝑜𝑙
𝑀𝐻2 𝑂 18
De posse dos números de mols do soluto e do solvente, podemos calcular a fração molar desejada
do soluto.
𝑛1 0,34 0,34 3,4
𝑥1 = = = =
𝑛1 + 𝑛2 0,34 + 5,56 5,90 59
Não vamos nos precipitar em fazer essa conta, pois sabemos que é muito comum que os
enunciados deem números trabalhados, de modo a conseguir cortar. Voltando à expressão da Lei de
Raoult.
Δ𝑃 3,4
= 𝑥1 ∴ Δ𝑃 = 𝑃0 . 𝑥1 = 23,8. = 0,4.3,4 = 1,37 ≅ 1,4
𝑃0 59

Gabarito: E

AULA 10 – SOLUÇÕES 137


Prof. Thiago Cardoso

(ITA – 2017)
A reação do mercúrio metálico com excesso de ácido sulfúrico concentrado a quente produz um gás
mais denso do que o ar. Dois terços deste gás são absorvidos e reagem completamente com uma
solução aquosa de hidróxido de sódio, formando 12,6 g de um sal. A solução de ácido sulfúrico utilizada
tem massa específica igual a 1,75 g·cm–3 e concentração de 80 % em massa. Assinale a alternativa que
apresenta o volume consumido da solução de ácido sulfúrico, em cm 3.
a) 11
b) 21
c) 31
d) 41
e) 51

Comentários
O ácido sulfúrico (H2SO4) é um poderoso agente oxidante que é capaz de oxidar o mercúrio
metálico a Hg2+. Poderíamos até imaginar em um primeiro momento a produção do óxido de mercúrio
(HgO).
[𝑂]
𝐻𝑔 → 𝐻𝑔𝑂
Porém, o óxido de mercúrio reagiria com o excesso de ácido sulfúrico, produzindo sulfato de
mercúrio (HgSO4). Enquanto o mercúrio se oxida, o ácido sulfúrico se reduz a dióxido de enxofre (SO2),
que é o produto mais comum do enxofre nas suas reações.
𝐻𝑔 + 𝐻2 𝑆𝑂4 → 𝐻𝑔𝑆𝑂4 + 𝑆𝑂2
Podemos notar que falta uma espécie química contendo hidrogênio para que os hidrogênios do
ácido sulfúrico sejam alocados no produto.

𝐻𝑔 + 𝐻2 𝑆𝑂4 → 𝐻𝑔𝑆𝑂4 + 𝑆𝑂2 + 𝐻2 𝑂

O dióxido de enxofre (SO2) é o gás que reage com o hidróxido de sódio, de acordo com a seguinte
reação ácido-base. Nas reações ácido-base, não ocorre oxirredução, portanto, o sal formado é o sulfito
de sódio (Na2SO3), em que tanto o sódio como o enxofre conservam o seu número de oxidação. Outra
forma de lembrar do sal é lembrando-se que SO2 reage com água formando ácido sulfuroso (H2O + SO2 →
H2SO3)
𝑆𝑂2 + 𝑁𝑎𝑂𝐻 → 𝑁𝑎2 𝑆𝑂3 + 𝐻2 𝑂
Como o H2SO3 é um diácido, precisamos de duas fórmulas de NaOH para a reação.

𝑆𝑂2 + 2 𝑁𝑎𝑂𝐻 → 𝑁𝑎2 𝑆𝑂3 + 𝐻2 𝑂

Podemos calcular o número de mols de sulfito de sódio produzido, dividindo-se a massa produzida
pela massa molar do sal.
𝑚𝑁𝑎2 𝑆𝑂3
𝑛𝑁𝑎2 𝑆𝑂3 =
𝑀𝑁𝑎2 𝑆𝑂3

AULA 10 – SOLUÇÕES 138


Prof. Thiago Cardoso

A massa molar do sal é calculada somando-se as massas de cada um de seus elementos,


lembrando-nos que Na = 23 g/mol, S = 32 g/mol, O = 16 g/mol.
𝑀𝑁𝑎2 𝑆𝑂3 = 2.23 + 1.32 + 3.16 = 46 + 32 + 48 = 126 𝑔/𝑚𝑜𝑙
Note que é bastante comum as questões de prova produzirem números que sejam fáceis de cortar.
Façamos o cálculo do número de mols de sulfito de sódio (Na2SO3) produzidos.
𝑚𝑁𝑎2 𝑆𝑂3 12,6
𝑛𝑁𝑎2𝑆𝑂3 = = = 0,1 𝑚𝑜𝑙
𝑀𝑁𝑎2 𝑆𝑂3 126
Podemos calcular, agora, o número de mols de SO2 utilizados nessa reação pela proporção
estequiométrica. Note que os coeficientes são 1 SO2 e 1 Na2SO3.
𝑛𝑆𝑂2 𝑛𝑁𝑎2 𝑆𝑂3 0,1
= ∴ 𝑛𝑆𝑂2 = = 0,1 𝑚𝑜𝑙
1 1 1
Note que o enunciado falou que apenas 2/3 do SO2 foi absorvido e reagiu com o hidróxido de
sódio. Portanto, dois terços do número de mols produzidos de SO2 na etapa anterior é igual a 0,1 mol.
2 0,1.3
𝑛𝑆𝑂2 = 0,1 ∴ 𝑛𝑆𝑂2 = = 0,15 𝑚𝑜𝑙
3 2
Agora, vamos buscar a massa de H2SO4 que reagiu com o mercúrio (Hg). Para isso, precisamos
balancear a equação de oxidação do mercúrio pelo H2SO4 pelo Método da Oxirredução.

O número de elétrons perdidos pelo mercúrio (Hg) deve ser igual ao número de elétrons ganhos
pelo enxofre (S). Vale notar que somente o enxofre presente em d SO2 ganha 2 elétrons. Nem todos os
elétrons presentes em b H2SO4 sofre redução, porque parte dessas moléculas reage simplesmente como
ácido base formando o sulfato de mercúrio (II) – HgSO4.
𝑎. 2 = 2. 𝑑 ∴ 𝑎 = 𝑑
Podemos fazer a = 1 e d = 1.
1 𝐻𝑔 + 𝐻2 𝑆𝑂4 → 𝐻𝑔𝑆𝑂4 + 1 𝑆𝑂2 + 𝐻2 𝑂
Podemos balancear o mercúrio. Para isso, precisamos do coeficiente 1 HgSO4.
1 𝐻𝑔 + 𝐻2 𝑆𝑂4 → 1 𝐻𝑔𝑆𝑂4 + 1 𝑆𝑂2 + 𝐻2 𝑂
Podemos balancear agora o enxofre. Temos dois átomos no lado dos produtos, portanto,
precisamos de dois átomos no lado dos reagentes.
1 𝐻𝑔 + 2 𝐻2 𝑆𝑂4 → 1 𝐻𝑔𝑆𝑂4 + 1 𝑆𝑂2 + 𝐻2 𝑂

AULA 10 – SOLUÇÕES 139


Prof. Thiago Cardoso

Por fim, podemos balancear os hidrogênios colocando o coeficiente 2 H2O.


1 𝐻𝑔 + 2 𝐻2 𝑆𝑂4 → 1 𝐻𝑔𝑆𝑂4 + 1 𝑆𝑂2 + 2 𝐻2 𝑂
Agora, vamos calcular o número de mols de ácido sulfúrico (H2SO4) de acordo com a proporção
estequiométrica.
𝑛𝐻2 𝑆𝑂4 𝑛𝑆𝑂2 𝑛𝐻2 𝑆𝑂4 0,15
= ∴ = ∴ 𝑛𝐻2 𝑆𝑂4 = 2.0,15 = 0,30 𝑚𝑜𝑙
2 1 2 1
Precisamos, agora, calcular a massa molar do ácido sulfúrico.
𝑀𝐻2 𝑆𝑂4 = 2.1 + 1.32 + 4.16 = 2 + 32 + 64 = 98 𝑔/𝑚𝑜𝑙
A massa utilizada de ácido sulfúrico é igual ao produto entre o número de mols consumidos e a
massa molar do ácido.
𝑚𝐻2 𝑆𝑂4 = 𝑛𝐻2 𝑆𝑂4 . 𝑀𝐻2 𝑆𝑂4 = 0,30.98 = 29,4 𝑔
Como a solução tem 80% em massa de H2SO4, podemos calcular a massa de solução utilizada.
29,4
0,80. 𝑚𝑠𝑜𝑙𝑢çã𝑜 = 29,4 ∴ 𝑚 = = 36,75 𝑔
0,80
Como a densidade da solução é 1,75 g/cm³, podemos utilizar a definição de densidade para
calcular o volume da solução.
𝑚 𝑚 36,75
𝑑= ∴𝑉= =
𝑉 𝑑 1,75
Podemos multiplicar por 4 o numerador o denominador para facilitar as contas.
36,75.4 147
𝑉= = = 21 𝑐𝑚³
1,75.4 7

Gabarito: B

(ITA – 2017)
Considere duas soluções, X e Y, de um mesmo soluto genérico. A solução X tem 49% em massa do soluto,
enquanto a solução Y possui 8% em massa do mesmo soluto. Quer-se obter uma terceira solução, que
tenha 20% em massa deste soluto, a partir da mistura de um volume VX da solução X com um volume
VY da solução Y. Considerando que todas as soluções envolvidas exibem comportamento ideal, assinale
a opção que apresenta a razão VX/VY CORRETA.
a) 12/29.
b) 29/12.
c) 19/12.
d) 12/19.
e) 8/49.

Comentários
Utilizaremos os índices 1 para o soluto, 2 para o solvente e sem índice para a solução. A solução X
tem 49% em massa do soluto e a solução Y tem 8% em massa do soluto. Portanto, podemos dizer que:

AULA 10 – SOLUÇÕES 140


Prof. Thiago Cardoso

𝑚1𝑋 = 0,49. 𝑚 𝑋
𝑚1𝑌 = 0,08. 𝑚𝑌
A massa de soluto na solução final é a soma.
𝑚1 = 𝑚1𝑋 + 𝑚1𝑌
𝑚1 = 0,49. 𝑚 𝑋 + 0,08. 𝑚𝑌
A concentração do soluto na terceira solução é 20% em massa. A massa da terceira solução é igual
à soma das massas das duas primeiras, portanto, podemos escrever que:
𝑚1 = 0,20. 𝑚3 = 0,20. (𝑚 𝑋 + 𝑚𝑌 )
Igualando as duas equações, temos:
0,20. (𝑚 𝑋 + 𝑚𝑌 ) = 0,49. 𝑚 𝑋 + 0,08. 𝑚𝑌
Supondo que as densidades das soluções X e Y sejam iguais, podemos dividir pela densidade.
𝑚 𝑋 𝑚𝑌 𝑚𝑋 𝑚𝑌
0,20. ( + ) = 0,49. + 0,08.
𝑑 𝑑 𝑑 𝑑
A razão da massa pela densidade é igual ao volume das soluções.
0,20. (𝑉𝑋 + 𝑉𝑌 ) = 0,49. 𝑉𝑋 + 0,08. 𝑉𝑌
Como queremos a razão VX/VY, podemos dividir a equação por VY.
𝑉𝑋 𝑉𝑌 𝑉𝑋 𝑉𝑌
0,20. ( + ) = 0,49. + 0,08.
𝑉𝑌 𝑉𝑌 𝑉𝑌 𝑉𝑌
Vamos chamar de r a razão pedida. Ou seja, r = VX/VY.
0,20. (𝑟 + 1) = 0,49. 𝑟 + 0,08
Agora, basta fazer as contas.
0,20. 𝑟 + 0,20 = 0,49. 𝑟 + 0,08
0,20 − 0,08 = 0,49. 𝑟 − 0,20. 𝑟
0,12 = 0,29. 𝑟
0,12 12
∴𝑟= =
0,29 29

Gabarito: A

AULA 10 – SOLUÇÕES 141


Prof. Thiago Cardoso

(IME – 2013)
Um erlenmeyer contém 10,0 mL de uma solução de ácido clorídrico, juntamente com algumas gotas de
uma solução de fenolftaleína. De uma bureta, foi-se gotejando uma solução 0,100 M de hidróxido de
sódio até o aparecimento de leve coloração rósea. Nesse momento, observou-se um consumo de 20,0
mL da solução alcalina. Pode-se afirmar que a concentração de HC na solução ácida original era de:

Dados:
Massas atômicas: H = 1,00 u, O = 16,0 u, Na = 23,0 u, Cl = 35,5 u

a) 3,65 x 10–3 g/cm3


b) 7,30 x 10–3 g/cm3
c) 4,00 x 10–3 g/cm3
d) 3,20 x 10–3 g/cm3
e) 2,00 x 10–3 g/cm3

Comentários
O aparecimento da leve coloração rosa é o indicativo da neutralização.
Para montarmos a Equação da Titulação, devemos observar que o hidróxido de sódio (NaOH) é
uma monobase, enquanto que o ácido clorídrico (HC) é um monoácido.
1. [𝑁𝑎𝑂𝐻]. 𝑉𝑁𝑎𝑂𝐻 = 1. [𝐻𝐶𝑙]. 𝑉𝐻𝐶𝑙
1.0,100.20 = 1. [𝐻𝐶𝑙]. 10
1.0,1.20
∴ [𝐻𝐶𝑙] = = 0,2 𝑚𝑜𝑙/𝐿
10
Podemos converter para a unidade de massa multiplicando a concentração molar pela massa
molar.
𝑀𝐻𝐶𝑙 = 1.1 + 1.35,5 = 36,5 𝑔/𝑚𝑜𝑙
𝐶 = 0,2.36,5 = 7,3 𝑔/𝐿
Agora, basta converter para a unidade g/L.
7,3 𝑔 7,3 𝑔 7,3 𝑔
𝐶= = = = 7,3.10−3 𝑔/𝑐𝑚³
1𝐿 1000 𝑚𝐿 1000 𝑐𝑚³
Gabarito: B

AULA 10 – SOLUÇÕES 142


Prof. Thiago Cardoso

(ITA – 2015)
Considere uma solução saturada do sal MX que é pouco solúvel em água destilada a 25 °C. Seja y a
condutância da água destilada é (y + 2,0.10-7) ohm-1 cm-1 a condutância da solução. Sabendo que as
condutividades iônicas molares dos íons M+ e X– são, respectivamente, 60 ohm-1cm² mol-1 e 40 ohm-
1cm² mol-1, determine a solubilidade do MX em água em mol dm -3.

Comentários
A condutividade da solução é igual à soma das condutividades de cada um dos íons. Denotemos
por 𝜎 a condutância e por 𝜎𝑀+ a condutividade molar.
𝜎 = 𝜎𝐻2 𝑂 + 𝜎𝑀+ + 𝜎𝑋 − = 𝜎̅𝑀+ [𝑀+ ] + 𝜎̅𝑋 − [𝑋 − ] = 60. 𝑆 + 40. 𝑆 = 100. 𝑆
𝑦 + 2,0.10−7 = 𝑦 + 100. 𝑆

2,0.10−7 = 100𝑆

2,0.10−7
∴𝑆= = 2.10−9 𝑚𝑜𝑙/𝑐𝑚³
100
Podemos converter para a unidade pedindo utilizando o fator de conversão 1 cm³ = 10-3 dm³.

2,0.10−7 2.10−9 𝑚𝑜𝑙


𝑆= = 2.10−9 𝑚𝑜𝑙/𝑐𝑚³ = = 2.10−6 𝑚𝑜𝑙/𝑑𝑚3
100 10−3 𝑑𝑚3

Gabarito: 2.10–6

(ITA SP/2013)
Nas condições ambientes, 0,500g de um resíduo sólido foi dissolvido completamente em
aproximadamente 13 mL de uma mistura dos ácidos nítrico e fluorídrico (HNO 3 : HF = 10 : 3). A solução
aquosa ácida obtida foi quantitativamente transferida para um balão volumétrico com capacidade de
250 mL e o volume do balão completado com água desmineralizada. A análise quantitativa dos íons de
ferro na solução do balão revelou que a quantidade de ferro nesta solução era igual a 40,0 mgL–1.
Respeitando o número de algarismos significativos, determine a quantidade de ferro (em % em massa)
presente no resíduo sólido. Mostre o raciocínio e os cálculos realizados para chegar à sua resposta.

Comentários
A massa de ferro pode ser obtida diretamente pela concentração em mg/L da solução. Basta
multiplicar a concentração o metal pelo volume da solução.
𝑚𝐹𝑒
𝐶= ∴ 𝑚𝐹𝑒 = 𝐶. 𝑉 = 40.0,25 = 10 𝑚𝑔
𝑉

AULA 10 – SOLUÇÕES 143


Prof. Thiago Cardoso

O percentual em massa do ferro no sólido original é igual ao título em massa do ferro, que pode
ser obtido como a razão entre a massa de ferro e a massa total do sólido.
𝑚𝐹𝑒 10 2
𝜏= = = = 2%
𝑚𝑠ó𝑙𝑖𝑑𝑜 500 100
Gabarito: 2%

(ITA SP/2014)
A pressão de vapor de uma solução ideal contendo um soluto não-volátil dissolvido é diretamente
proporcional à
a) fração molar do soluto.
b) fração molar do solvente.
c) pressão osmótica do soluto.
d) molaridade, em mol  L–1, do solvente.
e) molalidade, em mol  kg–1, do solvente.

Comentários
Em uma solução ideal contendo um soluto não-volátil, a pressão de vapor é totalmente devido ao
solvente. Ela é igual ao produto da pressão de vapor do solvente puro pela fração molar do solvente na
solução.
𝑃 = 𝑃0 𝑥2
Tome cuidado, porque a Lei de Raoult diz que o abaixamento da pressão de vapor é proporcional
à fração molar do soluto. O abaixamento, não a pressão de vapor em si como perguntou o enunciado.
Gabarito: B

(ITA – 2013)
Uma solução líquida constituída por dois componentes A e B e apresentando comportamento ideal,
conforme Lei de Rauolt, está em equilíbrio com seu vapor. Utilizando a notação:

xA e xB para as respectivas frações em mol para as respectivas frações em A e B na solução líquida,


pA e pB para as respectivas pressões de vapor de A e B no vapor em equilíbrio com a solução líquida, e
p 0A e p 0B para as respectivas pressões de vapor de A puro e B puro numa mesma temperatura,

assinale a opção que apresenta a relação CORRETA para a pressão de vapor de A (pA) em equilíbrio com
a solução líquida.

AULA 10 – SOLUÇÕES 144


Prof. Thiago Cardoso

a) pA = p 0A (1 – xA)
b) PA = p 0B (1 – xB)

c) pA = p 0B xA

d) pA = p 0A xA
e) pA = p 0B xB

Comentários
Em uma solução ideal, a pressão de vapor de cada componente é igual ao produto de sua pressão
de vapor quando puros pela sua fração molar real na solução.
𝑃𝐴 = 𝑃𝐴0 . 𝑥𝐴
𝑃𝐵 = 𝑃𝐵0 . 𝑥𝐵
Gabarito: D

(ITA – 2015)
Assinale a opção que apresenta os instrumentos de medição de volume mais indicados para a realização
de uma titulação.

a) Bureta e erlenmeyer
b) Proveta e erlenmeyer
c) Pipeta volumétrica e erlenmeyer
d) Proveta e béquer
e) Pipeta volumétrica e béquer

Comentários
Suponha que desejamos titular uma amostra de 10 mL de solução de ácido acético com uma
solução 0,1 mol/L de hidróxido de sódio (NaOH). Como não sabemos a concentração da solução de ácido
acético, precisamos de instrumentos de medição de volume graduados, porque não sabemos prevíamos
quantos mL de solução de hidróxido de sódio serão utilizados.
É até possível utilizar uma pipeta, porém, não pode ser a volumétrica, mas sim a pipeta graduada.
A proveta também não pode ser utilizada, porque ela não é graduada. Com isso, eliminamos as letras de
B a E.
Podemos utilizar sim a bureta. O Erlenmeyer sempre poder ser utilizado para processar a reação,
pois é bastante útil para misturar os reagentes. Para a medição de volume, é necessário que seja um
Erlenmeyer graduado. Vale notar que nem todos são, mas é bastante comum encontrar graduação nesse
tipo de instrumento.

Gabarito: A

AULA 10 – SOLUÇÕES 145


Prof. Thiago Cardoso

(ITA – 2011)
A solução aquosa 6% em massa de água oxigenada (H2O2) é geralmente empregada como agente
branqueador para tecidos e cabelos. Pode-se afirmar que a concentração aproximada dessa solução
aquosa, expressa em volumes, é
a) 24.
b) 20.
c) 12.
d) 10.
e) 6.
Obs.: Considere o volume molar de gases nas CNTP é igual a 22,4 L/mol.

Comentários
A reação característica da água oxigenada é:
2𝐻2 𝑂2 (𝑙) → 2𝐻2 𝑂 (𝑙) + 𝑂2 (𝑔)
Consideremos o volume inicial de 1 litro de solução de água oxigenada. Podemos calcular a massa
de peróxido de hidrogênio (H2O2) presente nessa solução, considerando que ela tem a massa de 1000 g e
que o H2O2 corresponde a uma fração em massa de 6% da solução.
𝑚𝐻2𝑂2 = 0,06.1000 = 60 𝑔
Podemos, agora, calcular o número de mols de H2O2. Para isso, precisamos primeiramente calcular
a massa molar da substância.
𝑀𝐻2 𝑂2 = 2.1 + 2.16 = 2 + 32 = 34 𝑔/𝑚𝑜𝑙
Agora, basta dividir a massa de peróxido pela massa molar.
𝑚𝐻2 𝑂2 60
𝑛𝐻2 𝑂2 = = = 1,765 𝑚𝑜𝑙
𝑀𝐻2 𝑂2 34
O número de mols de oxigênio que podem ser liberados a partir dessa solução pode ser calculado
pela proporção estequiométrica. Os coeficientes do peróxido de hidrogênio e do oxigênio são 2 H 2O2 e 1
O2.
𝑛𝐻2 𝑂2 𝑛𝑂2 1,765
= ∴ 𝑛𝑂2 = = 0,88 𝑚𝑜𝑙
2 1 2
O volume obtido de oxigênio pode ser obtido multiplicando-se o número de mols de oxigênio pelo
volume de 1 mol que é 22,4 L.
𝑉𝑂2 = 0,88.22,4 = 19,8 𝐿
O número de volumes da água oxigenada pode ser obtido como a razão entre o volume de gás
oxigênio obtido pelo volume de solução empregado.
𝑉𝑂2 19,8
𝑥= = = 19,8 𝑣𝑜𝑙𝑢𝑚𝑒𝑠
𝑉𝑠𝑜𝑙𝑢çã𝑜 1
Gabarito: B

AULA 10 – SOLUÇÕES 146


Prof. Thiago Cardoso

(ITA – 2011)
A 25 °C, as massas específicas do etanol e da água, ambos puros, são 0,8 g cm –3 e 1,0 g cm–3,
respectivamente. Adicionando 72 g de água pura a 928 g de etanol puro, obteve-se uma solução com
1208 cm3 de volume.

Assinale a opção que expressa a concentração desta solução em graus Gay-Lussac (°GL).

a) 98
b) 96
c) 94
d) 93
e) 72

Comentários
Devemos nos lembrar que a concentração em graus Gay-Lussac (°GL) corresponde ao título em
volume de etanol.
Portanto, devemos calcular os volumes de água pura e de etanol puro utilizados.
𝑚𝐻2𝑂 72
𝑉𝐻2𝑂 = = = 72 𝑐𝑚³
𝑑𝐻2 𝑂 1,0
𝑚𝑒𝑡𝑎𝑛𝑜𝑙 928
𝑉𝑒𝑡𝑎𝑛𝑜𝑙 = = = 1160 𝑐𝑚³
𝑑𝑒𝑡𝑎𝑛𝑜𝑙 0,8
Questão bastante interessante. A dissolução do etanol em água ocorre com uma leve redução de
volume. A soma dos volumes das soluções empregadas é 1232 cm³, enquanto que o volume total da
solução é 1208 cm³.
O título em volume do etanol deve ser obtido como a razão entre o volume de etanol e o volume
da solução.
𝑉𝑒𝑡𝑎𝑛𝑜𝑙 1160
𝜏= = ≅ 0,96 = 96%
𝑉𝑠𝑜𝑙𝑢çã𝑜 1208
Vale ressaltar que o título em volume da água também deve ser obtido dessa forma e que,
portanto, a soma dos títulos em volume não é necessariamente igual a 100%.
Gabarito: B

(Estratégia Militares – TFC - Inédita)


Considere uma solução de um soluto X em um solvente Y. A soma dos títulos do soluto e do solvente
em número de mols, em massa e em volume devem ser necessariamente iguais a 100%? Justifique a
sua resposta.

AULA 10 – SOLUÇÕES 147


Prof. Thiago Cardoso

Comentários
Os títulos serão iguais a 100% quando forem válidas as relações.
𝑛 = 𝑛1 + 𝑛2
𝑚 = 𝑚1 + 𝑚2
𝑉 = 𝑉1 + 𝑉2
A primeira relação é sempre válida. O número de mols presentes na solução é sempre igual à soma
do número de mols dos reagentes com o número de mols dos produtos. Logo, a soma dos títulos em
número de mols do soluto e do solvente é igual a 100%.
A segunda relação é considerada válida, devido à Lei da Conservação das Massas de Lavoisier,
quando se despreza os efeitos relativísticos. Fora do nível de processos radioativos, eles são muito
pequenos, portanto, podemos considerar que a massa em uma dissolução se conserva. Portanto,
podemos considerar que é aproximadamente válido que a soma dos títulos em massa do solvente e do
soluto serão iguais a 100%.
Por fim, o volume de uma solução não se conserva. Pelo contrário, é bastante comum que uma
solução seja formada com redução e, em alguns casos, expansão de volume. Portanto, não é possível
afirmar que a soma dos títulos em volume do solvente e do soluto serão iguais a 100%.
Gabarito: sim; sim; não

(ITA – 2008)
Qual das substâncias abaixo não é empregada na fabricação da pólvora negra?
a) trinitrotolueno
b) enxofre
c) carvão
d) nitrato de sódio
e) nitrato de potássio

Comentários
A pólvora negra é composta basicamente por: nitratos de sódio e potássio, carvão e enxofre. O
trinitrotolueno (TNT) é outro tipo de explosivo.
Gabarito: A

AULA 10 – SOLUÇÕES 148


Prof. Thiago Cardoso

(IME – 2008)
Dispõe-se de uma mistura sulfonítrica de composição mássica igual a 60% de H2SO4, 11,2% de HNO3 e
28,8% de H2O. A 1000 kg desta mistura são adicionados 100 kg de solução de HNO3 88% (m/m) e 200 kg
de H2SO4 60% (m/m). Indique a composição mássica da mistura sulfonítrica final.
a) 55,4% de H2SO4; 15,4% de HNO3; 29,2% de H2O.
b) 59,6% de H2SO4; 16,6% de HNO3; 23,8% de H2O.
c) 59,0% de H2SO4; 16,4% de HNO3; 24,6% de H2O.
d) 55,9% de H2SO4; 15,5% de HNO3; 28,6% de H2O.
e) 64,3% de H2SO4; 15,1% de HNO3; 20,6% de H2O.

Comentários
Calcularemos a composição de cada uma das soluções. Primeiramente, vamos para a primeira
mistura, que é a mistura sulfonítrica 60% em H2SO4, 11,2% em HNO3 e 28,8% em H2O.
𝑚1𝐻2 𝑆𝑂4 = 0,60.1000 = 600 𝑘𝑔
𝑚1𝐻𝑁𝑂3 = 0,112.1000 = 112 𝑘𝑔
𝑚1𝐻2 𝑂 = 0,288.1000 = 288 𝑘𝑔
Para a segunda solução, quando dizemos uma solução de HNO3 88% em massa, devemos supor
que o restante (12%) é formado por água, que é o solvente universal.
2
𝑚𝐻𝑁𝑂3
= 0,88.100 = 88 𝑘𝑔
2
𝑚𝐻 2𝑂
= 0,12.1000 = 12 𝑘𝑔
Para a terceira solução, que é 60% em massa de H2SO4, devemos supor também que os demais
40% são formados por água.
3
𝑚𝐻 2 𝑆𝑂4
= 0,60.200 = 120 𝑘𝑔
3
𝑚𝐻 2𝑂
= 0,40.200 = 80 𝑘𝑔
Montemos o diagrama com as massas de cada um dos componentes nas três soluções e a mistura
formada.

AULA 10 – SOLUÇÕES 149


Prof. Thiago Cardoso

A massa de cada componente na solução final é igual à soma das massas em cada uma das soluções
iniciais. Ou seja, devemos somar as massas de água nas três soluções chegando a 380 kg de água no total.
Para o ácido sulfúrico, somamos as soluções 1 e 3, chegando a 720 kg (não tem ácido sulfúrico na solução
2). E, da mesma forma para o ácido nítrico, somamos as soluções 1 e 2, cegando a 200 kg (não tem ácido
nítrico na solução 3).
Sendo assim, chegamos a uma solução final de 1300 kg, formada por 720 kg de H 2SO4, 200 kg de
HNO3 e 380 kg de H2O. Agora, vamos calcular as frações mássicas de cada componente.
𝑚𝐻2𝑆𝑂4 720
𝜏𝐻2 𝑆𝑂4 = = = 0,554 = 55,4%
𝑚 1300
𝑚𝐻𝑁𝑂3 88
𝜏𝐻𝑁𝑂3 = = = 0,068 = 6,8%
𝑚 1300
𝑚𝐻2𝑂 380
𝜏𝐻2 𝑂 = = = 0,292 = 29,2%
𝑚 1300
Sendo assim, as frações mássicas são: 55,4% de H2SO4; 6,8% de HNO3 e 29,2% de H2O.
Gabarito: A

(IME RJ/2007)
Oleum, ou ácido sulfúrico fumegante, é obtido através da absorção do trióxido de enxofre por ácido
sulfúrico. Ao se misturar oleum com água obtém-se ácido sulfúrico concentrado. Supondo que uma
indústria tenha comprado 1.000 kg de oleum com concentração em peso de trióxido de enxofre de 20%
e de ácido sulfúrico de 80%, calcule a quantidade de água que deve ser adicionada para que seja obtido
ácido sulfúrico com concentração de 95% em peso.
Dados:
Massas atômicas (u.m.a): S = 32; O = 16; H = 1
a) 42 kg
b) 300 kg
c) 100 kg
d) 45 kg
e) 104,5 kg

Comentários
Questão bem interessante. O oleum em questão é composto por 200 kg de SO3 e 800 kg de H2SO4.
Queremos adicionar água para formar uma solução 95% em massa de H2SO4. Os restantes 5%
devem ser água, que é o solvente universal. Muito cuidado, pois muitos alunos tendem a pensar que seria
5% de SO3. Mas, quando o enunciado não diz nada sobre o solvente, subentende-se que é a água.
A equação de conversão balanceada do trióxido de enxofre (SO3) em ácido sulfúrico (H2SO4) pela
adição de água (H2O) é bem simples.
𝑆𝑂3 (𝑔) + 𝐻2 𝑂 (𝑙) → 𝐻2 𝑆𝑂4 (𝑙)
Vamos calcular as massas molares dos compostos envolvidos.
𝑀𝑆𝑂3 = 1.32 + 3.16 = 32 + 48 = 80 𝑔/𝑚𝑜𝑙

AULA 10 – SOLUÇÕES 150


Prof. Thiago Cardoso

𝑀𝐻2 𝑂 = 2.1 + 1.16 = 2 + 16 = 18 𝑔/𝑚𝑜𝑙


𝑀𝐻2 𝑆𝑂4 = 2.1 + 1.32 + 4.16 = 2 + 32 + 64 = 98 𝑔/𝑚𝑜𝑙
Agora, montemos a tabela estequiométrica com as proporções de massa entre os participantes da
reação.

𝑺𝑶𝟑 (𝒈) + 𝑯𝟐 𝑶 (𝒍) → 𝑯𝟐 𝑺𝑶𝟒 (𝒍)

Proporção em Massa 80 18 98

Com base na proporção estequiométrica, podemos calcular a massa de água necessária para
consumir completamente o trióxido de enxofre.
𝑚𝑆𝑂3 𝑚𝐻2 𝑂 18 18
= ∴ 𝑚𝐻2 𝑂 = 𝑚𝑆𝑂3 = . 200 = 45 𝑘𝑔
80 18 80 80
Com base na Lei da Conservação das Massas, podemos calcular a massa de ácido sulfúrico que foi
produzida.
𝑚𝐻2 𝑆𝑂4 = 𝑚𝑖𝑛𝑖𝑐𝑖𝑎𝑙 + 𝑚𝐻2 𝑂 = 1000 + 45 = 1045 𝑘𝑔
No entanto, como a solução final é 95% em massa de ácido sulfúrico, na realidade, deve-se
𝑚𝐻2 𝑆𝑂4 = 0,95. 𝑚
1045 = 0,95. 𝑚
1045
∴𝑚= = 1100 𝑘𝑔
0,95
Sendo assim, deve-se adicionar água, de modo que a massa total
𝑚𝑎𝑑𝑖𝑐𝑖𝑜𝑛𝑎𝑑𝑎 = 𝑚𝑓𝑖𝑛𝑎𝑙 − 𝑚𝑖𝑛𝑖𝑐𝑖𝑎𝑙 = 1100 − 1000 = 100 𝑘𝑔
Gabarito: C

(ITA – 2006)
São fornecidas as seguintes informações a respeito de titulação ácido-base:
I – A figura mostra as curvas de titulação de 30,0 mL de diferentes ácidos (I, II, III, IV e V), todos a 0,10
mol L–1, com uma solução aquosa 0,10 mol L–1 em NaOH.
II – O indicador fenolftaleína apresenta o intervalo de mudança de cor entre pH 8,0 a 10,0, e o indicador
vermelho de metila, entre pH 4,0 a 6,0.

AULA 10 – SOLUÇÕES 151


Prof. Thiago Cardoso

Considerando estas informações, é CORRETO afirmar que:


a) o indicador vermelho de metila é mais adequado que a fenolftaleína para ser utilizado na titulação
do ácido IV.
b) o indicador vermelho de metila é mais adequado que a fenolftaleína para ser utilizado na titulação
do ácido V.
c) o ácido III é mais forte que o ácido II.
d) os dois indicadores (fenolftaleína e vermelho de metila) são adequados para a titulação do ácido I.
e) os dois indicadores (fenolftaleína e vermelho de metila) são adequados para a titulação do ácido III.

Comentários
Em uma titulação, é importante que o indicador ácido base tenha cores diferentes antes do início
da titulação e ao seu final.
Vamos analisar as letras individualmente.
a) No caso do ácido IV, a titulação já se inicia com o pH igual a 6,0, situação em que ele já
completou a sua zona de viragem. Logo, o vermelho de metila praticamente não mudaria de
cor durante essa titulação. Portanto, a fenolftaleína é mais adequada. Afirmação incorreta.
b) No caso do ácido V, a titulação já se inicia em pH igual a 7,0, portanto, o vermelho de metila é
ainda mais inadequado. Afirmação incorreta.
c) A solução aquosa do ácido III é mais alcalina, porque tem pH superior. Portanto, a afirmação
está incorreta.
d) No caso do ácido I, a titulação se inicia em pH igual a 2 e termina em pH próximo a 10. Portanto,
abrange a zona de viragem de ambos os indicadores. Afirmação correta.
e) No caso do ácido III, a titulação se inicia em pH próximo a 5, no meio da zona de viragem do
vermelho de metila. Embora ainda seja possível, utilizar esse indicador, seria mais adequado
utilizá-lo nas titulações dos ácidos I e II, porque as titulações abrangeriam inteiramente a faixa
de 4 a 6, em que ocorre a viragem do vermelho de metila.

Gabarito: D

(ITA – 2005)
Esta tabela apresenta a solubilidade de algumas substâncias em água, a 15°C:

AULA 10 – SOLUÇÕES 152


Prof. Thiago Cardoso

Quando 50 mL de uma solução aquosa 0,10 moI L–1 em sulfato de zinco são misturados a 50 mL de uma
solução aquosa 0,010 moI L–1 em sulfito de sódio, à temperatura de 15°C, espera-se observar:
a) a formação de uma solução não saturada constituída pela mistura das duas substâncias.
b) a precipitação de um sólido constituído por sulfeto de zinco.
c) a precipitação de um sólido constituído por sulfito de zinco.
d) a precipitação de um sólido constituído por sulfato de zinco.
e) a precipitação de um sólido constituído por sulfeto de sódio.

Comentários
É importante perceber que ao se mistura sulfato de zinco (ZnSO4) e sulfito de sódio (Na2SO3),
forma-se um sal que é bem menos solúvel, que é o sulfito de zinco (ZnSO3). Devemos examinar, portanto,
se a quantidade de sal colocada é suficiente para provocar a precipitação ou não.

Com a diluição da mistura, houve a formação de 5.10–4mol de ZnSO3 em 100 mL de água. Podemos
calcular a massa corresponde desse sal. Para isso, basta multiplicar pela massa molar.

AULA 10 – SOLUÇÕES 153


Prof. Thiago Cardoso

É importante destacar que o número de mols formado de ZnSO3 é apenas 5.10–4 mol, porque não
existe sulfito suficiente em solução. Tem-se zinco em excesso.
𝑀𝑍𝑛𝑆𝑂3 = 1.65 + 1.32 + 3.16 = 65 + 32 + 48 = 145 𝑔/𝑚𝑜𝑙
A massa de sulfito de zinco presente na solução é, portanto, igual a:
𝑚𝑍𝑛𝑆𝑂3 = 𝑛𝑍𝑛𝑆𝑂3 . 𝑀𝑍𝑛𝑆𝑂3 = 5.10−4 . 145 = 725.10−4 𝑔 = 0,0725 𝑔 < 0,16 𝑔
Lembrando-nos que 100 mL de água corresponde a 100 g, temos que a massa dissolvida de sulfito
de zinco foi inferior a 0,16 g, que é o seu coeficiente de solubilidade. Portanto, não ocorre precipitação
do sal citado, mas sim a formação de uma solução ainda insaturada.

Gabarito: A

(ITA – 2004)
Deseja-se preparar 57 gramas de sulfato de alumínio[Al 2(SO4)3] a partir de alumínio sólido (Al)
praticamente puro, e ácido sulfúrico (H2SO4). O ácido sulfúrico disponível é uma solução aquosa 96 %
(m/m), com massa específica de 1,4gcm-1.
a) Qual a massa, em gramas, de alumínio necessária para preparar a quantidade de Al 2(SO4)3
especificada? Mostre os cálculos realizados.
b) Qual a massa, em gramas, de ácido sulfúrico necessária para preparar a quantidade de Al 2(SO4)3
especificada? Mostre os cálculos realizados.
c) Nas condições normais de temperatura e pressão (CNTP), qual é o volume, em litros, de gás formado
durante a preparação da quantidade de Al2(SO4)3 especificada? Mostre os cálculos realizados.
d) Caso a quantidade especificada de Al2(SO4)3 seja dissolvida em água acidulada, formando 1 L de
solução, qual a concentração de íons Al3+ e de íons SO42– existentes nesta solução?

Comentários
O ácido sulfúrico é um dos agentes oxidantes mais fortes da natureza, sendo capaz de oxidar
metais, como o alumínio. Em geral, ele se reduz a SO2.
𝐴𝑙 + 𝐻2 𝑆𝑂4 → 𝐴𝑙2 (𝑆𝑂4 )3 + 𝑆𝑂2 + 𝐻2 𝑂
A massa molar do sal pode ser calculada somando-se as massas molares dos elementos.
𝑀𝐴𝑙2 (𝑆𝑂4)3 = 2.27 + 3. (1.32 + 4.16) = 54 + 3.96 = 342 𝑔/𝑚𝑜𝑙
Podemos, agora, calcular o número de mols do sulfato de alumínio que foram produzidos.
𝑚 57 1
𝑛= = = 𝑚𝑜𝑙
𝑀 342 6
Para calcular o número de mols de alumínio e de ácido sulfúrico, é necessário balancear a equação
da reação, o que pode ser feito pelo Método da Oxirredução.

AULA 10 – SOLUÇÕES 154


Prof. Thiago Cardoso

É interessante observar que nem todos os átomos de enxofre do H2SO4 sofrem redução. Alguns
deles participam de uma reação ácido-base, formando sulfato de alumínio, portanto, permanecem o
número de oxidação inalterado.
3. 𝑎 = 2. 𝑑
Basta fazer a = 2 e d = 3.
2 𝐴𝑙 + 𝐻2 𝑆𝑂4 → 𝐴𝑙2 (𝑆𝑂4 )3 + 3 𝑆𝑂2 + 𝐻2 𝑂
Notamos que o alumínio já está balanceado, pois temos dois átomos de cada lado. Podemos
balancear agora o enxofre. Temos 6 átomos de enxofre nos produtos, logo, precisamos do coeficiente 6
H2SO4.
2 𝐴𝑙 + 6 𝐻2 𝑆𝑂4 → 1 𝐴𝑙2 (𝑆𝑂4 )3 + 3 𝑆𝑂2 + 𝐻2 𝑂
Para terminar o balanceamento, só precisamos balancear os hidrogênios. Temos 12 átomos no
lado dos reagentes, logo, precisamos de 6 H2O nos produtos.
2 𝐴𝑙 + 6 𝐻2 𝑆𝑂4 → 1 𝐴𝑙2 (𝑆𝑂4 )3 + 3 𝑆𝑂2 + 6𝐻2 𝑂
Podemos verificar que temos 24 átomos de oxigênio no lado dos reagentes e também temos 24
no lado dos produtos. Logo, a reação está balanceada.
Podemos calcular o número de mols de alumínio e de sulfato através da proporção
estequiométrica.
𝑛𝐴𝑙 𝑛𝐴𝑙2 (𝑆𝑂4)3 1 1
= ∴ 𝑛𝐴𝑙 = 2. 𝑛𝐴𝑙2 (𝑆𝑂4)3 = 2. = 𝑚𝑜𝑙
2 1 6 3
𝑛𝐻2 𝑆𝑂4 𝑛𝐴𝑙2 (𝑆𝑂4)3 1
= ∴ 𝑛𝐻2 𝑆𝑂4 = 6. 𝑛𝐴𝑙2 (𝑆𝑂4)3 = 6. = 1 𝑚𝑜𝑙
6 1 6
Podemos, portanto, calcular as massas (em gramas) do alumínio e do ácido sulfúrico utilizadas.
Basta, para isso, multiplicar o número de mols pela massa molar. As massas molares, por sua vez, podem
ser calculadas somando as massas de cada elemento.
𝑀𝐴𝑙 = 1.27 = 27 𝑔/𝑚𝑜𝑙
𝑀𝐻2 𝑆𝑂4 = 2.1 + 1.32 + 4.16 = 2 + 32 + 64 = 98 𝑔/𝑚𝑜𝑙
Podemos, agora, obter as massas em gramas de alumínio e de ácido sulfúrico utilizadas.
1
𝑚𝐴𝑙 = 𝑛𝐴𝑙 . 𝑀𝐴𝑙 = . 27 = 9 𝑔
3

AULA 10 – SOLUÇÕES 155


Prof. Thiago Cardoso

𝑚𝐻2 𝑆𝑂4 = 𝑛𝐻2 𝑆𝑂4 . 𝑀𝐻2 𝑆𝑂4 = 1.98 = 98 𝑔

O gás produzido na reação é o dióxido de enxofre (SO2).


𝑛𝑆𝑂3 𝑛𝐴𝑙2 (𝑆𝑂4)3 1 1
= ∴ 𝑛𝑆𝑂3 = 3. 𝑛𝐴𝑙2 (𝑆𝑂4)3 = 3. = 𝑚𝑜𝑙
3 1 6 2
O volume de gás pode ser obtido multiplicando-se o volume molar pela quantidade de mols.
1
𝑉𝑆𝑂2 = . 22,4 = 11,2 𝐿
2
Foi produzido 1/6 mol de sulfato de alumínio – Al2(SO4)3. A dissolução dessa quantidade de matéria
produzirá os íons Al3+ e SO43–.

Devido à estequiometria do sal, são liberados 1/3 mol de Al3+ e 1/2 mol de SO42–. Sendo assim, o
número de mols
𝑛 1/3
[𝐴𝑙 3+ ] = = = 0,333 𝑚𝑜𝑙/𝐿
𝑉 1
𝑛 1/2
[𝑆𝑂42− ] = = = 0,5 𝑚𝑜𝑙/𝐿
𝑉 1

Gabarito: a) 9 g; b) 98 g; c) 11,2 L; d) [Al3+] = 0,333 mol/L; SO42-= 0,50 mol/L

(ITA – 1999)
Para preparar 80 L de uma solução aquosa 12% (massa/massa) de KOH (massa específica da solução =
1,10 g/cm3) foram adicionados x litros de uma solução aquosa 44% (massa/massa) de KOH (massa
específica da solução = 1,50 g/cm3) e y litros de água deionizada (massa específica = 1,00 g/cm3). Os
valores de x e y são respectivamente:
a) 12 L e 68 L
b) 16 L e 64 L
c) 30 L e 50 L
d) 36 L e 44 L
e) 44 L e 36 L

Comentários
A massa de KOH na solução final pode ser obtida multiplicando-se o título pela massa total da
solução final. Essa massa pode ser obtida a partir da massa específica da solução, que foi fornecida.
𝑚
𝑑 = ∴ 𝑚 = 𝑑𝑉 = 1,10.80 = 88 𝑘𝑔
𝑉

AULA 10 – SOLUÇÕES 156


Prof. Thiago Cardoso

𝑚𝐾𝑂𝐻 = 0,12. 𝑚 = 0,12.88 𝑘𝑔


Deixaremos assim, pois veremos que essa conta será simplificada. Toda essa massa é proveniente
da solução mais concentrada (44%), tendo em vista que a outar solução é formada unicamente por água
destilada.
0,12.88
0,44. 𝑚𝑠𝑜𝑙𝑢çã𝑜 = 0,12.88 ∴ 𝑚𝑠𝑜𝑙𝑢çã𝑜 = = 24 𝑘𝑔
0,44
Dessa maneira, o volume dessa solução pode ser obtido a partir de sua massa específica, que é de
1,50 g/cm³.
𝑚 𝑚
𝑑= ∴𝑉=
𝑉 𝑑
24
𝑥= = 16 𝐿
1,5
O examinador considerou que o volume total da solução final é igual à soma dos volumes das duas
soluções componentes, podemos escrever que:
𝑥 + 𝑦 = 80
16 + 𝑦 = 80 ∴ 𝑦 = 80 − 16
∴ 𝑦 = 64 𝐿
Gabarito: B

(ITA – 1998)
Fazendo-se borbulhar gás cloro através de 1,0 litro de uma solução de hidróxido de sódio, verificou-se
ao final do experimento que todo hidróxido de sódio foi consumido, e que na solução resultante foram
formados 2,5 mols de cloreto de sódio. Considerando que o volume não foi alterado durante todo o
processo, e que na temperatura em questão tenha ocorrido apenas a reação correspondente à seguinte
equação química, não balanceada:

OH-(aq) + Cl2(g) → Cl-(aq) + ClO3-(aq) + H2O(L)

Qual deve ser a concentração inicial do hidróxido de sódio?


a) 6,0 mol/L
b) 5,0 mol/L
c) 3,0 mol/L
d) 2,5 mol/L
e) 2,0 mol/L

Comentários
A fórmula do cloreto de sódio é NaC. Portanto, em 2,5 mol de fórmulas de cloreto de sódio,
encontram-se exatamente 2,5 mol de cloret.o

AULA 10 – SOLUÇÕES 157


Prof. Thiago Cardoso

Ao se borbulhar o gás cloro na solução, podemos verificar a ocorrência de um


desproporcionamento da molécula de cloro.

Usando o fato de que o número de elétrons ganhos pelo íon cloreto (C–) é igual ao número de
elétrons perdidos pelo íon clorato (CO3–). No cloreto, cada átomo de cloro ganha 1 elétron, enquanto
que, no clorato, cada átomo de cloro perde 5 elétrons. Temos, portanto:
𝑏. 1 = 𝑐. 5
𝑏 = 5𝑐
Podemos fazer, portanto, c = 1 e b = 5.
𝐶𝑙2 (𝑔) + 𝑂𝐻 − (𝑎𝑞) → 5 𝐶𝑙 − (𝑎𝑞) + 1 𝐶𝑙𝑂3− (𝑎𝑞) + 𝐻2 𝑂 (𝑙)
Podemos observar que temos 6 átomos de cloro e 6 cargas negativas do lado dos produtos.
Portanto, precisamos de 3 Cl2 e de 6 OH–, respectivamente, para balancear.
3 𝐶𝑙2 (𝑔) + 6 𝑂𝐻 − (𝑎𝑞) → 5 𝐶𝑙 − (𝑎𝑞) + 1 𝐶𝑙𝑂3− (𝑎𝑞) + 𝐻2 𝑂 (𝑙)
Podemos, agora, balancear o hidrogênio, notando que temos 6 átomos no lado dos reagentes.
Precisamos, portanto, de 3 H2O para balancear esse elemento.
3 𝐶𝑙2 (𝑔) + 6 𝑂𝐻 − (𝑎𝑞) → 5 𝐶𝑙 − (𝑎𝑞) + 1 𝐶𝑙𝑂3− (𝑎𝑞) + 3 𝐻2 𝑂 (𝑙)
Usando a proporção estequiométrica, podemos obter o número de mols de hidróxido, já que
sabemos que o número de mols de cloreto é igual a 2,5 mol.
𝑛𝐶𝑙− 𝑛𝑂𝐻 − 6 6
= ∴ 𝑛𝑂𝐻 − = . 𝑛𝐶𝑙− = . 2,5 = 6.0,5 = 3 𝑚𝑜𝑙
5 6 5 5
A estequiometria do NaOH também é de 1:1, portanto, a solução que contém 3 mol de OH –
também conterá 3 mol de NaOH. Logo, a concentração dessa base na solução original é:
𝑛 3
[𝑁𝑎𝑂𝐻] = = = 3 𝑚𝑜𝑙/𝐿
𝑉 1
Gabarito: C

AULA 10 – SOLUÇÕES 158


Prof. Thiago Cardoso

(ITA – 2012 – adaptada)


Uma amostra de 210–2 g de um determinado composto orgânico é dissolvida em 300 mL de água a 25
°C, resultando numa solução de pressão osmótica 0,027 atm. Pode-se afirmar, então, que o composto
orgânico é o(a) :
a) ácido acético (C2H4O2).
b) etileno glicol (C2H6O2).
c) etanol (C2H6O).
d) uréia (CH4N2O).
e) tri-fluor-carbono (CHF3).

Comentários
A pressão osmótica é calculada em função da concentração molar do soluto.
𝜋 0,027
𝜋 = [𝑋]𝑅𝑇 ∴ [𝑋] = = = 0,0011 𝑚𝑜𝑙/𝐿 = 1,1.10−3 𝑚𝑜𝑙/𝐿
𝑅𝑇 0,082.298
A partir da concentração do soluto, podemos extrair o número de mols presentes na sua solução.
𝑛
[𝑋] = ∴ 𝑛 = [𝑋]. 𝑉 = 1,1.10−3 . 0,3 = 3,3.10−4 𝑚𝑜𝑙
𝑉
O enunciado forneceu a massa de 2.10 –2 g e acabamos de descobrir que isso corresponde à
quantidade de 3,3.10–4 mol. Com base nisso, podemos calcular a massa molar.
𝑚 𝑚 2.10−2
𝑛= ∴𝑀= = ≅ 0,6.10−2+4 = 60 𝑔/𝑚𝑜𝑙
𝑀 𝑛 3,3.10−4
Agora, basta procurar o soluto que possui a massa molar calculada acima.
A massa molar do ácido acético é:
𝑀𝐶2 𝐻4 𝑂2 = 2.12 + 4.1 + 2.16 = 24 + 4 + 32 = 60 𝑔/𝑚𝑜𝑙
Porém, o ácido acético sofre ionização parcial, portanto, nesse caso, haveria a incidência de um
fator de van’t Hoff na pressão osmótica, o que alteraria conta. Logo, esse ácido não pode ser a resposta.
𝑀𝐶2 𝐻6 𝑂2 = 2.12 + 6.1 + 2.16 = 24 + 6 + 32 = 62 𝑔/𝑚𝑜𝑙
Vamos calcular as massas molares restantes. Para o etanol:
𝑀𝐶2 𝐻6 𝑂 = 2.12 + 6.1 + 1.16 = 24 + 6 + 16 = 46 𝑔/𝑚𝑜𝑙
Para a uréia.
𝑀𝐶𝐻4 𝑁2𝑂 = 1.12 + 4.1 + 2.14 + 1.16 = 12 + 4 + 28 + 16 = 60 𝑔/𝑚𝑜𝑙
Por fim, o trifluorcarbono.
𝑀𝐶𝐻𝐹3 = 1.12 + 1.1 + 3.19 = 12 + 1 + 57 = 70 𝑔/𝑚𝑜𝑙
Como a ureia não se dissocia, ela é o composto orgânico tratado na questão.
Gabarito: D

AULA 10 – SOLUÇÕES 159


Prof. Thiago Cardoso

(ITA SP/2003)
O abaixamento da temperatura de congelamento da água numa solução aquosa com concentração
molal de soluto igual a 0,100 mol kg–1 é
0,55oC. Sabe-se que a constante crioscópica da água é igual a 1,86oC kg mol–1. Qual das opções abaixo
contém a fórmula molecular CORRETA do soluto?
a) [Ag(NH3)2]Cl
b) [Pt(NH3)4Cl2]Cl2
c) Na[Al(OH)4]
d) K3[Fe(CN)6 ]
e) K4[Fe(CN)6]

Comentários
Questão bastante complexa. O abaixamento da temperatura de congelamento da água é escrita
em função da concentração molal do soluto e do fator de van’t Hoff.
Δ𝑇𝐶 = 𝐾𝑐 𝑊𝑖
0,55 = 1,86.0,1. 𝑖
0,55
∴𝑖= = 2,96 ≅ 3
1,86.0,1
Precisamos, portanto, descobrir qual dos solutos produz o fator de van’t Hoff igual a 3. Para isso,
basta escrever as equações de dissolução. O fator de van’t Hoff é igual ao número de partículas produzidas
pelo composto iônico.
Nessa questão, é preciso atentar para o fato de que os íons complexos não se decompõem quando
misturados em água.
𝐻2 𝑂
[𝐴𝑔(𝑁𝐻3 )2 ]𝐶𝑙 → [𝐴𝑔(𝑁𝐻3 )2 ]+ (𝑎𝑞) + 𝐶𝑙 − (𝑎𝑞) ∴ 𝑖 = 2
𝐻2 𝑂
[𝑃𝑡(𝑁𝐻3 )4 𝐶𝑙2 ]𝐶𝑙2 → [𝑃𝑡(𝑁𝐻3 )4 𝐶𝑙2 ]2+ (𝑎𝑞) + 2 𝐶𝑙 − (𝑎𝑞) ∴ 𝑖 = 3
𝐻2 𝑂
𝑁𝑎[𝐴𝑙(𝑂𝐻)4 ] → 𝑁𝑎+ (𝑎𝑞) + [𝐴𝑙(𝑂𝐻)4 ]− (𝑎𝑞) ∴ 𝑖 = 2
𝐻2 𝑂
𝐾3 [𝐹𝑒(𝐶𝑁)6 ] → 3 𝐾 + (𝑎𝑞) + [𝐹𝑒(𝐶𝑁)6 ]3− (𝑎𝑞) ∴ 𝑖 = 4
𝐻2 𝑂
𝐾4 [𝐹𝑒(𝐶𝑁)6 ] → 4 𝐾 + (𝑎𝑞) + [𝐹𝑒(𝐶𝑁)6 ]4− (𝑎𝑞) ∴ 𝑖 = 5

Gabarito: B

AULA 10 – SOLUÇÕES 160


Prof. Thiago Cardoso

(ITA – 2005)
Dois frascos abertos, um contendo água pura líquida (frasco A) e o outro contendo o mesmo volume de
uma solução aquosa concentrada em sacarose (frasco B), são colocados em um recipiente que, a seguir,
é devidamente fechado. É CORRETO afirmar, então, que, decorrido um longo período de tempo,
a) os volumes dos líquidos nos frascos A e B não apresentam alterações visíveis.
b) o volume do líquido no frasco A aumenta, enquanto que o do frasco B diminui.
c) o volume do líquido no frasco A diminui, enquanto que o do frasco B aumenta.
d) o volume do líquido no frasco A permanece o mesmo, enquanto que o do frasco B diminui.
e) o volume do líquido no frasco A diminui, enquanto que o do frasco B permanece o mesmo.

Comentários
Ocorrerá osmose por via área, em que o frasco contendo água pura líquida (A) perderá água para
o frasco contendo a solução concentrada de sacarose. Portanto, o volume do frasco A diminui, enquanto
que o volume do frasco B aumenta.

Gabarito: C

(ITA – 2012 – adaptada)


Considere as seguintes misturas (soluto/solvente) na concentração de 10 % em mol de soluto:

I. acetona/clorofórmio
II. água/etanol
III. água/metanol
IV. benzeno/tolueno
V. n-hexano/n-heptano

Assinale a opção que apresenta a(s) mistura(s) para a(s) qual(is) a pressão de vapor do solvente na
mistura é aproximadamente igual à sua pressão de vapor quando puro multiplicada pela sua respectiva
fração molar.

a) Apenas I
b) Apenas I, II e III
c) Apenas II e III
d) Apenas IV e V
e) Apenas V

AULA 10 – SOLUÇÕES 161


Prof. Thiago Cardoso

Dica: Caso você não conheça as fórmulas estruturais dos compostos citados, por ainda não ter estudado
Química Orgânica, elas estão transcritas a seguir.

Comentários
A questão explora o conceito de solução ideal, que é aquela em que as interações intermoleculares
soluto-solvente são aproximadamente iguais às interações intermoleculares soluto-soluto e solvente-
solvente.
Para isso, as moléculas devem ser o mais parecidas possível.
No par I, as moléculas são bastante diferentes, sendo a acetona polar e o clorofórmio pouco polar.
Logo, esse par está bem distante de uma solução ideal.
Os pares IV e V são muito próximos de soluções ideais, pois mostram pares de moleculares
inteiramente apolares que diferem entre si apenas por um carbono.
As soluções da água com os álcoois (metanol e etanol) mostradas em II e III apresentam uma
mudança relativamente radical, em que um hidrogênio que forma pontes de hidrogênio é substituído por
um grupo apolar – CH3. Portanto, essa substituição afasta a solução da idealidade.
Convém observar que uma solução de metanol e etanol seria também um ótimo exemplo de
solução ideal.
Portanto, os melhores exemplos de solução ideal são os pares IV e V.

Gabarito: D

AULA 10 – SOLUÇÕES 162


Prof. Thiago Cardoso

(IME – 2010)
Calcule a massa de 1 L de uma solução aquosa de nitrato de zinco cuja concentração é expressa por
0,643 molar e por 0,653 molal.

Comentários
Em 1 litro de solução, encontramos 0,643 mol de nitrato de zinco, como evidenciado pela
concentração molar fornecida pelo enunciado.
Podemos, agora, calcular a massa do solvente em (kg) dessa solução. Pela concentração molal, em
1 kg de solvente encontramos 0,653 mol do soluto. Porém, como temos apenas 0,643 mol do soluto,
podemos calcular a massa de solvente presente, usando o fato de que ela é diretamente proporcional.

Montando a Regra de Três, segue que:


𝑚2 0,643
= = 0,9847 𝑘𝑔
1 0,653
Portanto, em 1 litro de solução, encontra-se 0,985 kg do soluto. Pode-se, agora, calcular a massa
presente do solvente. Primeiramente, precisamos calcular a massa molar do sal, cuja fórmula é Zn(NO3)2.
𝑀1 = 1.65 + 2. (1.14 + 3.16) = 65 + 2.62 = 189 𝑔/𝑚𝑜𝑙
Agora, a massa presente do soluto pode ser calculada multiplicando o número de mols presentes
em 1 litro de solução (0,643) pela massa molar do soluto.
𝑚1 = 𝑛1 . 𝑀1 = 0,643.189 = 121,5 𝑔 = 0,1215 𝑘𝑔
Por fim, a massa total da solução é igual à soma da massa do soluto com a massa do solvente.
𝑚 = 𝑚1 + 𝑚2 = 0,1215 + 0,9847 = 1,1062 𝑘𝑔
Gabarito: 1,1062 kg

AULA 10 – SOLUÇÕES 163


Prof. Thiago Cardoso

(ITA – 2007)
Dois béqueres, X e Y, contêm, respectivamente, volumes iguais de soluções aquosas: concentrada e
diluída de cloreto de sódio na mesma temperatura. Dois recipientes hermeticamente fechados,
mantidos à mesma temperatura constante, são interconectados por uma válvula, inicialmente fechada,
cada qual contendo um dos béqueres. Aberta a válvula, após o restabelecimento do equilíbrio químico,
verifica-se que a pressão de vapor nos dois recipientes é Pf .
Assinale a opção que indica, respectivamente, as comparações CORRETAS entre os volumes inicial (VXi)
e final (VXf ), da solução no béquer X e entre as pressões de vapor inicial (PY i) e final (Pf) no recipiente
que contém o béquer Y.
a) VXi < VXf e PYi = PYf
b) VXi < VXf e PYi > PYf
c) VXi < VXf e PYi < PYf
d) VXi > VXf e PYi > PYf
e) VXi > VXf e PYi < PYf

Comentários
Trata-se de uma situação semelhante à osmose, porém, ocorre por meio da fase vapor.

A solução menos concentrada (Y) perde moléculas de água. Com isso, o seu volume diminui. Com
a redução de volume, o seu soluto se torna mais concentrado, o que diminui a pressão de vapor da
solução. Logo, podemos dizer que PYi > PYf.
Por outro lado, a solução mais concentrada (X) ganha moléculas de água provenientes da fase de
vapor. Com isso, o seu volume aumenta, diminuindo a concentração do soluto. Logo, VX i < VXf.

Gabarito: B

AULA 10 – SOLUÇÕES 164


Prof. Thiago Cardoso

(ITA – 2007 – adaptada)


Prepara-se, a 25oC, uma solução por meio da mistura de 25 mL de n-pentano (C5H12) e 45 mL de n-
hexano (C6H14).

Dados:
massa específica do n-pentano = 0,63 g/mL;
massa específica do n-hexano = 0,66 g/mL;
pressão de vapor do n-pentano = 511 torr;
pressão de vapor do n-hexano = 150 torr.

Determine os seguintes valores, mostrando os cálculos efetuados:


a) Fração molar do n-pentano na solução.
b) Pressão de vapor da solução.
c) Fração molar do n-pentano no vapor em equilíbrio com a solução.

Comentários
Podemos calcular as massas correspondentes de n-pentano e n-hexano por meio das massas
específicas fornecidas.
𝑚
𝑑 = ∴ 𝑚 = 𝑑𝑉
𝑉
Basta substituir os valores fornecidos de volume e massa específica para o pentano e para o
hexano.
𝑚𝑝𝑒𝑛𝑡𝑎𝑛𝑜 = 0,64.25 = 16 𝑔
𝑚ℎ𝑒𝑥𝑎𝑛𝑜 = 0,66.45 = 29,7 𝑔
Podemos calcular as massas molares com base nas fórmulas químicas (C5H12 e C6H14).
𝑀𝑝𝑒𝑛𝑡𝑎𝑛𝑜 = 5.12 + 12.1 = 60 + 12 = 72 𝑔/𝑚𝑜𝑙
𝑀ℎ𝑒𝑥𝑎𝑛𝑜 = 6.12 + 14.1 = 72 + 14 = 84 𝑔/𝑚𝑜𝑙
Com base nas massas da amostra e na massa molar, podemos calcular o número de mols de cada
uma das espécies envolvidas:
𝑚𝑝𝑒𝑛𝑡𝑎𝑛𝑜 16 2
𝑛𝑝𝑒𝑛𝑡𝑎𝑛𝑜 = = = ≅ 0,22
𝑀𝑝𝑒𝑛𝑡𝑎𝑛𝑜 72 9
𝑚ℎ𝑒𝑥𝑎𝑛𝑜 29,7
𝑛ℎ𝑒𝑥𝑎𝑛𝑜 = = ≅ 0,35
𝑀ℎ𝑒𝑥𝑎𝑛𝑜 84

Sendo assim, a fração molar do n-pentano na solução é:


𝑛𝑝𝑒𝑛𝑡𝑎𝑛𝑜 0,22 0,22
𝑥𝑝𝑒𝑛𝑡𝑎𝑛𝑜 = = = ≅ 0,39 = 39%
𝑛𝑝𝑒𝑛𝑡𝑎𝑛𝑜 + 𝑛ℎ𝑒𝑥𝑎𝑛𝑜 0,22 + 0,35 0,57

AULA 10 – SOLUÇÕES 165


Prof. Thiago Cardoso

Considerando a solução ideal, podemos calcular a pressão de vapor da solução. A pressão de vapor
de cada componente é igual ao produto da sua fração molar pela sua pressão de vapor quando puro.

𝑃 = 0,39.511 + 0,61.150 = 199 + 91 = 290


Sendo assim, na fase gasosa, podemos estabelecer que a fração molar do n-pentano é igual à razão
entre a pressão parcial e a pressão total da mistura.
𝑓𝑎𝑠𝑒 𝑔𝑎𝑠𝑜𝑠𝑎 199
𝑥𝑝𝑒𝑛𝑡𝑎𝑛𝑜 = = 0,65 = 65%
290
Como o pentano é mais volátil que o hexano, a sua fração molar na fase gasosa é bem superior ao
que se registra na fase líquida.
Gabarito: a) 39% b) 290,8 torr; c) 65%

(ITA – 2005)
Considere as afirmações abaixo, todas relativas à pressão de 1 atm:
I. A temperatura de fusão do ácido benzóico puro é 122°C, enquanto que a da água pura é 0°C.
II. A temperatura de ebulição de uma solução aquosa 1,00 moI L–1 de sulfato de cobre é maior do que a
de uma solução aquosa 0,10 mol L–1 deste mesmo sal.
III. A temperatura de ebulição de uma solução aquosa saturada em cloreto de sódio é maior do que a
da água pura.
IV. A temperatura de ebulição do etanol puro é 78,4°C, enquanto que a de uma solução alcoólica 10%
(m/m) em água é 78,2°C.

Das diferenças apresentadas em cada uma das afirmações acima, está(ão) relacionada(s) com
propriedades coligativas:
a) apenas I e III.
b) apenas l.
c) apenas II e III.
d) apenas II e IV.
e) apenas III e IV.

Comentários
As propriedades coligativas são relacionadas à alteração nas propriedades físicas do solvente
devido à adição de um soluto. Com base nisso, vamos analisar as afirmações:

AULA 10 – SOLUÇÕES 166


Prof. Thiago Cardoso

I – Nesse caso, os líquidos estão puros, portanto, não existe nenhum efeito coligativo. Afirmação
errada.
II – Trata-se do efeito ebulioscópico, que depende da concentração molal do soluto. AFirmaçõ
correta.
III – Trata-se do efeito ebulioscópico com o cloreto de sódio. Afirmação correa.
IV – Embora seja uma ligação, o etanol não é um soluto não volátil, portanto, não se trata de um
efeito coligativo como os estudados nesse capítulo. Afirmação errada.

Gabarito: C

(ITA – 2006)
Considere soluções de SiCl4/CCl4 de frações molares variáveis, todas a 25°C. Sabendo que a pressão de
vapor do CCl4 a 25°C é igual a 114,9 mmHg, assinale a opção que mostra o gráfico que melhor representa
a pressão de vapor de CCl4 (PCCl4) em função da fração molar de SiCl4 no líquido ( XSiCl4 )

a) b) c)

b) e)

Comentários
Considerando a solução ideal, a pressão de vapor de um componente qualquer é diretamente
proporcional à sua fração molar.
𝑃𝐶𝐶𝑙4 = 𝑃0 . 𝑥𝐶𝐶𝑙4
Considerando que a solução seja composta apenas por CCl4 e SiCl4, temos que a soma das frações
molares é igual a 1.
𝑃𝐶𝐶𝑙4 = 𝑃0 . (1 − 𝑥𝑆𝑖𝐶𝑙4 )

AULA 10 – SOLUÇÕES 167


Prof. Thiago Cardoso

Sendo assim, a pressão de vapor do solvente decresce linearmente com a pressão de vapor do
soluto. Será igual à sua própria pressão natural de 114,9 mmHg quando puro (x soluto = 0) e decrescerá a
zero quando o soluto ocupar toda a solução (xsoluto = 1).

Gabarito: E

(IME – 2004)
Um calcário composto por MgCO3 e CaCO3 foi aquecido para produzir MgO e CaO. Uma amostra de 2,00
gramas desta mistura de óxidos foi tratada com 100 cm3 de ácido clorídrico 1,00 molar. Sabendo-se que
o excesso de ácido clorídrico necessitou de 20,0 cm3 de solução de NaOH 1,00 molar para ser
neutralizado, determine a composição percentual, em massa, de MgCO 3 e CaCO3 na amostra original
desse calcário.

Comentários
Uma questão bastante interessante sobre a análise de uma mistura. Vamos calcular o número de
mols de ácido clorídrico. Tenha em mente que 1 cm³ = 0,001 L.
𝑖𝑛𝑐𝑖𝑎𝑖𝑠
𝑛𝐻𝐶𝑙 = [𝐻𝐶𝑙]. 𝑉 = 1.0,1 = 0,1 𝑚𝑜𝑙
Porém, houve um excedente que foi titulado com a solução de hidróxido de sódio. Como o HCé
um monoácido e o NaOH é uma monobase, temos que a reação ocorre na proporção 1:1.
𝑠𝑜𝑏𝑟𝑎
𝑛𝐻𝐶𝑙 𝑛𝑁𝑎𝑂𝐻
=
1 1
O número de mols de hidróxido de sódio que foram utilizados na titulação pode ser calculado de
forma semelhante.
𝑛𝑁𝑎𝑂𝐻 = [𝑁𝑎𝑂𝐻]. 𝑉 = 1.0,020 = 0,02 𝑚𝑜𝑙
A quantidade de ácido clorídrico que reagiu com o calcário pode ser obtida como a diferença entre
o que foi inicialmente colocado em meio reacional e o que sobrou.
𝑟𝑒𝑎𝑔𝑖𝑢 𝑖𝑛𝑖𝑐𝑖𝑎𝑖𝑠 𝑠𝑜𝑏𝑟𝑎
𝑛𝐻𝐶𝑙 = 𝑛𝐻𝐶𝑙 − 𝑛𝐻𝐶𝑙
𝑟𝑒𝑎𝑔𝑖𝑢
𝑛𝐻𝐶𝑙 = 0,1 − 0,02 = 0,08 𝑚𝑜𝑙
Agora, vamos anotar as reações do HC com os óxidos derivados do calcário. São reações
características entre óxidos básicos e um ácido forte.
𝑀𝑔𝑂 + 2𝐻𝐶𝑙 → 𝑀𝑔𝐶𝑙2 + 𝐻2 𝑂
𝐶𝑎𝑂 + 2𝐻𝐶𝑙 → 𝐶𝑎𝐶𝑙2 + 𝐻2 𝑂
O número de mols de ácido que reagiram é igual à soma do que reagiu nas duas reações
supracitadas. Podemos escrever que:
𝑟𝑒𝑎𝑔𝑖𝑢
𝑛𝐻𝐶𝑙 = 2. 𝑛𝐶𝑎𝑂 + 2. 𝑛𝑀𝑔𝑂 = 0,08

0,08
∴ 𝑛𝐶𝑎𝑂 + 𝑛𝑀𝑔𝑂 = = 0,04
2

AULA 10 – SOLUÇÕES 168


Prof. Thiago Cardoso

Além disso, sabemos que a amostra tinha 2 gramas. A massa da amostra pode ser escrita como a
soma entre os dois compostos
𝑚𝐶𝑎𝑂 + 𝑚𝑀𝑔𝑂 = 2
As massas de óxido de cálcio e de magnésio podem ser obtidas como o produto entre o número
de mols e a massa molar. As massas molares são:
𝑀𝐶𝑎𝑂 = 40 + 16 = 56 𝑔/𝑚𝑜𝑙
𝑀𝑀𝑔𝑂 = 24 + 16 = 40 𝑔/𝑚𝑜𝑙
Agora, podemos escrever:
𝑛𝐶𝑎𝑂 . 𝑀𝐶𝑎𝑂 + 𝑛𝑀𝑔𝑂 . 𝑀𝑀𝑔𝑂 = 2
𝑛𝐶𝑎𝑂 . 56 + 𝑛𝑀𝑔𝑂 . 40 = 2

56𝑛𝐶𝑎𝑂 + 40 𝑛𝑀𝑔𝑂 = 2

Podemos resolver o sistema de equações destacadas. Para isso, podemos multiplicar a primeira
por 40.
56𝑛𝐶𝑎𝑂 + 40 𝑛𝑀𝑔𝑂 = 2
40𝑛𝐶𝑎𝑂 + 40𝑛𝑀𝑔𝑂 = 1,6
Procedendo à subtração.
56𝑛𝐶𝑎𝑜 − 40𝑛𝐶𝑎𝑂 = 2 − 1,6
16𝑛𝐶𝑎𝑂 = 0,4

0,4
∴ 𝑛𝐶𝑎𝑂 = = 0,025 𝑚𝑜𝑙
16

Podemos calcular também o número de mols de MgO.


𝑛𝐶𝑎𝑂 + 𝑛𝑀𝑔𝑂 = 0,04
0,025 + 𝑛𝑀𝑔𝑂 = 0,04

∴ 𝑛𝑀𝑔𝑂 = 0,04 − 0,025 = 0,015 𝑚𝑜𝑙

Podemos agora observar que o número de mols de carbonato de cálcio (CaCO3) no minério original
é igual ao número de mols de óxido de cálcio (CaO) após o aquecimento. O mesmo é válido para o
carbonato de magnésio (MgCO3).
𝑛𝐶𝑎𝐶𝑂3 = 𝑛𝐶𝑎𝑂
𝑛𝑀𝑔𝐶𝑂3 = 𝑛𝑀𝑔𝑂
A razão para isso é que as reações de decomposição térmica dos carbonatos ocorrem na proporção
1:1.
Δ
𝐶𝑎𝐶𝑂3 (𝑠) → 𝐶𝑎𝑂 + 𝐶𝑂2 (𝑔)
Δ
𝑀𝑔𝐶𝑂3 (𝑠) → 𝑀𝑔𝑂 + 𝐶𝑂2 (𝑔)

AULA 10 – SOLUÇÕES 169


Prof. Thiago Cardoso

Agora, podemos calcular as massas de CaCO3 e de MgCO3 originalmente presentes no minério.


Basta, para isso, multiplicar o número de mols pela massa molar dos carbonatos.
𝑀𝐶𝑎𝐶𝑂3 = 1.40 + 1.12 + 3.16 = 40 + 12 + 48 = 100 𝑔/𝑚𝑜𝑙
𝑀𝑀𝑔𝐶𝑂3 = 1.24 + 1.12 + 3.16 = 24 + 12 + 48 = 84 𝑔/𝑚𝑜𝑙
𝑚𝐶𝑎𝐶𝑂3 = 𝑛𝐶𝑎𝐶𝑂3 . 𝑀𝐶𝑎𝐶𝑂3 = 0,025.100 = 2,5 𝑔
𝑚𝑀𝑔𝐶𝑂3 = 𝑛𝑀𝑔𝐶𝑂3 . 𝑀𝑀𝑔𝐶𝑂3 = 0,015.84 = 1,26 𝑔
A massa total minério original é a soma das massas dos dois carbonatos.
𝑚 = 𝑚𝐶𝑎𝐶𝑂3 + 𝑚𝑀𝑔𝐶𝑂3 = 2,5 + 1,26 = 3,76 𝑔
Com isso, podemos calcular a fração mássica do carbonato de cálcio (CaCO3) na mistura.
𝑚𝐶𝑎𝐶𝑂3 2,5
%𝐶𝑎𝐶𝑂3 = = = 0,665 = 66,5%
𝑚 3,76
O teor de carbonato de magnésio (MgCO3) é, portanto, o que sobra.
%𝑀𝑔𝐶𝑂3 = 100% − 66,5% = 33,5%

Gabarito: CaCO3 = 66,5%; MgCO3 = 33,5%

(IME – 2005)
Determine o abaixamento relativo da pressão de vapor do solvente quando 3,04 g de cânfora (C 10H16O)
são dissolvidos em 117,2 mL de etanol (C a 25oC.
Dado: massa específica do etanol = 0,785 g/mL

Comentários
O abaixamento relativo da pressão de vapor do solvente, dado pela Lei de Raoult, é igual à fração
molar do soluto.

Δ𝑃 𝑛1
= 𝑥1 =
𝑃0 𝑛1 + 𝑛2

Como a fração molar do soluto é calculada a partir do número de mols do soluto e do solvente,
precisamos calculá-lo.
A cânfora, por ter massa molar muito elevada, é considerada um soluto não volátil em etanol.
𝑀𝐶10 𝐻16 𝑂 = 10.12 + 16.1 + 1.16 = 120 + 16 + 16 = 152 𝑔/𝑚𝑜𝑙
O número de mols da cânfora (soluto) pode ser calculado pela razão entre a massa e a massa
molar.
𝑚𝐶10𝐻16 𝑂 3,04
𝑛1 = 𝑛𝐶10 𝐻16 𝑂 = = = 0,02 𝑚𝑜𝑙
𝑀𝐶10 𝐻16 𝑂 152
Para o etanol, devemos utilizar a definição de massa específica para calcular a massa presente na
mistura.

AULA 10 – SOLUÇÕES 170


Prof. Thiago Cardoso

𝑚
𝑑= ∴ 𝑚 = 𝑑𝑉 = 0,785.117,2 ≅ 92 𝑔
𝑉
Para calcular o número de mols de etanol (solvente), devemos calcular a massa molar e proceder
à divisão entre a massa presente na mistura e a massa molar.
𝑀𝐶2 𝐻6 𝑂 = 2.12 + 6.1 + 1.16 = 24 + 6 + 16 = 46 𝑔/𝑚𝑜𝑙
𝑛𝐶2 𝐻6 𝑂 92
𝑛2 = = = 2 𝑚𝑜𝑙
𝑀𝐶2𝐻6 𝑂 46
Agora, podemos calcular a fração molar do soluto.
𝑛1 0,02 0,02
𝑥1 = = = ≅ 0,01 = 1%
𝑛1 + 𝑛2 2 + 0,02 2,02
Gabarito: 1%

(IME – 2004)
Na produção de uma solução de cloreto de sódio em água a 0,90% (p/p), as quantidades de solvente e
soluto são pesadas separadamente e, posteriormente, promove-se a solubilização. Certo dia, suspeitou-
se que a balança de soluto estivesse descalibrada. Por este motivo, a temperatura de ebulição de uma
amostra da solução foi medida, obtendo-se 100,14ºC. Considerando o sal totalmente dissociado,
determine a massa de soluto a ser acrescentada de modo a produzir um lote de 1000kg com a
concentração correta.
Dado: Constante Ebulioscópica da Água = 0,52 °C/molal

Comentários
Podemos primeiramente obter a molalidade real da solução. Para isso, é importante observar que
o cloreto de sódio se dissocia em água.
𝐻2 𝑂
𝑁𝑎𝐶𝑙 (𝑠) → 𝑁𝑎 + (𝑎𝑞) + 𝐶𝑙 − (𝑎𝑞)
Como a dissociação do cloreto de sódio é completa (100%), já que ele é um composto iônico, e são
duas as partículas liberadas na dissociação, temos que o fator de van’t Hoff desse sal é igual a 2.
A temperatura de ebulição da água pura é de 100 °C. Com isso, podemos calcular o efeito
ebuliscópico.
Δ𝑇𝐶 = 100,14 − 100 = 0,14 °𝐶
De posse do aumento da temperatura de ebulição, podemos calcular a molalidade da solução.
Δ𝑇𝐸 = 𝐾𝐸 . 𝑊. 𝑖
0,14 = 0,52. 𝑊. 2
0,14
∴𝑊= ≅ 0,135 𝑚𝑜𝑙/𝑘𝑔
0,52.2
Essa molalidade significa que, em 1 kg de solvente, encontram-se 0,135 mol do soluto. Podemos
calcular a massa em gramas de cloreto sódio presente nessa solução.
𝑀𝑁𝑎𝐶𝑙 = 1.23 + 1.35,5 = 58,5 𝑔/𝑚𝑜𝑙
𝑚𝑁𝑎𝐶𝑙 = 𝑛𝑁𝑎𝐶𝑙 . 𝑀𝑁𝑎𝐶𝑙 = 0,135.58,5 ≅ 7,9 𝑔

AULA 10 – SOLUÇÕES 171


Prof. Thiago Cardoso

A solução desejada tem um percentual de massa de 0,9% de cloreto de sódio. Isso significa que,
em 1 kg de solução, deve-se ter 9 g de soluto e 991 g de solvente.
Na solução original, temos 7,9 g de soluto para cada 1 kg de solvente. Porém, precisamos saber
exatamente a massa de soluto presente nos 991 g característicos da solução desejada.
𝑖𝑛𝑖𝑐𝑖𝑎𝑙
𝑚𝑁𝑎𝐶𝑙 7,9 𝑖𝑛𝑖𝑐𝑖𝑎𝑙
= ∴ 𝑚𝑁𝑎𝐶𝑙 = 7,9.0,991 = 7,83 𝑔
0,991 1
A solução disponível em laboratório tem, portanto, 7,83 g de soluto para cada 991 g de solvente.
É necessário, portanto, acrescentar 1,17 g de soluto para cada 991 g de solvente.
Para o lote de 1000 kg, precisamos, portanto, de 1170 g ou 1,17 kg (basta multiplicar por 1000) de
cloreto de sódio.
Essa quantidade de cloreto de sódio deve ser misturada à seguinte massa de solução.
𝑖𝑛𝑖𝑐𝑖𝑎𝑙
𝑚𝑠𝑜𝑙𝑢çã𝑜 = (7,83 + 991). 1000 = 998,83 𝑘𝑔
Ao adicionar 1,17 kg de cloreto de sódio à solução inicial, consegue 1000 kg de solução, contendo
exatamente 9 kg de cloreto de sódio.

Gabarito: 1,17kg

(IME – 2001)
Uma solução contendo 0,994g de um polímero, de fórmula geral (C2H4)n, em 5,00g de benzeno, tem
ponto de congelamento 0,51ºC mais baixo que o do solvente puro. Determine o valor de n.
Dado: Constante crioscópia do benzeno = 5,10ºC/molal

Comentários
A massa molar do polímero é dada em função de n.
𝑀 = 𝑛. (2.12 + 4.1) = 𝑛. (24 + 4) = 28𝑛
O abaixamento da temperatura de congelamento do solvente é dado pela equação do efeito
criscópico.
Δ𝑇𝐶 = 𝐾𝐶 𝑊. 𝑖
Como o polímero não se dissocia, o seu fator de van’t Hoff é igual a 1.
0,51 = 5,10. 𝑊. 1
0,51
∴𝑊= = 0,1 𝑚𝑜𝑙/𝑘𝑔
5,10
Usando a definição de molalidade, podemos calcular o número de mols presentes do soluto.
𝑛1
𝑊= ∴ 𝑛1 = 𝑊. 𝑚2 = 0,1.0,005 = 5.10−4 𝑚𝑜𝑙
𝑚2
O número de mols é igual à razão entre a massa do polímero e a sua massa molar.
𝑚1 𝑚1 0,994 1,998
𝑛1 = ∴ 𝑀1 = = −4
= = 1998
𝑀1 𝑛1 5.10 10−3

AULA 10 – SOLUÇÕES 172


Prof. Thiago Cardoso

Já havíamos obtido que a massa molar era múltipla de 28.


1998
28𝑛 = 1998 ∴ 𝑛 = = 71
28
O n é exatamente o termo contido na fórmula molecular do polímero em estudo. Portanto, a sua
fórmula molecular é (C2H4)71.
Gabarito: 71

(IME – 2015 – 1ª Fase)


Um pesquisador verificou, em uma determinada posição geográfica, por meio da análise de amostras
de água do mar extraídas do local, que a massa específica média da água do mar era 1,05 g/mL, a
concentração média de espécies dissolvidas era 0,80 mol/L e a temperatura média era de 290 K. O
mesmo pesquisador, com o objetivo de colher água doce em seu estudo, planeja envolver, com uma
membrana semipermeável ideal, uma das extremidades abertas de um longo tubo, a qual será imersa
na água do mar. A que profundidade mínima, em metros, o tubo deveria ser imerso?

a) 1930,0.
b) 183,4.
c) 73,7.
d) 19,4.
e) 9,7.
Dados: R = 0,08 atm.L/(K.mol) = 8,3 J/(K.mol) = 62,3 mmHg.L/(K.mol) ; g = 10 m/s²; 1 atm = 103750 Pa.
Obs.: Essa questão requer o conhecimento de Hidrostática, assunto da Física

Comentários
Como já fornecida a concentração total das espécies, a pressão osmótica pode ser calculada
diretamente por esse valor.

𝜋 = [𝑋]𝑅𝑇 = 0,8.0,08.290 = 18,56 𝑎𝑡𝑚

A coluna de água do mar deve exercer uma pressão equivalente à pressão osmótica de 18,56 atm.
Essa pressão pode ser calculada pela expressão conhecida da Hidrostática.

Na Hidrostática, a pressão é calculada pela expressão.

𝑃𝑐𝑜𝑙𝑢𝑛𝑎 = 𝑑𝑔ℎ
A pressão é obtida nas unidades do SI (Pascal) quando a densidade é fornecida em kg/m³, a
gravidade em m/s² e a altura em m. Portanto, vamos converter a pressão osmótica em Pa e a densidade
para kg/m³.

1,05𝑔 1,05.10−3 𝑘𝑔 1,05.103 𝑘𝑔


𝑑= = =
1 𝑚𝐿 1.10−6 𝑚3 𝑚3
𝜋 = 18,56 𝑎𝑡𝑚 = 18,56.103750 = 1925600 𝑃𝑎

AULA 10 – SOLUÇÕES 173


Prof. Thiago Cardoso

Feitas as conversões de unidades necessárias, podemos calcular a coluna de água.

𝑃𝑐𝑜𝑙𝑢𝑛𝑎 = 1,05.103 . 10. ℎ = 1925600


1925600
∴ℎ= ≅ 183,4 𝑚
1,05.103 . 10
Gabarito: B

(Estratégia Militares – TFC – Inédita)


Uma solução aquosa de 1 L de HC 0,1 mol/L foi titulada com uma solução aquosa de NaOH de mesma
concentração molar. Assinale a alternativa que apresenta a curva de condutância da solução em função
do volume de NaOH adicionado.

a)

b)

c)

AULA 10 – SOLUÇÕES 174


Prof. Thiago Cardoso

d)

e)

Comentários
Perceba que teremos uma reação de neutralização:
𝐻𝐶𝑙 (𝑎𝑞) + 𝑁𝑎𝑂𝐻 (𝑎𝑞) → 𝑁𝑎𝐶𝑙 (𝑎𝑞) + 𝐻2 𝑂 (𝑙)
Então, enquanto houver a neutralização, a condutância da solução decresce, devido a menor
presença de eletrólitos, visto que cada 2 mols de eletrólitos, nos reagentes, resultam 1 mol de eletrólito
nos produtos (sal).
Contudo, ao continuar adicionando NaOH, após o ponto de equivalência, identificado pelo vértice
no gráfico, temos o aumento da condutância, pois o NaOH é eletrólito forte e não está mais sendo
consumido. Esse acréscimo não é linear, porque haverá o efeito da diluição. Esse é o erro da letra A
Quando a quantidade de hidróxido de sódio adicionada tender ao infinito, a concentração será
equivalente a uma solução NaOH 0,1 mol/L.
Gabarito: A

(Estratégia Militares – TFC – Inédita)


O cloreto de potássio é utilizado como sal light, pois partilha muitas propriedades do íon sódio, inclusive
o sabor. A respeito desses dois importantes cátions, assinale a alternativa correta:
a) Quando uma solução de cloreto de potássio é aquecida, libera uma luz violeta, enquanto o íon sódio
libera uma luz amarela.
b) Os sais do potássio geralmente são mais solúveis que os sais de sódio, pois o potássio apresenta
maior raio iônico.

AULA 10 – SOLUÇÕES 175


Prof. Thiago Cardoso

c) É de se esperar que uma solução 1 mol/L de cloreto de potássio seja melhor condutora que uma
solução 1 mol/L de cloreto de sódio, devido às diferenças de mobilidades dos íons.
d) O sódio e o potássio são importantes no mecanismo das contrações musculares, por meio da bomba
de sódio e potássio, mecanismo pelo qual ambos os íons são transferidos por osmose através das
membranas celulares.
e) O íon potássio pode ser facilmente precipitado, borbulhando-se H2S em solução aquosa.

Comentários
Vamos analisar cada assertiva.
a) Verdadeiro. Essa situação é conhecida como teste de chama e é conhecida que a chama do
potássio é violeta, enquanto a do sódio é amarela.
b) Falso. É justamente o contrário, quanto maior o raio, menor a solubilidade, pois enfraquece o
campo elétrico existente na solvatação dos íons, o que define a solubilidade.
c) Falso. Quanto menor o íon, maior a mobilidade iônica, visto que eles têm a mesma carga.
d) Falso. O transporte dos íons é feito através do processo de difusão. A osmose é um tipo de
difusão, onde ocorre a passagem de água através da membrana celular, enquanto na bomba de sódio e
potássio temos a passagem dos próprios íons entre os ambientes de diferentes concentrações.
e) Falso. Regra geral, todos os sais da família IA são solúveis. Assim, o K2S que poderia ser formado
no borbulhamento é um sal solúvel, logo, não haveria precipitação de K+.
Gabarito: A

(Estratégia Militares – TFC – Inédita)


A fim de determinar o número de unidades estruturais de um polímero derivado do polietileno (CH2)n,
uma amostra de 5,6 g foi dissolvida em 100 mL benzeno. Admitindo que a constante crioscópica do
benzeno seja igual 5,10 °C.molal –1 e que o ponto de congelamento da solução foi superior em 0,51 °C
ao ponto de congelamento do benzeno puro, assinale a opção que indica o número de unidades
estruturais do polímero.
Dado: Densidade do benzeno = 0,88 g/cm³
a) 18
b) 30
c) 45
d) 50
e) 60

Comentários:
Pela expressão do Efeito Crioscópico, podemos escrever a variação da temperatura de
congelamento como o produto da constante crioscópica pela molalidade. O polímero é apolar e não se
dissocia, portanto, o seu fator de van’t Hoff é igual a 1.
Δ𝑇𝐶 = 𝐾𝐶 . 𝑊. 𝑖

AULA 10 – SOLUÇÕES 176


Prof. Thiago Cardoso

0,51 = 5,10. 𝑊. 1
0,51
∴𝑊= = 0,1 𝑚𝑜𝑙/𝑘𝑔
5,10
Pela definição, a concentração molal é calculada como a razão entre o número de mols do soluto
e a massa do solvente.
𝑛1
𝑊= = 0,1
𝑚2
A massa do solvente pode ser obtida a partir da densidade. Como foi informado pelo enunciado,
utilizou-se 100 mL de benzeno. Portanto, a massa do solvente é:
𝑚
𝑑 = ∴ 𝑚 = 𝑑𝑉 = 0,88.100 = 88 𝑔 = 0,088 𝑘𝑔
𝑉
Note que, para o cálculo da concentração molal, precisamos da massa do solvente em kg. Agora,
passemos para o número de mols do soluto.
𝑛1
𝑊= ∴ 𝑛1 = 𝑊. 𝑚2 = 0,1.0,088 = 0,0088 𝑚𝑜𝑙
𝑚2
O número de mols do soluto pode ser obtido como a razão entre a sua massa e a sua massa molar.
A massa molar é um múltiplo da massa da fórmula mínima, que é CH2, cuja massa molar é:
𝑀𝐶𝐻2 = 1.12 + 2.1 = 14 𝑔/𝑚𝑜𝑙
A massa molar do polímero será, portanto, igual a 14n.
𝑚1
𝑛1 =
𝑀
5,6
0,0088 =
14𝑛
5,6 560 560 560.9
∴ 14𝑛 = = = =
0,0088 0,88 8 8
9
Nessa conta, usamos a aproximação de que 8/9 = 0,888…
560.9 90
𝑛= = = 45
8.14 2
Gabarito: C

(Estratégia Militares – TFC – Inédita)


A temperatura de ebulição de uma solução aquosa 0,1 mol/L de um ácido orgânico não-volátil HX é
0,026 °C superior à temperatura do benzeno puro. Sabendo que a constante ebulioscópica do solvente
é igual a 0,52 °C.molal-1, determine o valor aproximado da constante de dissociação do ácido HX.
a) 6,0.10-4
b) 9,2.10-4
c) 1,1.10-3
d) 4,4.10-3
e) 7,5.10-3

AULA 10 – SOLUÇÕES 177


Prof. Thiago Cardoso

Comentários:
A elevação da temperatura de ebulição do benzeno pela adição de um soluto não-volátil é dada pela
equação do efeito ebulioscópico.
Δ𝑇𝐸 = 𝐾𝐸 𝑊. 𝑖
A constante ebulioscópica nos foi fornecida. A molalidade, no entanto, deve ser calculada.
𝑛ú𝑚𝑒𝑟𝑜 𝑑𝑒 𝑚𝑜𝑙𝑠 𝑑𝑜 𝑠𝑜𝑙𝑢𝑡𝑜
𝑊=
𝑚𝑎𝑠𝑠𝑎 𝑑𝑜 𝑠𝑜𝑙𝑣𝑒𝑛𝑡𝑒 𝑒𝑚 𝑞𝑢𝑖𝑙𝑜𝑠
Considerando que a solução é diluída, em 1 litro de solução, temos 1 kg de água, portanto, a
molaridade da solução é aproximadamente igual à molalidade.
Agora, vamos aplicar na equação do efeito ebulioscópico.
Δ𝑇𝐸 = 𝐾𝐸 𝑊. 𝑖
0,0572 = 0,52.0,1. 𝑖
Com isso, podemos obter o fator de van’t Hoff.
0,0572
∴𝑖= = 1,1
0,52.0,1
A partir do fator de van’t Hoff, podemos calcular o grau de ionização:
𝑖 =1+𝛼
𝛼 = 𝑖 − 1 = 1,1 − 1 = 0,1
Por fim, podemos calcular a constante de dissociação do ácido
𝛼2 (0,1)2 0,01 10−3
𝐾= .𝑀 = . 0,1 = . 0,1 = ≅ 1,1.10−3
1−𝛼 0,9 0,9 0,9
Gabarito: C

(Estratégia Militares – TFC – Inédita)


A temperatura de ebulição de uma solução aquosa 0,1 mol/L de um ácido orgânico não-volátil HX é
0,026 °C superior à temperatura do benzeno puro. Sabendo que a constante ebulioscópica do solvente
é igual a 0,52 °C.molal-1, determine o valor aproximado da constante de dissociação do ácido HX.
a) 6,0.10-4
b) 9,2.10-4
c) 1,1.10-3
d) 4,4.10-3
e) 7,5.10-3

Comentários:
A elevação da temperatura de ebulição do benzeno pela adição de um soluto não-volátil é dada pela
equação do efeito ebulioscópico.

AULA 10 – SOLUÇÕES 178


Prof. Thiago Cardoso

Δ𝑇𝐸 = 𝐾𝐸 𝑊. 𝑖
A constante ebulioscópica nos foi fornecida. A molalidade, no entanto, deve ser calculada.
𝑛ú𝑚𝑒𝑟𝑜 𝑑𝑒 𝑚𝑜𝑙𝑠 𝑑𝑜 𝑠𝑜𝑙𝑢𝑡𝑜
𝑊=
𝑚𝑎𝑠𝑠𝑎 𝑑𝑜 𝑠𝑜𝑙𝑣𝑒𝑛𝑡𝑒 𝑒𝑚 𝑞𝑢𝑖𝑙𝑜𝑠
Considerando que a solução é diluída, em 1 litro de solução, temos 1 kg de água, portanto, a
molaridade da solução é aproximadamente igual à molalidade.
Agora, vamos aplicar na equação do efeito ebulioscópico.
Δ𝑇𝐸 = 𝐾𝐸 𝑊. 𝑖
0,0572 = 0,52.0,1. 𝑖
Com isso, podemos obter o fator de van’t Hoff.
0,0572
∴𝑖= = 1,1
0,52.0,1
A partir do fator de van’t Hoff, podemos calcular o grau de ionização:
𝑖 =1+𝛼
𝛼 = 𝑖 − 1 = 1,1 − 1 = 0,1
Por fim, podemos calcular a constante de dissociação do ácido
𝛼2 (0,1)2 0,01 10−3
𝐾= .𝑀 = . 0,1 = . 0,1 = ≅ 1,1.10−3
1−𝛼 0,9 0,9 0,9
Gabarito: C

(Estratégia Militares – TFC – Inédita)


O nitrogênio em um aquário marinho pode se encontrar na forma de amônia, nitrito, nitrato ou de
moléculas orgânicas. Supondo que o aquário tenha 6,2 mg/L de nitrato e 11 mg/L do elemento
nitrogênio no total, determine o percentual de nitrogênio que se encontra na forma de nitrato:
a) 12,7%
b) 25,4%
c) 32,5%
d) 40,0%
e) 56,3%

Comentários

A concentração molar de nitrogênio proveniente do nitrato pode ser representada por [N] 1. Essa
concentração pode ser obtida pelo teor de nitrogênio no nitrato.

[𝑁]1 = 𝑡𝑒𝑜𝑟 𝑑𝑒 𝑛𝑖𝑡𝑟𝑜𝑔ê𝑛𝑖𝑜 ⋅ [𝑁𝑂3− ]

Para obter o teor de nitrogênio no íon nitrato, precisamos calcular a massa molar do íon nitrato.

𝑀𝑁𝑂3− = 1.14 + 3.16 = 62 𝑔/𝑚𝑜𝑙

AULA 10 – SOLUÇÕES 179


Prof. Thiago Cardoso

Da massa de 62 g/mol, note que 14 g/mol corresponde à fração do nitrogênio. Portanto, o teor de
nitrogênio é igual à razão 14/62. Portanto, temos que a concentração do elemento proveniente do nitrato
é:

14
∴ [𝑁]1 = ⋅ [𝑁𝑂3− ]
62
14
∴ [𝑁]1 = ⋅ 6,2 = 14.0,1 = 1,4 𝑚𝑔/𝐿
62
Portanto, a concentração 1,4 mg/L de nitrogênio é proveniente do nitrato. No entanto, a água
contém 11 mg/L. Dessa forma, a fração pedida no enunciado é:

𝑛𝑖𝑡𝑟𝑜𝑔ê𝑛𝑖𝑜 𝑝𝑟𝑜𝑣𝑒𝑛𝑖𝑒𝑛𝑡𝑒 𝑑𝑜 𝑛𝑖𝑡𝑟𝑎𝑡𝑜 1,4


%𝑁 = = ≅ 0,127 = 12,7%
𝑛𝑖𝑡𝑟𝑜𝑔ê𝑛𝑖𝑜 𝑡𝑜𝑡𝑎𝑙 11

Gabarito: A

(Estratégia Militares – TFC – Inédita)


Uma solução de 20,0 g de qual sal hidratado dissolvido em 200 g H 2O terá o menor ponto de
congelamento?
a) CuSO4 . 5 H2O (M = 250)
b) NiSO4 . 6 H2O (M = 263)
c) MgSO4 . 7 H2O (M = 246)
d) Na2SO4 . 10 H2O (M = 286)
e) Todas as soluções apresentam o mesmo ponto de congelamento.

Comentários
A solução que terá o menor ponto de congelamento é aquela que tiver o menor número de
partículas em solução, pois lembremos que é justamente disso que se trata a crioscopia: abaixamento da
temperatura de fusão em decorrência da adição de um soluto não volátil.
Outra linha de raciocínio seria pensar na fórmula em si:
Δ𝑇 = 𝐾𝑐 . 𝑊. 𝑖
Em que Kc é uma constante que depende apenas do solvente; W é a molalidade do soluto e i é o
fator de van't Hoff.
Para obtermos a quantidade de partículas em solução, basta que encontremos o número de mols
que há em 20,0 g do sal e multiplicar pela quantidade de íons que estará em solução quando o sal for
dissolvido. Então analisar cada uma das alternativas.
É importante ressaltar que a água de cristalização não exerce nenhuma influência no ponto de
congelamento, pois ela não é um soluto. Em vez disso, ela se misturará ao solvente da solução.
a) 𝐶𝑢𝑆𝑂4 (𝑎𝑞 ) → 𝐶𝑢2+ (𝑎𝑞 ) + 𝑆𝑂42− (𝑎𝑞)
Para essa dissolução, o fator de Van’t Hoff é 2, uma vez que cada molécula de sal dissolvida gera
2 partículas de íons em solução.

AULA 10 – SOLUÇÕES 180


Prof. Thiago Cardoso

𝑚 20,0
𝑛= = = 0,08 𝑚𝑜𝑙 𝑑𝑜 𝑠𝑎𝑙
𝑀 250
Logo, o número de partículas formadas pela dissolução é:
𝑁 = 2.0,08 = 0,16 𝑚𝑜𝑙 𝑑𝑒 𝑝𝑎𝑟𝑡í𝑐𝑢𝑙𝑎𝑠
b) Analogamente ao item anterior, temos:

𝑁𝑖𝑆𝑂4 (𝑎𝑞) → 𝑁𝑖 2+ (𝑎𝑞) + 𝑆𝑂42−


𝑖=2
20,0
𝑛= = 0,076 ∴ 𝑁 = 𝑖. 𝑛 = 2.0,076 = 0,152 𝑚𝑜𝑙 𝑑𝑒 𝑝𝑎𝑟𝑡í𝑐𝑢𝑙𝑎𝑠
263
c) Reação:
𝑀𝑔𝑆𝑂4 (𝑎𝑞) → 𝑀𝑔2+ (𝑎𝑞) + 𝑆𝑂42−
𝑖=2
20,0
𝑛= = 0,081 ∴ 𝑁 = 𝑖. 𝑛 = 2.0,081 = 0,162 𝑚𝑜𝑙 𝑑𝑒 𝑝𝑎𝑟𝑡í𝑐𝑢𝑙𝑎𝑠
246
d) Reação:
𝑁𝑎2 𝑆𝑂4 (𝑎𝑞) → 2 𝑁𝑎 + (𝑎𝑞) + 𝑆𝑂42−
𝑖=3
20,0
𝑛= = 0,07 ∴ 𝑁 = 𝑖. 𝑛 = 3.0,07 = 0,21 𝑚𝑜𝑙 𝑑𝑒 𝑝𝑎𝑟𝑡í𝑐𝑢𝑙𝑎𝑠
286
Portanto, temos que o sal que terá o menor ponto de congelamento será o sulfato de sódio, da
letra D, por produzir uma maior quantidade de partículas livres quando dissolvido em água.
Gabarito: D

(Estratégia Militares – TFC – Inédita)


Uma luminária caiu em um aquário marinho liberando íons estanhosos em solução. A fim de detectar a
quantidade desses íons que foram dissolvidos, é possível utilizar uma mistura permanganato de
potássio e ácido sulfúrico como agente titulante.
Sabendo o manganês se reduz a íons Mn2+ incolores e que para neutralizar uma amostra de 100 mL da
solução aquosa foi utilizado 1 mL de uma solução aquosa 0,001 mol/L de permanganato de potássio,
assinale a alternativa que indica a concentração aproximada em massa de íons Sn2+ presentes na
solução.
a) 1,2 mg/L
b) 1,8 mg/L
c) 2,0 mg/L
d) 2,4 mg/L
e) 3,0 mg/L

Comentários
Primeiramente, vamos escrever a reação, notando que:
• O estanho se oxida de Sn2+ a Sn4+;

AULA 10 – SOLUÇÕES 181


Prof. Thiago Cardoso

• O manganês se reduz de +7 em MnO4– para +2 em Mn2+;


• O hidrogênio (H+) permanece com o mesmo número de oxidação nos produtos, portanto, forma
H2O.
𝑆𝑛2+ (𝑎𝑞) + 𝑀𝑛𝑂4− (𝑎𝑞) + 𝐻 + → 𝑆𝑛4+ (𝑎𝑞) + 𝑀𝑛2+ (𝑎𝑞) + 𝐻2 𝑂
Observe que o estanho perde 2 elétrons, enquanto o manganês ganha 5 elétrons. Por esse motivo,
podemos utilizar a técnica de inversão dos coeficientes, temos:
𝟓 𝑆𝑛2+ (𝑎𝑞) + 𝟐 𝑀𝑛𝑂4− (𝑎𝑞) + 𝐻 + → 𝑆𝑛4+ (𝑎𝑞) + 𝑀𝑛2+ (𝑎𝑞) + 𝐻2 𝑂
Observe que balancear esse pedaço da reação é suficiente, pois queremos apenas calcular a
concentração de íons Sn2+. Pela proporção estequiométrica, podemos escrever:
𝑛𝑆𝑛2+ 𝑛𝑀𝑛𝑂4−
=
5 2
O número de mols pode ser obtido como o produto da concentração pelo volume da solução.
[𝑆𝑛2+ ]. 𝑉𝑆𝑛2+ [𝑀𝑛𝑂4− ]. 𝑉𝑀𝑛𝑂4−
=
5 2
Pela proporção estequiométrica, podemos calcular o número de mols de CO2 produzidos:
[𝑆𝑛2+ ]. 100 0,001.1
=
5 2
2+
Portanto, a concentração de íons Sn é:
5.0,001.1
∴ [𝑆𝑛2+ ] = = 2,5.10−3−2 = 2,5.10−5 𝑚𝑜𝑙/𝐿
2.100
Por fim, basta multiplicar pela massa molar do estanho para obter a concentração em massa do
íon na solução.
𝐶 (𝑆𝑛2+ ) = [𝑆𝑛2+ ]. 𝑀 = 2,5.10−5 . 118
𝐶(𝑆𝑛2+ ) = 295.10−5 = 2,95.10−3 ≅ 3.10−3 = 3 𝑚𝑔/𝐿
Gabarito: E

(Estratégia Militares – TFC – Inédita)


Determine a condutividade de uma solução aquosa 0,2 mol/L de sulfato de sódio em S.m –1.
Dados: σ (SO42–) = 160 S cm² mol–1; σ (Na+) = 50 S cm² mol–1
a) 4,2
b) 5,2
c) 6,2
d) 52
e) 62

AULA 10 – SOLUÇÕES 182


Prof. Thiago Cardoso

Comentários

Primeiramente, devemos calcular o valor da concentração do sódio e do sulfato, observando que,


por força da estequiometria do sal, existem 2 mols de sódio para cada mol de sal.

[𝑁𝑎 + ] = 2 . 0,2 = 0,4 𝑚𝑜𝑙/𝐿

[𝑆𝑂4−2 ] = 0,2 𝑚𝑜𝑙/𝐿

Como as condutividades específicas foram fornecidas em cm, devemos converter a expressão para

0,4 𝑚𝑜𝑙 0,4 𝑚𝑜𝑙


[𝑁𝑎+ ] = = = 4.10−4 𝑚𝑜𝑙. 𝑐𝑚−3
1𝐿 1000 𝑐𝑚3
0,2 𝑚𝑜𝑙 0,2 𝑚𝑜𝑙
[𝑆𝑂4−2 ] = = = 2.10−4 𝑚𝑜𝑙. 𝑐𝑚−3
1𝐿 1000 𝑐𝑚³
Para a expressão da condutividade da solução aquosa, devemos utilizar a soma das condutividades
de cada íon, que podem ser obtidas como o produto da condutividade específica pela concentração molar
do íon.

𝜎 = σ (𝑆𝑂4−2 ). [𝑆𝑂4−2 ] + σ (𝑁𝑎+ ). [𝑁𝑎+ ]

𝜎 = 160 . 2.10−4 + 50 . 4.10−4

𝜎 = 320.10−4 + 200.10−4 = 520.10−4 = 5,2.10−2 𝑆 𝑐𝑚−1

Devemos agora converter para S.m–1:

5,2.10−2 𝑆 5,2.10−2 𝑆
𝜎= = = 5,2 𝑆. 𝑚−1
1 𝑐𝑚 10−2 𝑚
Gabarito: B

(Estratégia Militares – TFC – Inédita)


Uma amostra de prata metálica foi tratada por excesso de ácido sulfúrico concentrado a quente,
consumindo 24,5 cm³ de uma solução aquosa, cuja densidade é igual a 1,75 g/cm³ e concentração de
80% em massa. Assinale a alternativa que indica a massa de gás liberada nessa reação.
a) 0,7 g
b) 1,4 g
c) 2,8 g
d) 11,2 g
e) 22,4 g

AULA 10 – SOLUÇÕES 183


Prof. Thiago Cardoso

Comentários

A prata metálica pode ser oxidada pelo ácido sulfúrico. Porém, é importante notar que o ácido
sulfúrico é reduzido, produzindo SO2, e não H2. Como a prata é um metal nobre, jamais ela poderia
produzir gás hidrogênio por deslocamento.
𝐴𝑔 (𝑠) + 𝐻2 𝑆𝑂4 (𝑎𝑞) → 𝐴𝑔2 𝑆𝑂4 (𝑎𝑞) + 𝑆𝑂2 (𝑔) + 𝐻2 𝑂 (𝑙)
Agora, vamos balancear a reação. Para isso, devemos notar que a prata é oxidada de 0 a +1,
perdendo 1 elétron. Por outro lado, o enxofre tem seu número de oxidação reduzido de +6 (em H2SO4)
para +4 (em SO2). Então, usando a técnica de inversão dos coeficientes, teremos:
2 𝐴𝑔 (𝑠) + 𝐻2 𝑆𝑂4 (𝑎𝑞) → 𝐴𝑔2 𝑆𝑂4 (𝑎𝑞) + 1 𝑆𝑂2 (𝑔) + 𝐻2 𝑂 (𝑙)
Dessa forma, são dois átomos de enxofre nos produtos. Logo, precisamos de dois átomos de
enxofre nos reagentes.
2 𝐴𝑔 (𝑠) + 2 𝐻2 𝑆𝑂4 (𝑎𝑞) → 𝐴𝑔2 𝑆𝑂4 (𝑎𝑞) + 1 𝑆𝑂2 (𝑔) + 𝐻2 𝑂 (𝑙)
Dessa forma, podemos escrever pela proporção estequiométrica, teremos:
𝑛𝐻2 𝑆𝑂4 𝑛𝑆𝑂2 𝑛𝐻 𝑆𝑂
= ∴ 𝑛𝑆𝑂2 = 2 4
2 1 2
Agora, vamos calcular o número de mols de H2SO4 que foram utilizados na reação. Para isso,
notemos que a massa utilizada da solução pode ser calculada a partir de sua densidade.
𝑚
𝑑 = ∴ 𝑚 = 𝑑𝑉 = 1,75.24,5
𝑉
Vamos deixar a conta assim, pois ela será bastante facilitada no próximo passo. Notemos que a
massa de ácido sulfúrico pode ser obtida como 80% da massa total da solução.
𝑚𝐻2 𝑆𝑂4 = 0,80. 𝑚 = 0,80.1,75.24,5 = 1,4.24,5 = 34,3 𝑔
Podemos, agora, calcular o número de mols de ácido sulfúrico dividindo-se a massa total pela
massa molar do ácido, que pode ser calculada somando-se as massas de cada elemento químico que o
constitui.
𝑀𝐻2 𝑆𝑂4 = 2.1 + 1.32 + 4.16 = 98 𝑔/𝑚𝑜𝑙
E, agora, o número de mols pode ser obtido como a razão entre a massa da amostra e a massa
molar da substância.
𝑚𝐻2 𝑆𝑂4 34,3
𝑛𝐻2 𝑆𝑂4 = = ≅ 0,35 𝑚𝑜𝑙
𝑀𝐻2 𝑆𝑂4 98
Dessa forma, podemos calcular o número de mols de SO2 liberados na reação.
𝑛𝐻2 𝑆𝑂4 0,35
𝑛𝑆𝑂2 = = 𝑚𝑜𝑙
2 2
Por fim, a massa de gás produzida pode ser obtida como o produto do número de mols à massa
molar.
𝑀𝑆𝑂2 = 1.32 + 2.16 = 64 𝑔/𝑚𝑜𝑙
0,35
𝑚𝑆𝑂2 = 𝑛𝑆𝑂2 . 𝑀𝑆𝑂2 = ⋅ 64 = 0,35.32 = 11,2 𝑔
2
Gabarito: D

AULA 10 – SOLUÇÕES 184


Prof. Thiago Cardoso

(Estratégia Militares – TFC – Inédita)


Os corais do gênero Acropora são extremamente sensíveis à presença de íons nitrato, resultantes do
ciclo do nitrogênio na água. Para a manutenção da saúde desses animais, é necessário que a água tenha
um teor limite de até 5 mg/L, sendo mais adequado níveis abaixo de 1 mg/L
Uma forma simples de se controlar o teor de nitrato em aquários marinhos é com o uso da vodka, que
pode ser considerada como uma solução aquosa com teor de 40% de etanol em volume.
O etanol é capaz de provocar a redução dos íons nitrato a nitrogênio gasoso. Sabendo que um aquário
de 400 litros tinha inicialmente um teor de 4,96 mg/L de nitrato e o que aquarista deseja reduzir a 1,24
mg/L, determine o volume de vodka a ser adicionado.
Dado: densidade do etanol = 0,8 g/mL.

Comentários
Vamos escrever a reação de oxidação o etanol (C2H6O) a ácido acético (C2H4O2) em meio ácido.
𝐶2 𝐻6 𝑂 (𝑎𝑞) + 𝑁𝑂3− (𝑎𝑞) + 𝐻 + (𝑎𝑞) → 𝐶2 𝐻4 𝑂2 (𝑎𝑞) + 𝑁2 (𝑔) + 𝐻2 𝑂 (𝑙)
O número de oxidação do carbono no etanol é igual a –1. Já, no ácido acético, o seu número de
oxidação passou a ser igual a 0.

Usando a técnica de inversão dos coeficientes, podemos colocar o coeficiente 10 no C 2H6O e o


coeficiente 2 em N2. Porém, podemos também simplificar os coeficientes por 2, tornando 5 e 1.

Vamos balancear os átomos de carbono e nitrogênio de cada lado.

Note que podemos parar por aqui, pois já encontramos a proporção entre etanol e nitrato na
reação. Agora, basta utilizar a proporção estequiométrica para determinar a proporção molar entre os
dois reagentes.
𝑛𝐶2𝐻6 𝑂 𝑛𝑁𝑂3− 5
= ∴ 𝑛𝐶2𝐻6 𝑂 = ⋅ 𝑛𝑁𝑂3−
5 2 2
Vamos calcular, agora, o número de mols de nitrato que precisam ser consumidos nesse aquário.
Note o abaixamento necessário de concentração é:
𝛥𝐶 [𝑁𝑂3− ] = 4,96 − 1,24 = 3,72 𝑚𝑔/𝐿
A concentração de nitrato foi abaixada em 3,72 mg/L. Podemos calcular a variação da
concentração molar dividindo-se pela massa molar:

AULA 10 – SOLUÇÕES 185


Prof. Thiago Cardoso

3,72.10−3
Δ[𝑁𝑂3− ] = = 0,06.10−3 = 6.10−5 𝑚𝑜𝑙
62
Agora, vamos calcular o número de mols consumido multiplicando pelo volume da solução.
𝑛(𝑁𝑂3− ) = 𝛥[𝑁𝑂3− ]. 𝑉 = 6.10−5 . 400 = 24.10−3 𝑚𝑜𝑙
Dessa forma, o número de mols de etanol necessários para o ambiente reacional é:
5 5
𝑛𝐶2𝐻6 𝑂 =⋅ 𝑛𝑁𝑂3− = ⋅ 24 ⋅ 10−3 = 60.10−3 𝑚𝑜𝑙 = 6.10−2 𝑚𝑜𝑙
2 2
Para calcular a massa de etanol, precisamos multiplicar pela massa molar.
𝑀𝐶2𝐻6 𝑂 = 2.12 + 6.1 + 1.16 = 46 𝑔/𝑚𝑜𝑙
A massa de etanol é igual ao número de mols multiplicado pela massa molar.
𝑚𝐶2𝐻6 𝑂 = 𝑛𝐶2𝐻6 𝑂 . 𝑀𝐶2 𝐻6 𝑂 = 6.10−2 . 46 = 276.10−2 = 2,76 𝑔
O volume de etanol pode ser obtido a partir da densidade.
𝑚 𝑚
𝑑= ∴𝑉=
𝑉 𝑑
𝑚𝐶2𝐻6 𝑂 2,76
𝑉𝐶2𝐻6 𝑂 = = = 3,45 𝑚𝐿
𝑑𝐶2𝐻6 𝑂 0,8
Considerando que o etanol corresponde a 40% do volume da vodka, temos:
0,40. 𝑉 = 𝑉𝐶2𝐻6𝑂 = 3,45 𝑚𝐿
3,45
∴𝑉= = 8,625 𝑚𝐿
0,40
O aquarismo é realmente um sistema bastante preciso. Pequenas doses fazem grandes diferenças.
Gabarito: 8,625 mL

(Estratégia Militares – TFC – Inédita)


O iodeto de potássio é um sal incolor e bastante solúvel em água, comercializado na forma de uma
solução aquosa, vendida comercialmente como lugol. Ele pode ser utilizado para a detecção de íons
chumbo (Pb2+).
Uma forma interessante de visualizar essa reação é utilizando um recipiente cilíndrico raso, como uma
placa de Petri, em que foi colocada uma fina lâmina de água suficiente apenas para cobrir o fundo do
recipiente. Nesse recipiente, coloca–se em dois pontos diametralmente opostos da placa uma pequena
quantidade de soluções bastante concentradas de nitrato de chumbo e de iodeto de potássio.
Nessa situação, forma-se uma listra amarela. A respeito dessa situação:
a) escreva a reação química que explica a formação da substância amarela citada.
b) a listra amarela se formará mais perto do ponto em que foi gotejada a solução de nitrato de chumbo
ou do ponto em que foi gotejada a solução de iodeto de potássio? Faça um esboço de como se
apresentará o sistema final.

AULA 10 – SOLUÇÕES 186


Prof. Thiago Cardoso

Comentários
a) A mistura dos dois sais formará um sal insolúvel, que é o iodeto de chumbo (PbI 2), que é um
precipitado amarelo.
𝑃𝑏(𝑁𝑂3 )2 (𝑎𝑞) + 2 𝐾𝐼 (𝑎𝑞) → 𝑃𝑏𝐼2 (𝑠) + 2 𝐾𝑁𝑂3 (𝑎𝑞)
b) Observe que a reação de precipitação ocorre essencialmente entre os íons chumbo (Pb 2+) e
iodeto (I–).
Como o chumbo é mais pesado que o iodo, os íons chumbo serão mais lentos. Portanto, a listra se
formará mais perto dos íons Pb2+.

Gabarito: discursiva

(Estratégia Militares – TFC – Inédita)


Em um experimento de laboratório, conectou-se um led (diodo emissor de luz) a uma bateria e dois
eletrodos imersos em 100 mL de uma solução aquosa de cloreto de potássio 0,1 mol/L.
A intensidade da luz emitida pelo LED é aproximadamente proporcional à raiz quadrada da
condutividade dessa solução aquosa. Por isso, pode-se utilizar uma proveta para adicionar um certo
volume de solução 0,8 mol/L de sulfato de prata.

Desenhe um gráfico da intensidade da luz emitida pelo LED em função da quantidade de solução de
hidróxido de cálcio adicionado, respondendo aos seguintes itens:

AULA 10 – SOLUÇÕES 187


Prof. Thiago Cardoso

a) Qual o volume adicionado de Ag2SO4 quando a intensidade luminosa do led é mínima?


b) Qual a relação aproximadamente entre a intensidade do led quando o volume acrescentado da
solução de Ag2SO4 for muito grande e a intensidade inicial do led?

Comentários
Quando se começa a adicionar a solução de sulfato de prata à solução de cloreto de prata,
inicialmente, verifica-se a formação de um sal insolúvel, que é o cloreto de prata.
2 𝐾𝐶𝑙 (𝑎𝑞) + 𝐴𝑔2 𝑆𝑂4 (𝑎𝑞) → 2 𝐴𝑔𝐶𝑙 (𝑠) + 𝐾2 𝑆𝑂4 (𝑎𝑞)
Devido à formação de sal insolúvel, forma-se uma solução mais diluída em íons.
A condutividade elétrica da solução será mínima no exato momento em que se completar a
neutralização. Podemos calcular esse volume pela lei da titulação. Primeiramente, vamos escrever que o
número de mols de KCl e Ag2SO4 segue a proporção estequiométrica.
𝑛𝐾𝐶𝑙 𝑛𝐴𝑔2 𝑆𝑂4
=
2 1
O número de mols de cada sal pode ser obtido como o produto da concentração molar do sal pelo
volume da solução adicionado.
[𝐾𝐶𝑙]. 𝑉𝐾𝐶𝑙 [𝐴𝑔2 𝑆𝑂4 ]. 𝑉𝐴𝑔2𝑆𝑂4
=
2 1
[𝐾𝐶𝑙] 𝑉𝐾𝐶𝑙 0,1 100
∴ 𝑉𝐴𝑔2 𝑆𝑂4 = ⋅ = ⋅ = 6,25 𝑚𝐿
[𝐴𝑔2 𝑆𝑂4 ] 2 0,8 2
Observe que, como a concentração de Ag2SO4 é igual a 0,8 mol/L e o sal tem ainda 3 mols de íons
por fórmula, essa solução terá 2,4 mol/L de íons. Já a solução inicial de KCl é 0,1 mol/L e o sal tem 2 mols
de íons por fórmula, logo, essa solução tem 0,2 mol/L. Dessa forma, a razão entre as concentrações de
íons:
2,4
𝑟= = 12
0,2
Portanto, é de se esperar que a intensidade do led multiplique pela raiz quadrada de 12, ou seja,
por aproximadamente 3,46.
Dessa forma, podemos traçar o gráfico da intensidade do led.

AULA 10 – SOLUÇÕES 188


Prof. Thiago Cardoso

Gabarito: E

8. Considerações Finais

Chegamos ao final de mais uma aula.


O assunto de Soluções tem uma teoria muito curta, porém, você precisa estar bem afiado, porque
ele é cobrado em altíssimo nível e com muita frequência pelas provas do ITA e IME.
Não hesite em entrar em contato pelo Fórum de Dúvidas. Suas dúvidas são muito importantes não
só para você, mas também para mim, pois elas me ajudam a melhorar esse material.
Também se sinta livre para falar sobre o que você gostou desse curso e o que você não gostou,
pois nós buscaremos melhorar.
Bons estudos para você e até a nossa próxima aula.
Continue devorando esse material. Seu esforço valerá a pena

AULA 10 – SOLUÇÕES 189

Você também pode gostar